Ευκλειδης Β 41

84

Upload: demi-de

Post on 27-Mar-2016

234 views

Category:

Documents


6 download

DESCRIPTION

 

TRANSCRIPT

Page 1: Ευκλειδης Β 41
Page 2: Ευκλειδης Β 41
Page 3: Ευκλειδης Β 41

Υ πcύΒυvοι Έκδοση-; Ευσταθίου Ευάγγελος, Μπαραλής Γεώργιος, Στρατής Ιωάννης

Συvιακηκή Ομάδα: Βακαλόπουλος Κώστας Βάρναλης Νίκος Γιαννοσπύρος Σωτήρης Δούναβης Αντώνης Καλίκ:ας Σταμάτης Καρκ:άνης Βασίλης Κατσοόλης Γιώργος Κερασαρίδης Γιάννης Κηπουρός Χρήστος Κόντζιας Νίκος Κυριακόπουλος Αντώνης Λαζαρίδης Χρήστος Λουρίδας Σωτήρης Μέιλαφέκ:ας θανάσης Μαρούλη Βιολέτα Παπαμικ:ροόλης Αντώνης Πισπινής Δημήτρης ΣαϊτηΕόα ' Σακελλάρης Βασίλης Τασσόπουλος Γιώργος Τσικ:αλουδάχης Γιώργος Τσιούμας θανάσης Χαραλαμποποόλου Λίνα Χαραλάpπους θάνος Χριστόπουλος Παναγιώτης

Σuveρyάιc-;;: Δαμουλή Ευαγγελία Φελοόρης Αργόρης Παπαϊωάννου Α Γιαννακόπουλος Σπύρος Δάφνης Δημήτριος Δάφνης Σπύρος

ΕΚΔΟΣΗ τι-ΙΣ ΕΛΛΠΝΙΚΠΣ ΜΑΘΗΜΑΊΊΚΗΣ ΕΤΑΙf>ΕΙΑΣ ΠΑΝΕΓΠΣτΗΜΙΟΥ 34 - 106 79 ΑθΗΝΑ Τη�36 17 784- 3616 532 Fa:x: 36 41 02.5 Ειι:δόιη�: Αλεξανδρής Νικ:όλασς ΛιευlJυνιή�: Τυρλής Ιωάννης

ISSN: 1105-7998

Ειιιμέkια· Έιι:δοοης: Μαραyκάκης· ΣτέλιΘς

ΣΥΝΛΙ>ΟΜΕΣ: Τεύχοg 700 δρχ

ΕΛΛΗΝΙΚΗ ΜΑθΗΜΑΤΙΚΗ ETAIPEIA Τεύχος 41 • ΙούJ\ιος Αύyοuστος Σεπτέμβριος • 2001 • δρχ. 700 Ευrο: 2,05

e-mail: [email protected] www.hms.gr

ΜΑΘΗΜΑ ηιω ΠΕΡΙΟΔΙΙ<Ο ΠΑ ΤΟ Λ YKEIO

ΠΕΡΙΕΧΟΜΕΝΑ 3 Αρχαία Ελληνικά ΜαΟημαιικά

6 Το Βήμα ιου Ευκλείδη

9 Ο 'Ευκλείδη�· προιείνει ... Ευκλείδη και ΔιόφαvιοΤ

12 ΜαΟημαιι.κ:οί Διαyωvισμοί - ΜαΟημαιικέ� Ολυμπιάδε�

22 Ειοοyωyή σιη θεωρία Γραφημάιωv

28 Η σιήλη ιη� Αλληλοyραφία�

30 Η σιήλη ιου ΜαΟηιή

I ΜαDημαιικά yια ιηv Α' Τά�η ιου Λυκείου I 32 Τριyωνομειρικό� Κύκλο�

35 Απόλυιε� Τιμέ�

39 Αποδεικιικ� ΜέΟοδόι (Συνεπαyωyή�)

42 Αvισόιηιε�

I Μα8ημαιικά yια ιηv Β' Τάξη ιου Λuιreίou I 45 Τριyωνομειρικ� Συναριήσει�

49 Ομοιόιηια

52 Γεωμειρία Γενική� Παιδεία�

55 Ο W. Hanύlton 1843 ιο ονόμασε Διάνυσμα (VECTOR)

I Μα8ημαιικά yια ιηv Γ' Τάξη ιοu Λυιreίου I

59 Διαφορικό� Λοyισμό� Γενική� Παιδεία�

65 Σχειική Οέση Γραφικών Παρασιάσεων δυο Συναριήσεων

69 Μερικ� Επισημάνσει� πάνω σιου� Μιyαδικού� Αριθμού�

7 4 Μέyισια και ελάχισια μέιρων και ορισμάιων Μιyαδικών

- �·. · 78 ΗΟΜΟ MATHEMAτiCUS.

Ετήσια. συνδροpή 2.800 δρχ (Σχολ) ΣυνδροΙDl.τέ!; 2,800+600 δρχ (Τ αχ.) 1------------------------------1 Ορyαv'ιομοί: 5.()(Χ) δρχ ΣιοιχειοΟεοία - Σελιδοποίηση Ταs. Ειιηαy(� Τ. Ι'ικιφt<iο AOήvu 54, Τ.θ: :i0044 Ελληvικι\ Μαθηματική Εταιρεία

Εκτύπωση ΙΝΊ'ΕΡΙΙΡF.Σ A..F�, Ιερά οδός 81- 83

ΥπευΟ. Τuποyραιpείοu Ν. Αδάκτυλο!;-Τηλ:3474654

Page 4: Ευκλειδης Β 41

11 λ.ΡΧλ.Ιλ. θλλΗΝΙΙ<λ. Νlλ.θΗΜλ.ΤΙΙ<λ.ιι IJ Αγrιφriιν 1 ιιΙJφιιιriιs

, κ•ι το '''' rou

ο Α ντιφών έζησε στην Αθήνα το πρώτο μισό

του 5ου π.Χ. αιώνα. Φαίνεται πως ήταν σΊΥγχρονος του Σωκράτη, δεν ξέρουμε όμως

πότε γεννήθηκε. Ο Ξενοφών (Απομν., I, IV, 1 και 11 ), αναφερόμενος στις συνομιλίες του σοφιστή με το Σωκράτη, τον ονομάζει «Αντιφώντα σοφιστήν)). Στο λεξικό «Σούδω) ονομάζεται «Αντιφών Αθηναί­ος, τερατοσκόπος και εποποιός και σοφιστής και 'λιJ­γομάγειροφ. Στοιχεία για τον Αντιφώντα αναφέρο­νται στον Αριστοτέλη, στον Ερμογένη, στο Γαληνό, στο Φιλόστρατο, στον Ωριγένη, στο Στοβαίο, στο Φώτιο, στο Θεμίστιο, στον Πρόκ'λιJ, κ.ά .. Ο Πλάτων όμως δεν αναφέρει τίποτα για τον σοφιστή και το έρ­γο του.([5], σελ.114). Από τις υπάρχουσες πληροφο­ρίες διαφαίνεται ότι υπήρξε ο πρώτος Αττικός σοφι­στής συγγραφέας, με την έννοια του επαγγελματία δασκά'λι:Jυ των νέων και προίσταται της ομάδας των Σοφιστών μαζί με τους Πρωταγόρα και Γοργία κα­θώς και ότι κατείχε πολύ καλά τις βασικές αρχές της Ιωνικής και Ελεατικής φι'λι:Jσοφίας.

Οι γνώμες για τον Αντιφώντα διίστανται. Συ­γκεκριμένα γίνεται σύγχυση ανάμεσα στο σοφιστή Α ντιφώντα και στο σΊΥγχρονό του ρήτορα και πολιτι­κό Αντιφώντα. Ο Alfred Croiset με αφορμή τα στοι­χεία που βρέθηκαν στην Αίγυπτο από χαμένα έργα του Αντιφώντα υποστήριξε πως δεν υπήρξαν δύο διανοούμενοι με το ίδιο όνομα, ο πολιτικός και ο φι­λόσοφος, αλλά ένας, ο πολιτικός. Τα επιχειρήματα αυτά δεν είναι καθό'λι:Jυ πειστικά, γιατί η αρχαία πα­ράδοση διατυπώνει ξεκαθαρισμένες αντιλήψεις πάνω στο ζήτημα αυτό. Ο Γραμματικός Δίδυμος ο Χαλκέ­ντερος διακρίνει δύο πρόσωπα με το όνομα Α ντιφών που έζησαν ταυτόχρονα στην Αθήνα τον 5ο π.Χ. αι­ώνα. Ο πρώτος ήταν ο Α ντιφών ο ρήτορας, ο οποίος ονομάζεται και Αντιφών της Ραμνούντας, ο συγγρα­φέας του έργου «Τετραλογίε9) που ήταν ειδικές πε­ριλήψεις ομιλιών για κατηγορίες ή υπερασπίσεις συ­γκεκριμένων πραγματικών ή φανταστικών δο'λι:Jφό­νων. Πήρε μέρος στην πολιτική στην Αθήνα και κα­ταδικάστηκε σε θάνατο το 411 π.Χ. όταν ανατράπη­κε η ολιγαρχική συνωμοσία στην Αθήνα (Θουκυδί­δης VIII,68). Ο Α. Lesky λέγει χαρακτηριστικά: «Υ-

του Γ.Η. Μπαραλή

ποφερτά καλά είμαστε πληροφορημένοι για τον ρή­τορα Αντιφώντα τον Ραμνούσιο, που για τις ικανότη­τές του ο Θουκυδίδης (VIII,68) δίνει μια θαυμάσια μαρτυρία: ένα από τα καλύτερα μυαλά της εποχής του, κρατιόταν γενικά στο πίσω πλάνο, μπορούσε ό­μως να προσφέρει αποτελεσματική βοήθεια σ' αu­τούς που αγωνίζονταν μπροστά και στο δικαστήριο και στο λαό. Ήταν η ψυχή του ολιγαρχικού πραξικο­πήματος του 411, και όταν κατάρρευσε η εξουσία των τετρακοσίων, καταδικάστηκε σε θάνατΟ)). Για τον Πλάτωνα ο Ραμνούσιος (Μεν. 236α) ήταν ένας ρητοροδιδάσκα'λιJς περιωπής, συνέπεια του οποίου είναι να του αποδοθούν όσα ρητορικά διδακτικά συγγράμματα είναι γνωστά με το όνομα Αντιφώντας ([11], σελ. 505). Ο άλ'λιJς ήταν ο οιωνοσκόπος και ο ερμηνευτής των ονείρων, όπως τον περιη�άφει. Ο Ερμογένης (Περί Ιδεών, 11, 1 1 , 7-9) διαχώpισε τους δύο συγγραφείς στηριζόμενος στο ιδιαίτερο ύφος τους που υπάρχει ανάμεσα στις «Τετραλογίε9) και τις άλλες εργασίες. Ο Caecilius of Ca1acte, που έ­γραψε λίγο αργότερα από το Δίδυμο, όπως παρου­σιάζεται στο «Liνes of the ten oratorS)) στο P suνdo -P1utarch θεωρεί ότι υπήρχε ένας Αντιφών. Οι σύγ­χρονοι λόγιοι θεωρούν ότι υπήρχαν τρεις. σοφιστές με το όνομα Αντιφών. Αλ'λι:Jι επίσης θεωρούν ότι ο Α ντιφών ο ρήτορας και ο σοφιστής ήταν το ίδιο και το αυτό πρόσωπο. Σύγχρονοι σχολιαστές στηριζόμε­νοι στις απόψεις του Ερμογένη αναφέρουν ότι ο πο­λιτικός και ρήτορας Αντιφών ήταν φανατικός ολι­γαρχικός και για τις αντιδημοκρατικές του ιδέες κα­ταδικάστηκε σε θάνατο, ενώ ο σοφιστής Αντιφών παρουσιάζεται δημοκρατικός. Α πό τα πολλά συγγράμματα του σοφιστή σώ-

. .. θηκ� ένας αριθμός απο?πασμάτων

, που εμ­

········ �- φανιστηκε με την ανακαλυψη παπυρων. Τα αποσπάσματα αυτά μας πληροφορούν για τις πολι­τειακές, κοινωνιολογικές και εmστημονικές του α­ντιλήψεις. Από το έργο του Αντιφώντα που διασώ­θηκε συμπεραίνεται ότι έyραψε τέσσερα συγγράμμα­τα: «Περί Ομονοίας)), «Περί Αληθεία9) (2 βιβλία), «Πολιτικόφ και «Περί κρίσεως ονείρων)). Μερικές πηγές αναφέρουν ότι υπήρχε και ένα πέμπτο έργο με

ΕΥΚΛΕΙΔΗΣ Β' λ.ε. τ.Ι/2

Page 5: Ευκλειδης Β 41

Αρχαία Ελληνικά Μαθηματικά

τίτλο «Η τέχνη να αποφεύγει τον πόνο>> . Ασχολήθη- θείας» βρέθηκε στον Οξύρρυγχο της Αι-yι)πτου. Το κε μεταξύ των ά)J..JJ)ν με προβλήματα της αστρονο- έΡΎΟ αυτό του σοφιστή γράφτηκε για να υποστηρίξει μίας και της γεωμετρίας, καθώς και με την κοσμολο- τις απόψεις του Ελεάτη Παρμενίδη (5 1 5-440 π.Χ.), γία και την ψυχολογία. αρχηγού της Ελεατικής Σχολής, ο οποίος πίστευε

Τα αποσπάσματα που έχουν διασωθεί όσον α- στην ολότητα, ότι δηλαδή κάθε μέγεθος είναι μια μο­φορά τα έΡΎα του «Πολιτικός» και «Περί κρίσεως ναδική οντότητα και δεν μπορεί να διαιρεθεί σε ωJ.n. ονείρων» είναι λίγα και δεν μας παρέχουν επαρκείς μικρότερα μεγέθη. Υπερασπίστηκε τις ίδιες φιλοσο­πληροφορίες. Ο Α. Lesky αναφέρει: «Για το σύy- φικές ιδέες που ο Ζήνων ο Ελεάτης (495-435 π.Χ.) γραμμα Κατά Αλκιβιάδου, για τον Πολιτικό και την υποστήριξε με τα παράδοξά του. Ο Γαληνός διασώ­Ονείρων κρίση δεν θα ήταν φρόνιμο να διατυπώσου- ζει την αρχή του συγγράμματος αναφέροντας: με υποθέσεις πατρότητας, εξαιτίας της κατάστασης «οοπερ καί ό Α. έν τct> προτέρ<,u τής Αληθείας οϋτω του προβλήματος.»([11], σελ. 508). λέγων· ταύτα δέ γνούς είσι;ι εν τι οuδέν όν αuτct>

Στο έΡΎΟ του «Περί Ομονοίας» ο Αντιφών οuτε ών όψει όρq. (ό όρών) μακρότατα οuτε ών υποδεικνύει τους τρόπους με τους οποίους θα γνώμι;ι γιγνούσει ό μακρότατα γιγνώσκων» ([8], μπορέσει ο άνθρωπος να κατανοήσει τη ζωή του. σελ. 337 _ 338). Θεωρεί ότι η ομόνοια, δηλαδή η ταυτότητα των ο Αντιφών είχε την αντίληψη ότι η φύση συμ­ιδεών και αντιλήψεων μεταξύ των πολιτών, αποτελεί φωνεί περισσότερο με την αλήθεια παρότι 0 νόμος. το θεμελιώδες και πρωταρχικό στοιχείο της Με τον όρο νόμο εννοεί τις ανθρώmνες δοξασίες και κοινωνικής συμβίωσης. « ... αλλά μήν καί όμόνοιά γενικότερα τον ανθρώmνο πολιτισμό. Επομένως, η γε μέγιστόν τε άγαθόν δοκεϊ ταϊς πόλεσιν είναι αντίθεση μεταξύ νόμου και φύσης ισοδυναμεί με την καί πλειστάκις έν αUταϊς αϊ τε γερουσίαι καί οί αντίθεση που υφίσταται στον κόσμο μεταξύ πολιτι­ιiριστοι ιiνδρες παρακελεύονται τοϊς πολίταις σμού και φυσικής κατάστασης. Έτσι ο Αντιφών υ­όμονοεϊν, καί πανταχού έν τ(ι 'Fλλάδι νόμος κείται ποστήριζε πως οι άνθρωποι είναι από τη φύση τους τούς πολίτας όμνύναι όμονοήσειν, καί πανταχού ίσοι και γι' αυτό δεν πρέπει να υπάρχουν κοινωνικές όμνύουσι τόν δρκον τούτον ... ». ([8], σελ. 356) και εθνικές διακρίσεις, ούτε αριστοκράτες και

Ο Α ντιφών ήταν υποστηρικτής της ομόνοιας και άσημοι, ούτε ελεύθεροι και δούλοι, ούτε Έλληνες και θεωρούσε πως η αναρχία είναι το μεγαλύτερο κακό Βάρβαροι « ... τούς έκ καλών πατέρων έπαιδούμε­μέσα στην πολιτεία. « ... άναρχίας δ' οuδέν κάκιον θά τε καί σεβόμεθα, τούς δέ έκ μή καλού οίκου άνθρώποις· ταύτα γιγνώσκοντες οί πρόσθεν ιiν- όντας οuτε έπαιδούμεθα οuτε σεβόμεθα. εν τούτ4J θρωποι άπό τής άρΧJϊς εiθιζον τούς παϊδας ιiρχε- δέ πρός άλλήλους βεβαρβαρώμεθα, έπεί φύσει σθαι καί τό κελευόμενον ποιεϊν, iνα μή έξανδρού- · πάντα πάντες όμοίως πεφύκαμεν καί βάρβαροι μενοι είς μεγάλην μεταβολήν iόντες έκπλήσσοι- καί 'Έλληνες είναι» ([8], σελ. 352- 353). Σύμφωνα με ντο» ([8], σελ. 365). Υποστήριξε ότι ο άνθρωπος πρέ- το σοφιστή η αλήθεια εmκρατεί στον κόσμο της φύ­πει να συγκρατεί το θυμό του και να περιορίζει τα σης, ενώ αντίθετα ο ανθρώmνος κόσμος χαρακτηρί­πάθη του, γιατί διαφορετικά δεν μπορεί να υπάρχει ζεται από την απάτη και το ψεύδος. Ο ανθρώmνος κοινωνική συμβίωση. Η σωφροσύνη σύμφωνα με νόμος μπορεί να σταθεί επιζήμιος στη φυσική ελευ­τον Α ντιφώντα είναι το μεγαλύτερο προτέρημα στον θερία των ανθρώπων δημιουΡΎώντας έτσι κοινωνικές άνθρωπο, γιατί μόνο αυτή τον βοηθά να κρίνει και εθνικές ανισότητες. σωστά και να ξεπερνά το θυμό του. «σωφροσύνην ο Αντιφών είχε μια πρόωρη και σημαντική δέ άνδρός οuκ aν λλλου όρθότερόν τις κρίνειεν, η συμβολή στα μαθηματικά και ειδικότερα

δσι:ις τού θυμού ταϊς παραχρημα ήδοναϊς εμ- στη Γεωμετρία, όταν έκανε μια προσπάθεια

φράσσει αuτός έαυτόν κρατεϊν τε καί νικάν i]δυνή- για την επίλυση του προβλήματος του τετραγωνι­

θη αuτός έαυτόν» ([8], σελ. 364). σμού του κύκλου. Με αυτό τον τρόπο είναι ο πρώτος

Πολύ σημαντική για τον άνθρωπο θεωρεί την που σκέφτηκε ότι, αν γράψει τα εγγεγραμμένα σε ένα

παιδεία, που σύμφωνα με το γενικό πρόγραμμα των κύκλο κανονικά πολύγωνα 4, 8, 16, 32, ... πλευρών ή

σοφιστών, αποτελεί το mo αξώλογο από τα τα κανονικά 3, 6, 12, 24, ... πλευρών, διαδοχικά, · μπορεί όταν οι πλευρές των πολυγώνων συμπέσουν ανθρώmνα πράγματα. «... πρώτον, οίμαι, τών εν με τον κύκλο, λόγω του μικρού μεγέθους τους, να τε-

άνθρώποις έστί παίδευσις· όταν γάρ τίς τραγωνίσει τον κύκλο, και αυτό γιατί τα κανονικά πράγματος κιiν ότουούν τήν άρχήν όρθώς πολύγωνα τετραγωνίζονται. Η μέθοδος αυτή του Α­ποιήσηται, εiκός καί τήν τελευτήν όρθώς ντιφώντα υποδεικνύεται από το Σιμπλίκιο και Θεμί­γίγνεσθαι» ([8], σελ. 365). στιο αντίστοιχα. Χαρακτηριστικά οι μέθοδοι αυτές

Ένας πάπυρος στον οποίο υπάρχει ένα απόσπα- περιγράφονται με τους ακόλουθους δύο τρόπους: ό σμα από το σύγγραμμα του Αντιφώντα «Περί Αλη- δε Α. γράψας κύκλον ένέγραψέ τι χωρίον είς αuτόν

ΕΥΚΛΕΙΔΗΣ Β' λ.ε. τ.l/3

Page 6: Ευκλειδης Β 41

Αρχαία ΕU.ηνικά Μαθηματικά

δΕ: Α.. γράψας κύκλον ένέγραψέ τι χωρίον εiς αότόν πολύγωνον τών έγγράφεσθαι δυναμένων. έστω δΕ: εi τύχοι τετράγωνον το έγγεγραμμένον. έπειτα έκάστην τών τού τετραγώνου πλευρών δίχα τέ­μνων iιπο τής τομής -έπι τας περιφερείας πρΟς όρθας ήγε γραμμάς, αί δηλονότι δίχα έτεμνον έκάστη το καθ' αύτi]ν τμήμα τού κύκλου. έπειτα iιπο τής τομής έπεζεύγνυεν έπι τα πέρατα τών γραμμών τού τετραγώνου εuθείας, ώς γίνεσθαι τέτταρα τρίγωνα τα iιπο τών εuθειών, το δΕ: δλον σχήμα το έγγεγραμμένον οκτάγωνον. και οuτως πάλιν κατα την αuτi]ν μέθοδον, έκάστην τών τού οκταγώνου πλευρών δίχα τέμνων iιπο τής τομής έπι τι)ν περιφέρειαν πρΟς όρθας άγων και έπιζευγνύς iιπο τών σημείων, καθ' α αί προς ορθας iιχθείσαι έφήπτοντο τών περιφερειών, εuθείας έπι τα πέρατα τών διt;]ρημένων εuθειών, έκκαιδεκάγωνον έποίει το έγγραφόμενον. και κατα τον αuτον πάλιν λόγον τέμνων τας πλευρας τού έκκαιδεκαγώνου τού έγγεγραμμένου και έπιζευγνύς εuθείας και διπι\ασιάζων το έγ­γραφόμενον πολύγωνον και τούτο αει ποιών c:;Jετό ποτε δαπανωμένου τού έπιπέδου έγγραφήσεσθαί τι πολύγωνον τούτφ τιf> τρόπφ έν τιf> κύκλφ, ού αί πλευραι διiι σμικρότητα έφαρμόσουσι τή τού κύ­κλου περιφερείι;t. παντι δΕ: πολυγώνφ Lσον τετρά­γωνον δυνάμενοι θέσθαι, ώς έν τοϊς Στοιχείοις [Eucl. Π 14] παρει\άβομεν, διiι το iσον ύποκείσθαι το πολύγωνον τιf> κύκλφ έφαρμόζον αuτιf>, έσόμεθα και κύκλφ Lσον τιθέντες τετράγωνον. ΊΉΕΜΙSΤ. Phys. 4, 2 πρΟς ΑντιΦώντα δΕ: οuκέτ' άν έχοι λέγειν ό γεωμέτρης, δς έγγράφων τρίγωνον i.σόπλευρον εiς τον κύκλον και έφ' έκάστης τών πλευρών έτερον i.σοσκει\Ες συνιστας πρΟς τή περιφερείι;t τού κύκλου και τούτο έφεξi]ς ποιών c:;Jετό ποτε έφαρμόσειν τού τελευταίου τριγώνου τι)ν πλευραν εuθείαν ούσαν τή περιφερείι;t. τούτο δΕ: ήν <τού> τι)ν έπ' άπειρον τομην iιναιρούντος, ην ύπόθεσιν ό γεωμέτρης λαμβάνει". ([8], σελ. 340-341).

Σύμφωνα με το Θεμίστιο, όπως είδαμε παραπά­νω, ο Α ντιφών ξεκίνησε με ένα ισόπλευρο τρίγωνο. Αυτή η εκδοχή φαίνεται να είναι η αυθεντική. Ο Σι-

μπλίκιος υποστηρίζει ότι ο Α ντιφών ενέγραψε ένα από τα κανονικά πολύγωνα που μπορούν να εγγρα­φούν σε κύκλο, και έστω ότι αυτό είναι το τετράγω­νο. Στη συνέχεια με βάση την κάθε πλευρά του εγγε­γραμμένου τριγώνου ή τετραγώνου ορίζει ισοσκελές τρίγωνο με την κορυφή του στο τόξο του μικρότερου τμήματος του κύκλου, στο οποίο βαίνει η πλευρά. Με τον τρόπο αυτό, αν ενώσει τις κορυφές των ισο­σκελών τριγώνων, κάθε φορά θα προκύπτει κανονικό εγγεγραμμένο πολύγωνο με δtπλάσιο αριθμό πλευ­ρών από το αρχικό. Ο τρόπος αυτός του διπλασια­σμού του αριθμού των πλευρών του εγγεγραμμένου στον κύκλο πολυγώνου από τον Αντιφώντα, αν και ήταν ευφυής στη σύλληψή του, εντούτοις όμως δεν οδήγησε σε σωστά συμπεράσματα.

Η πρακτική αξία της μεθόδου του Α ντιφώντα φαίνεται από την πραγματεία «Κύκλου Μέτρηση» του Αρχιμήδη, όπου κατασκευάζοντας εγγεγραμμένα και περιγεγραμμένα κανονικά πολύγωνα με 96 πλευ­ρές, έφθασε με θαυμαστό τρόπο στον υπολσyισμό

' 310 31 της τιμης του π, 71 <

π < Ί .

Ο Αριστοτέλης σχολιάζοντας επικριτικά τις α­πόψεις του Α ντιφώντα για τη μέθοδό του σχετικά με τον τετραγωνισμό του κύκλου (Φυσικά 1.2, 1 85a 14-17) αναφέρει ότι δεν είναι καν ανάγκη να αντικρού­σει τον τετραγωνισμό του κύκλου του Αντιφώντα, γιατί δεν βασίζεται σε αναγνωρισμένες αρχές της γε­ωμετρίας. «aμα δ' οuδε λύειν &παντα προσήκει, iιι\ι\' η δσα έκ τών aρχών τις έπιδεικνύς ψεύδεται, δσα δε μη, ou· οίον τον τετραγωνισμον τον μεν διiι τών τμημάτων γεωμετρικού διαι\ύσαι, τον δ' ΑντιΦώντος ou γεωμετρικού.» ([8], σελ. 340).

Ο Τ. Heath αναφερόμενος στο έργο του Αντι­φώντα και σχολιάζοντας τις απόψεις του Σιμπλίκιου παρατηρεί ότι: «0 Σιμπλίκιος μας λέει ότι, ενώ σύμ­φωνα με τον Αλέξανδρο η γεωμετρική αρχή που πα­ραβιάζεται με αυτόν τον τρόπο είναι η αλήθεια σύμ­φωνα με την οποία ένας κύκλος εφάπτεται σε μια ευ­θεία (μόνο) σε ένα σημείο, ο Εύδημος είπε πιο σω­στά ότι επρόκειτο για την αρχή σύμφωνα με την ο­ποία τα μεγέθη μπορούν να διαμερισθούν επ' άπει­ρον. Γιατί, αν η επιφάνεια του κύκλου είναι απερώ­ριστα διαμερίσιμη, η διαδικασία που περιγράφεται από τον Α ντιφώντα δε θα οδηγήσει ποτέ στην εξά­ντληση ολόκληρης της' επιφάνειας του κύκλου, ούτε στο να καταστήσει τις πλευρές του πολυγώνου ικα­νές να πάρουν τη θέση της επιφάνειας του κύκλου. Όμως η αντίρρηση στη δήλωση του Αντιφώντα στην πραγματικότητα δεν είναι τίποτα περισσότερο από λεκτική» ([9], σελ. 277- 278).

Αναφερόμενος στη συνέχεια στην απόδειξη της πρότασης :ΧΠ 2 του Ευκλείδη «Δύο κύκλοι είναι α­νάλογοι των τετραγώνων των διαμέτρων του9>, γρά-

ΕΥΚΛΕΙΔΗΣ Β' λ.ε. τ.l/4

Page 7: Ευκλειδης Β 41

Αρχαία Ελληνικά Μαθηματικά

φει ότι ο Ευκλείδης χρηmμοποιεί την ίδια ακριβώς κατασκευή, μόνο που εκφράζει το συμπέρασμα με διαφορεnκό τρόπο ([2], σελ. 93 - 102).

Ο Ιωάννης Φιλόπονος σχολιάζοντας τη μέθοδο τετραγωνισμού του κύκλου από τον Βρύσωνα και τον Α ντιφώντα αναφέρει τα ακόλουθα.

«έλεγε δέ ό φιλόσαj>ος Πρόκλον τόν αύτοϋ δι­δάσκαλον έπισκήπτειν η; Αλεξάνδρου έξηγήσει, δτι, εl οϋτως έτετραγώνισεν ό Βρύσων τόν κύκλον, σννέτρεχε τ<tJ Αντι<j>ώντος τετραγωνισμψ το γιΧρ μεταξu τοϋ έγγραφομένοu καi. περιγραφομένου εuθuγράμμου γραΦόμενον σχήμα έφαρμόζειν η; τοϋ κύκλου περιφερείι;ι· τοϋτο καΙ. ό Αντι<j>ών έποίει, έως ού έφήρμοσεν, ώς έκείνος, εuθείαν πε­ριφερεία, όπερ aδύνατον» ([7], σfλ. 112).

Γεyονός πάντως εiναι ότι παρόλα τα αρνητικά σχόλια που έχ,ουν διατυπωθεί για τη μέθοδο του Α­ντιφώντα όσον αφορά τον τετραγωνισμό του κύκλου, μας βρίσκει σύμφωνους η άποψη του Heath σύμφω­να με την οποία: «0 Α ντιφών αξίζει μια τιμητική θέση στην ιστορία της Γεωμετρίας ως ο άνθρωπος που γέννησε την ιδέα της εξάντλησης μιας επιφά­νειας μέσω εyyεyραμμένων κανονικών ·πολυγώνων με αυξανόμενο αριθμό πλευρών, μια ιδέα στην οποία ο Εύδοξος θεμελίωσε τη μέθοδο της εξάντλησης, η οποία άφησε εποχή».

.....ΟΥΣ

ΒΙΒΛΙΟΙ'ΡΑΦΙΑ 1. θ.Γ. Εξαρχάκος, «Εισαγοηή στα Μαθηματικά», τόμος Β",

Ανάλυση, Αeήνα 1993. :Ζ. Κ.Ε.ΕΠ.ΕΚ, «Ευκλείδη "Στοιχεία"», τόμος ΙΠ, «Η Γεωμε­

τρία του χώρου», Βιβλία ΧΙ, ΧΠ, XIII, Κ.Ε.ΕΠ.ΕΚ., Αθήνα 2001.

3. Μ. Μπρίκα, «Τα περίφημα άλυτα γεωμετρικά προβλήματα της αρχαιότητας», Αeήνα 1970.

4. Β. Σπανδάγου- Ρ. Σπανδάyου -Δ. Τραυλού, «Οι Μαθημα­τικοί της Αρχαίας Ελλάδας», εκδ. Αίθρα, Αθήνα 1994.

5. τΣΕΛΛΕΡ-ΝΕΣΤΛΕ, «Iστορία της Ελληνικής Φιλοσαρί­ας», Μετ. Χ. Θεοδωρίδη, εκδ. Εστίας, Αeήνα

6. Δ. Τσψπουράκη, «Η Γεωμετρία και οι εργάτες της στην Αρχαία Ελλάδα», Αeήνα 1983.

7. Ιωάννης Φ1λόπονος, «ln Aήstotelis analytica posteήora com­mentaήa», Maximilianus Wallies, Berolini, vol 13,3 page 112.

8. Η. Diels- W. Κranz, «Die Fragmente der Vorsokratiker», Weidmann.

9. T.L. Heath, «Iστορία των Ελληνικών Μαθηματικών», τόμος I, Κ.Ε.ΕΠ.ΕΚ., Αeήνα 2001.

10. G. Β. Keιferd, «Dictionary of Scientific Biography ,. vol. Ι, Charles Scήbner' s sons NEW ΥΟRΚ. ( 1971 ).

11. Α. Lesky, Geschichte der Gήechishen Literatur, Μετφ. Α. Τσοπανάκη, Θεσσαλονίκη 1972.

l:Z. G. Loria, «Iστορία των Μαθηματικών», Μετάφραση Μ. Κωβαίου, τόμος πρώτος, Ε.Μ.Ε., Αθήνα 1971.

13. Suidae Lexicon, Ed. Α. ADLER, νο\1-4, Leipzig 1928-35. 14. B.L. νad der Waerden, «Η αφύπνιση της Επιστήμης», Απi>­

δοση στα Ελληνικά - Επιστημονική επιμέλεια: Γ. Χριστια­νίδης, Π.Ε.Κ., Ηράκλειο 2000.

.Ε.

Η Ε.Μ.Ε. διοργανώνει τον 62ο Πανελλ1Ίνιο Μαθητικό Διαγωνισμό (Π.Μ.Δ.) στα Μαθη­ματικά «0 ΘΑΛΗΣ» το Σάββατο 3 Νοεμβρίου 2001 και ώρα 9.00 π. μ.

Στο διαγωνισμό αυτό μπορούν να λάβουν μέρος μαθητές όλων των τάξεων των Λυκείων και των Β' και Γ' τάξεων των Γυμνασίων της χώρας, οι οποίοι και θα πρέπει να δηλώσουν συμμετοχή μέχρι και την 26η Οκτωβρίου 2001 για να μπορέσει η Επιτροπή Διαγωνισμών της Ε.Μ.Ε. ν' αποστείλει εγκαίρως ικανό αριθμό αντιγράφων των θεμάτων στα εξεταστικά κέ­ντρα.

Ο Διαγωνισμός θα διαρκέσει 3 ώρες, και θα απαρτίζεται μόνο από θέματα πλήρους ανά­πτυξης, γι' αυτό θα απαιτηθούν κόλλες αναφοράς.

Οι απαντήσεις των μαθητών κάθε εξεταστικού κέντρου θα πρέπει ν' αποσταλούν στην Ε­πιτροπή Διαγωνισμών της Ε.Μ.Ε για βαθμολόγηση και τα' αποτελέσματα θα αποσταλούν στις τοπικές Νομαρχιακές Επιτροπές για βράβευση των μαθητών που θα διακριθούν.

Οι μαθητές, που θα διακριθούν στο Διαγωνισμό<(() ΘΑΛΗΣ», θα κληθούν να διαγωνι-σθούν στον επόμενο Διαγωνισμό «0 ΕΙΔΗΣ», που θα διενεργηθεί στις 2 r 200Ζ και αυτοί που θα διακριθούν στον κ<ltΥΚ..ι\.ΕΙΔΗ» θα διαγωνισθούν στον «ΛΡΧΙΜΗΔ » στις _6 Φεβροοορ'ου 206 , για να επιλεγεί η Εθνική ομάδα που θα λάβει μέρος στη 19η Βαλ­κανική Μαθηματική Ολυμπιάδα (Κωνσταντινούπολη, Μαϊος 2002), στη XLIII Διεθνή Μαθη­ματική Ολυμπιάδα (Μεγάλη Βρετανία, Ιούλίος 2002) και στην 6η Ολυμπιάδα Μαθηματικών για μαθητές του Γυμνασίου (Ρουμανία, fούνιος 2002).

ΕΥΚΛΕΙΔΗΣ Β' λ.ε. τ.l/5

Page 8: Ευκλειδης Β 41

ο I I

Ο ρόλος του σχολείου στην κοινωνία της πληροφορίας και των νέων τεχνολογιών

Η. Ελληνική Μαθηματική Εταιρεία, όπως κάθε χρόνο, έτσι και φέτος, οργανώνει το

.-c .. � ετήσιο μαθηματικό της συνέδριο, με το παράρτημα της ΕΜΕ Ρόδου, έχοντας θέμα τον τίτ­λο του άρθρου.

Στην πανελλήνια αυτή εκδήλωση παίρνουν μέ­ρος διάφοροι σύνεδροι, που με τις επιστημονικές τους εισηγήσεις εκθέτουν απόψεις για το θέμα. Α­κούγονται προτάσεις, διερευνήσεις, αντιρρήσεις και γενικότερα διατυπώνονται πορίσματα με στόχο <<να αποκτήσουν τα σχολικά μαθηματικά ουσιαστικό νόημα και η μάθησή τους να έχει διάρκεια ... στην πρόληψη του μαθηματικού αναλφαβητισμού».

Η Unesco έδωσε τον εξής ορισμό: «Λειτουρ­γικά αναλφάβητο είναι ένα πρόσωπο ανίκανο να ασκήσει όλες τις δραστηριότητες για τις οποίες η μάθηση είναι αναγκαία για την καλή λειτουργία της ομάδας του και της κοινότητάς του . . . )). Δηλαδή ορισμένοι απόφοιτοι μέσης εκπαίδευσης, όταν βγουν στη ζωή δεν μπορούν να κάνουν καμιά ε­φαρμογή από τις γνώσεις που διδάχτηκαν.

Παραδε(yμcnα: Όταν δεν μπορεί να γράψει μια απλή αίτηση, τι ελληνικά έμαθε; Όταν δεν ξε­χωρίζει το στέγνωμα από την εξάτμιση πώς έχει στη Φυσική βαθμό 15; Όταν αδυνατούν να βρουν 2

04 ' δ ' ' θύ το S + , η να σχε ιασουν το μεσον ευ γραμ-

μου τμήματος και το χειρότερο όταν λένε, τι μας βασάνισαν στο σχολείο με τα μαθηματικά, αφού πουθενά δεν τα χρησιμοποιούμε;

Για όλα αυτά που συζητούνται στα συνέδρια και γίνεται προσπάθεια να εντοmσθεί αυτό που φταίει και να αλλάξει, δεν ενδιαφέρει μόνο τους σχολικούς φορείς, αλλά ολόκληρη την κοινωνία. Για το μαθη­ματικό συνέδρω, περισσότερο απ' όλους ενδιαφέρει τους μαθηματικούς δασκάλους και τους μαθητές τους.

του Γ. Ωραιόπουλου Γι' αυτό μπαίνει γενικό άρθρο στο περιοδικό

μας, να το διαβάσουν οι αναγνώστες του, να συζητή­σουν μεταξύ τους, με τους καθηγητές τους και με τους γονείς τους το σοβαρό αυτό θέμα. Α ν θέλουν μπορούν να γράψουν τις απόψεις τους, που θα δημο­σιευτούν.

Υπεύθυνοι για την κατάσταση της μαθηματικής εκπαίδευσης θεωρούνται: πρώτα το εκπαιδευτικό σύστημα, κατόm το αναλυτικό πρόγραμμα σπουδών, έπειτα τα σχολικά μαθηματικά βιβλία. Αλλά σημα­ντική είναι η ευθύνη που έχουν οι διδάσκοντες μα­θηματικά και μικρότερη οι διδασκόμενοι.

ΕΥΚΛΕΙΔΗΣ Β' λ.ε. τ.l/6

Page 9: Ευκλειδης Β 41

Το Βήμα του Ευκλείδη

Πρέπει αγαπητοί μαθητές να καταλάβετε τους στόχους που βάζει το Υπουργείο Παιδείας για την αξιολόγησή σας. Οι εξετάσεις, που δυστυχώς είναι πολλές, να θεωρηθούν συμπλήρωμα της εκπαιδευ­τικής διαδικασίας για να εξακριβωθεί: αν ο μαθη­τής αφομοίωσε τις βασικές γνώσεις, αν μπορεί να διατυπώσει κριτική γνώμη διακρίνοντας τα ουσιώ­δη από τα επουσιώδη, αν έχει την ικανότητα να ε­πεξεργαστεί και να εφαρμόσει ό,τι χρήσιμο διδά­χτηκε.

Στον εικοστό πρώτο αιώνα είναι aξιοσημείω­τος ο ρόλος και η αξία μιας σειράς από μαθηματι­κές έννοιες που βοηθούν όλες τις επιστήμες, τις νέες τεχνολογίες, τη συνεχή ανανεωμένη πληρο­φορική και γενικότερα τον πολιτισμό.

Οφείλουν λοιπόν όλοι οι υπεύθυνοι της Παι­δείας να βελτιωθούν, τόσο ο καθένας χωριστά, αυ­τοβελτίωση, όσο και συνεργατικά επειδή υπάρχει

α Μαθηματικά ως επιστήμη είναι δη-τ μιούργημα κυρίως του Ελληνικού πνεύ­ματος. Στηρίζο�ται πρ�τιστα στο λόyο, στη συγκροτημενη σκεψη και στην εκ­φρασή της μέσα από την ανταλλαγή από­

ψεων και την αναζήτηση της αλήθειας με τη βοή­θεια του διάλογου και της πράξης. Με την έννοια αυτή τα Μαθηματικά είναι αναπόσπαστο μέρος της Φιλοσοφίας και θα ήταν χρήσιμο να διδάσκο­νται και με τον τρόπο αυτό στη Δευτεροβάθμια και Τριτοβάθμια Εκπαίδευση. Οι ρίζες, η γένεση και η εξέλιξη της Φιλοσοφίας από την Αρχαιότητα ως τις μέρες μας προϋποθέτει καλή γνώση των μαθη­ματικών.

Η εκπαίδευση, συχνά αποσπασματική και υ­ποκειμενική εδώ και πολλές δεκαετίες, αδυνατεί να γαλουχήσει τους μαθητές με τον φιλοσοφικό στοχασμό που ευδοκίμησε στον ελλαδικό χώρο, ο οποίος θα επέτρεπε τη συμπόρευση των μαθηματι­κών με άλλους γνωστικούς τομείς.

Στο βιβλίο του «Τα παιδιά του Ευκλείδη», ο Ηλίας Κωνσταντόπουλος υπερασπίζεται τη θέση και την παιδαγωγική αποστολή του μαθηματικού στο σύγχρονο εκπαιδευτικό σύστημα. Είναι ένα βιβλίο ύμνος στη μαθηματική επιστήμη που όχι μόνο εκθειάζει αλλά αναδεικνύει και την παιδαγω­γική αξία των Μαθηματικών στα σχολεία μας; «Παιδιά του Ευκλείδψ είναι όλοι οι Έλληνες που διδάχτηκαν Γεωμετρία ως τις μέρες μας. Όλοι

διαλεκτική αλληλεξάρτηση μεταξύ τους. Αυτό πρέπει να είναι δίκαιο αίτημα του μαθητικού και σπουδαστικού πληθυσμού.

Έτσι και σεις, φίλοι μαθητές και μαθήτριες έ­χετε χρέος, αλλά και συμφέρον συνεχώς να βελ­τιώνετε την απόδοσή σας σ' όλα τα μαθήματα, όχι μόνο στα μαθηματικά, με την ενεργητική συμμε­τοχή σας στη σχολική εργασία σε συνεργασία με τους εκπαιδευτές σας. Οι σημερινές απαιτήσεις της ζωής και η δυναμική της επιστημονικής και τεχνο­λογικής προόδου, επιτάσσουν την ανάπτυξη του γενικού επιπέδου της μαθητικής νεολαίας.

Και επειδή μιλάμε για Μαθηματικά σας κα­λούμε να συνειδητοποιήσετε τη διαλεκτική σχέση της μαθηματικής σκέψης και πνευματικής ανάπτυ­ξης. Γι' αυτό χρειάζεται από σας πολλή δουλειά και αγάπη στην Επιστήμη μας. Είναι για το καλό σας.

της Ευαγγελίας Δαμουλή, Φιλόλογου

γνωρίζουν το Πυθαγόρειο Θεώρημα και προβλη­ματίστηκαν με διάφορα προβλήματα πάνω στην ε­φαρμογή του. Πολύ λίγοι όμως γνωρίζουν τη θεω­ρία του Πυθαγόρα για την αρμονία. Ποιος ήταν πραγματικά ο Πυθαγόρας, ο Ευκλείδης, ο Αρχιμή­δης και ποια ήταν η σημασία της εξέλιξης της σκέψης τους για την επιστήμη των Μαθηματικών αλλά και γενικότερα των Θετικών, και όχι μόνο, ε­πιστημών; Ο συγγραφέας προσπαθεί να μυήσει τους μαθητές σε μια διαφορετική προσέγγιση της Μαθηματικής Επιστήμης.

Στο βιβλίο του εκτός από το διαλεκτικό μέρος, συναντάμε τις απαραίτητες ιστορίες και σχόλια γύρω από την προσωπικότητα των μεγάλων Ελλή­νων Φιλοσόφων Μαθηματικών. Γίνεται προσπά­θεια ευαισθητοποίησης του μαθητή σε θέματα τέ­χνης εμπνευσμένα από τους Μαθηματικούς και τη φιλοσοφία τους. Επιστρατεύονται όλα τα απαραί­τητα για την επιτυχημένη διδακτική πράξη εποπτι­κά μέσα (διαφάνειες, εύστοχα παραδείγματα). Α­νατρέχει στα συγγράμματα των λαμπρών μαθημα­τικών και απομονώνει aποφθεγματικές προτάσεις. Ετυμολογεί όρους γεωμετρικούς, προσπαθώντας έτσι να καταρτίσει γλωσσικά τους μαθητές και να ενισχύσει τους εσωτερικούς δεσμούς του μαθήμα­τος με τα φιλολογικά μαθήματα και γενικότερα με τις ευρύτερα εννοούμενες aνθρωπιστικές σπουδές. Ο Κωνσταντόπουλος βλέπει τη διδασκαλία σαν «θεατρική πράξη» όπου ο καθηγητής καλείται να

ΕΥΚΛΕΙΔΗΣ Β' λ.ε. τ.Ι/7

Page 10: Ευκλειδης Β 41

Το Βήμα του Ευκλείδη

πείσει με την ερμηνεία του το κοινό, τους μαθητές για το ρόλο που ανέλαβε. Για το συγγραφέα η σχολική τάξη δεν είναι ένα φροντιστήριο όπου λύ­νονται ασκήσεις χωρίς αρχή και τέλος. Πριν και μετά, τις ασκήσεις υπάρχει ολόκληρη φιλοσοφία, ένα σύστημα, ένας διαφορετικός τρόπος αντίληψης της ζωής, ο Μαθηματικός. Ο δάσκαλος δεν είναι εντολοδόχος, ούτε ο γκρίζος άχρωμος τρομοκρα­τημένος από τη γραφειοκρατία υπάλληλος των έρ­γων· του Κάφκα. Πρέπει να έχει άποψη και να την εκφράζει, να πρωτοτυπεί και έτσι να έχει τη δυνα­τότητα δημιουργίας. Μόνον έτσι θα μπορεί να συ­νεπαίρνει τους μαθητές του, εμπνέοντάς τους για ό,τι ο ίδιος αγάπησε, πίστεψε και σπούδασε. Εκτός από ευχάριστο ανάγνωσμα «Τα παιδιά του Ευκλείδη» αποτέλεσαν και θεατρική παρά­σταση με την οποία συμμετείχε το 4° Λύκειο Αμα­ρουσίου στο 6° Μαθητικό Φεστιβάλ του Δήμου Αμαρουσίου, στις 6 Μαρτίου 2001, στο θέατρο Α­ναβρύτων. Ως θεατρικό έργο αποδίδει σκηνικά τα

13 «Στοιχεία» του μεγάλου μαθηματικού της αρ­χαιότητας, που συνέθεσε σ' αυτά τις προηγούμενες γνώσεις των Πυθαγορείων. Στη θεατρική ομάδα του 4ou Λυκείου Αμαρουσίου συμμετείχαν 13 μα­θήτριες και μαθητές στους εξής ρόλους: Χριστίνα Πυτιχούτη ως Υπατία. Έλενα Σιόρευτα ως Γνώμων. Αδάμ Γούπιος ως Διαβήτης. Γιάννης Αναγνώστης ως Κανονικός. Κωνσταντίνα Κρίγκου ως Εύδοξος. Εύη Μεσαρίτου ως Αλγόριθμος. Ξένια Κριμητζά ως Διαιρέτης. Μπάμπης Γερανιός ως Πρώτος. Γιώργος Μανώλης ως Ασύμμετρος. Όλγα Μπέλλου ως Άπειρος.

Λέτα Κουβελιώτη ως Πλάτων. Σωτήρης Μανώλογλου ως Πυθαγόρας. Τη σκηνοθεσία ανέλαβε ο ίδιος ο συγγραφέας, ο οποίος απέδωσε και το ρόλο του Ευκλείδη. Τα σκηνικά ήταν της Πόπης Γεωργοπούλου και της

Ντανιέλ Μπόλυτα. Οι μάσκες της Δώρας Βήτου. Την μουσική επιμέλεια είχαν ο Γιώργος Κυνηγα­λάκης και η Σωτηρία Κόλλια.

Η έμπνευση πρωτότυπη, το εγχείρημα δύσκο­λο και το αποτέλεσμα εντυπωσιακό. Ήταν ένα μά­θημα Μαθηματικών σε θεατρική μορφή που δεν είχαμε ποτέ παρακολουθήσει. Με το βιβλίο «Τα παιδιά του Ευκλείδη» και την ομώνυμη παράσταση, αποδείχθηκε για μία α­κόμη φορά ότι η θεωρία αφομοιώνεται μόνο μέσα από την πράξη. Η πράξη της παιδείας πρέπει να στρέφεται προς μια σφαιρική θεώρηση του αντι­κειμένου διδασκαλίας. Μ' αυτόν τον τρόπο γίνο­νται επεξεργάσιμα τα πρωταρχικά συστατικά της εκπαίδευσης και φωτίζονται όλες οι πλευρές της ανθρώπινης δυνατότητας για μια φιλοσοφική και διαλογική ταυτόχρονα θεμελίωση της μορφωτικής διαδικασίας. Έτσι αποτρέπεται η συρρίκνωση των θεμελιακών δεδομένων της Ελληνικής Παιδείας και αποφεύγεται η μαζοποίηση του μαθητή. Ο μα­θητής διατηρεί την ταυτότητά του και το ρόλο του, είναι μοναδικός και ανεπανάληπτος, όσο και ο κα­θηγητής του. Σημείωcnι του «Ευκλείδη)) Το βιβλίο «Τα παιδιά του «Ευκλείδη» του συ­ναδέλφου Ηλία Κωνσταντόπουλου διατίθεται σε όλα τα βιβλιοπολεία της Αθήνας από τις εκδόσεις

«Γκρίζελη». Την κεντρική διάθεση έχει το βιβλιο­πωλείο «Σαββάλα».

<<ΜΑΘΗΜΑΤΙΚΟΣ ΑΛΦΛΒΠΤΙΣΜΟΣ: Ο ΡΟΛΟΣ ΤΟΥ ΣΧΟΛΕJΟΥ ΣΤΗΝ KOJNΩNJA ΤΗΣ ΉΛΗΡΟΦΟΡΙΑΣ ΚΑΙ ΤΩΝ ΝΕΩΝ ΤΕΧΝΟΛΟΓΙΩΝJ>

Η διάγνωση και η έγκαφη πρόληψη φαινομένων μαθηματικού αναλφαβητισμού αποτελούν σήμερα διεθνή πρσε­ραιότητα στον σχεδιασμό και την πρακτική της μαθηματικής εκπαίδευσης.

" Έχει παρατηρηθεί ότι όλο και μεγαλύτεροι μαθητικοί πληθυσμοί, μετά την απομάκρυνσή τους από το σχολείο (από την υποχpεωτική εκπ«;lίδευση είiε από τό λύκειο) ξεκόβουν στο σχολείο και από την μαθηματική σκέψη, δεν χρησιμi­ποιούν τα μαθηματικά ως πνευματικό και επαγγελματικό εργαλείο σί>μφωνα με τις δυνατότητες που είχαν επιδιίξει. · ·

Ο ρόλος του σχολείου στην ανάπτυξη του μαθηματικού αλφαβητισμού έγκειται: στον εμπλουτισμό του μαθηματικού περιεχομένου με διεπιστημονικές συνδέσεις και κοινωνικά χρήσιμεςεφαρμοyές, στη διερεύνηση της διδακτικής πρακτικής με μαθησιακές δραστηριότητες που ενεργοποιούν το μαθητή στην εκτέ1ε­ση συγκεκριμένου έργου, ενθαρρύνουν τη συνεργατικότητα και αναπτύσσουν την αυτοεκτίμηση,

στην ιδιαίτερη ενασχόληση με μαθητικούς πληθυσμούς που εyκ:αταλείπουν γρήγορα το σχολείο ή f:χουν δυσκολίες ένταξης στη σχολική ζωή λόγω πολιτισμικών διαφορών ή/και κοινωνικού αποκλεισμού των οικογενειών τους (μzια-

ΕΥΚΛΕΙΔΗΣ Β' λ.ε. τ.l/8

Page 11: Ευκλειδης Β 41

• Ο Ακαδημαϊκός κος Νικόλαος Αρτεμιάδης προτείνει: ι. Να υπολογιστεί το εμβαδόν της επιφανείας

η οποία ευρίσκεται μεταξύ της περιγεγραμμένης και της εγγεγραμμένης περιφέρειας σε κανονικό πολύγωνο με 49 πλευρές, όπου κάθε πλευρά έχει μήκος 1 .

2. Το γενικό πρόβλημα του Απολλωνίου εί­ναι η κατασκευή κύκλου ο οποίος εφάπτεται τριών δοθέντων κύκλων. «Μερικές» περιπτώσέις προκύ­πτουν όταν μερικοί απ' τους δοθέντες κύκλους εί­ναι σημεία ή ευθείες. Να λυθεί το πρόβλημα όταν δίδονται δύο σημεία Κ, Α και ένας κύκλος C, όπου το Κ είναι το κέντρο του C, και Α είναι ένα τυχόν εσωτερικό σημείο του C.

• Ο καθηγής Ν. Βαδιβούλης προτείνει: Άσκηση ιη Τα ευθύγραμμα τμήματα των διχοτόμων δύο

εντός εναλλάξ γωνιών, που περιέχονται μεταξύ δύο ευθειών παραλλήλων είναι ίσα.

Αντίστροφα: Αν τα ευθύγραμμα τμήματα των διχοτόμων δύο εντός εναλλάξ γωνιών, που σχημα­τίζει μία τέμνουσα δύο ευθειών με αυτές, έχουν τα άκρα τους σ' αυτές και είναι ίσα, τότε οι δύο αυτές ευθείες είναι παράλληλες.

Άσκηση 2η Δίνονται τα τρίγωνα ΑΒΓ και Α1Β1Γ1 για τα

οποία ισχύουν: α1 < β1 � γ1 και α > β � γ . Να δεί­ξετε ότι:

βγtβ� + γ�)+ βιγια2 > βιγι (β2 + γ2 ) + βγα� ·

Στο 4° τεύχος της προηγούμενης περιόδου εί­χαν προταθεί απ' τον συνάδελφο Α. Ι ωαννίδη τα παρακάτω:

ι. Να δειχθεί ότι: e + log 4 < 4 (όπου το σύμβολο log 4 εννοεί το δεκαδικό λογάριθμο του 4).

Απάντηση: (απ' την Σ.Ε.) e + log4< 3 + log10 = 4 .

2. Να αποδείξετε ότι: Σε κάθε τρίΎωνο

ο Εuκlι:ίδιις uροτι:ίvι:ι ... ΕακΑι:ιΟιι

και ••• Jhόφavτo Δ

ΑΒΓ ισχύει Α· συν Β+ ημΑσυνΓ > Ο όπου Α, Β, Γ τα μέτρα των Ύωνιών του τριΎώνου ·σε ακτί­νια.

Απάντηση απ' τον Α. Ιωαννίδη (1): Έστω συν Β > Ο .

Αφού Ο<Α<π=>ημΑ > Ο . Επίσης ισχύει1 : «I χ 1>1 ημχ ι. Ύια κάθε χ Ε )R* )).

Άρα: Α > η μΑ . Οπότε ΑσυνΒ > ημΑσυνΒ ΑσυνΒ + ημΑσυνΓ > ημΑσυνΒ + ημΑσυνΓ ΑσυνΒ + ημΑσυνΓ > ημΑ (συνΒ + συν Γ)

Β + Γ Β-Γ ΑσυνΒ + ημΑσυνΓ > ημΑ2συν--συν--=> 2 2

Α Β-Γ ΑσυνΒ + ημΑσυνΓ > 2ημΑημ-συν-- >Ο 2 2

Ο< Β< π Διότι: =>-π< Β - Γ< π=> -π<-Γ<Ο π Β - Γ π Β - Γ --<--<-=>συν-- > 0 . 2 2 2 2

(11): Έστω συν Β< Ο Τότε: Β > � 2

Και συνεπώς: Ο< Α< π . 2 Ισχύει2: Α< εφΑ Οπότε ΑσυνΒ > εφΑσυνΒ

( αφού συνΒ<Ο) Άρα: ΑσυνΒ + ημΑσυνΓ > εφΑσυνΒ + ημΑσυνΓ ή ισοδύναμα ΑσυνΒ + ημΑσυνΓ > ημΑσυνΒ + ημΑσυνΓ ή συν Α

Α Β μΑ Γ μΑ(συνΒ + συνΑσυνΓ ). συν + η συν > η συν Α

Άρα:

Α Β μΑ Γ ημΑ[--συν(Α+Γ)+συνΑσυνΓ] συν +η συν >__::__ _________ _ συν Α

1 Θα γίνει γνωστό αργότερα στην Τρίτη τάξη (Θετική Κατεύθυνση). Εννοείται ότι το χ μετριέται σε ακτίνια 2 Αργότερα (τρίτη τάξη Θετική Κατεύθυνση) θα μά-

θ ' π ου με οτι: « ο < χ < -=> χ < εφχ »

2

ΕΥΚΛΕΙΔΗΣ Β' λ.ε. τ.l/9

Page 12: Ευκλειδης Β 41

Ο Ευκλεiδης προτείνει ... Ευκλεiδη και ••• Δώφοντο

Συνεπώς ΑσυνΒ + ημΑσυνΓ > ημΑ ( -συνΑσυνΓ + ημΑημΓ + συνΑσυνΓ) --�----------��--------�=>

συν Α 2Α μΓ ΑσυνΒ + ημΑσυνΓ > ημ η >Ο

συν Α

(διότι συνΑ>Ο [αφού Ae(o.�}· (111): Αν συνΒ =0 τότε Β= π. Οπότε η προς

2 απόδειξη σχέση είναι προφανής (μια και υποχρεω-

τικά Γ<�). 2

Απάντηση απ' τον Γιάννη Στρατή

α) Αν Β, Γ< π , τότε η συνθήκη είναι προφα-2

νής.

β) Α Β π . . ν > 2 , τοτε η ανισοτητα που μας εν-

διαφέρει είναι ισοδύναμη με την Ασυν(π-Α-Γ)+ημΑσυνΓ>Ο ή -Ασυν(Α+Γ)+ημΑσυνΓ>Ο ή ημΑσυνΓ-Α ( συνΑσυνΓ-ημΑημΓ) >Ο ή ΑημΑημΓ +συν Γ ( ημΑ-ΑσυνΑ) >Ο Η ανισότη­τα αυτή είναι ισοδύναμη με την ΑεφΑεφΓ+εφΑ-Α>Ο (αφού συνΑσυνΓ>Ο), που ισχύει επειδή εφΑεφΓ > Ο και εφΑ > Α .

γ) Αν Γ> π , τότε η ανισότητα που μας εν-2

διαφέρει είναι ισοδύναμη με την ΑσυνΒ+ημΑσυν(π-Α-Β)>Ο ή ΑσυνΒ-ημΑσυν(Α+Β)>Ο ή ΑσυνΒ>ημΑσυν(Α+Β), που ισχύει αφού:

π Γ>-2

Συνεπώς π ->Α+Β>Β 2

Άρα

συνΒ > συν(Α +Β)> Ο. ΑλλiJ. Α> ημΑ >Ο, οπό­τε ΑσυνΒ > ημΑσυν(Α +Β).

δ) Α Β π.Γ π θή . ν =2 η =2 η συν κη ειναι προ-

φανής.

Δ 3. Αν α, p, γ πλευρές τριγώνου ΑΒΓ να απο-δείξετε ότι

αpγ;;:: (α + Ρ- γ)(Ρ + γ - α)(γ + α- β).

λάκο και Γιώp'fΟ Τασσόπουλο β+γ-α =χ> 0}(1)

Έχουμε: γ+α-β=y>Ο (2) α+β-γ=ω>Ο (3)

Προσθέτοντας τις ( 1 ), (2), (3) κατά μέλη βρί­σκουμε: α+β+γ=χ+y+ω (4). Αφαιρώντας από την (4) τις (1), (2), (3) διαδοχικά βρίσκουμε: 2α = y +ω, 2β =ω+ χ , 2γ = χ+ y , δηλαδή

α= y+ω β= ω+ χ x+y . . 2

, 2

, γ=-2-

, οποτε αρκει να

δείξουμε ότι:

(y+ω)(ω+χ)(χ + y);;::8xyω.

Η τελευταία προκύπτει από πολλαπλασιασμό κατά μέλη των προφανών ανισοτήτων

y+ω;;::2�} ω+χ;;::2� x+y;;::2FY

4. Δίνεται η συνάρτηση f (χ) = 2συνχ ( 1 + ημχ) χ ε ( 0, π].

α) Να μελετηθεί ως προς τη μονοτονία και τα ακρότατα.

β) Να αποδείξετε ότι 3J3 η μΑ+ ημΒ + ημΓ � -- αν Α, Β, Γ τα μέτρα 2

γωνιών τριγώνου σε ακτίνια.

Απάντηση απ' τη Σ. Ε.

Κατ' αρχάς παρατηρούμε ότι αφού μας εν­διαφέρει τελικά το μέγιστο της συνάρτησης

r(Γ )= 2συν Γ (l+ημ Γ ) με Ο< Γ� π αρκεί να 2 2 2 2 2

μελετήσουμε ως προς τη μονοτονία και τα ακρό­τατα τη συνάρτηση f (χ ) = 2συνχ (1 + ημχ) στο

διάστημα (ο.�]. α) f'(x) = 2(1 + ημχ )(ι-2ημχ )ι( Ο,�].

f'(x)=O <=> ημχ =_!_ <=> χ=�. 2 6

f'(x) >ο<=> ι - 2ημχ >ο<=> ημχ <_!_<=>ο� χ<�. 2 6

Συγκροτούμε τον πίνακα μεταβολών: Απάντηση απ' τους συνάδελφους Αθαν. Κα­

ΕΥΚΛΕΙΔΗΣ Β' λ.ε. τ.Ι/10

Page 13: Ευκλειδης Β 41

Ο Ευκλείδης προτείνει . .. ΕυΚλείδη και . . . Διόφαντο

χ ο π/6 π/2

f(x) + -

f(x) TJE._/ T.M.�Tf: /" '\ (ολικό) / )\ 2 -4

β) Έχουμε: Α+Β Α-Β Γ ημΑ + ημΒ = 2ημ--συν--::;; συν-= 2 2 2 , Α+Β Γ αφου ημ--=συν-> Ο και ::;; 1 . 2 2

Συνεπώς: ημΑ +ημΒ+ ημΓ::;; 2συν� + 2ημ� συν�= 2 2 2 ::;;2συν�(l+ημ � )=r(�)::;; 3� .

τ , Γ π , Γ π 'θ ο «=» ισχυει για - = - η = - οποτε α 2 6 3

, , μΑ μΒ μΓ 3J3 ' πρεπει να εχουμε η +η +η = -2- η επει-δή Γ = π οπότε ημΓ = -J3 θα πρέπει 3 2 ημΑ+ημΒ=J3.

Α+Β Α-Β r;; Γ Α-Β r;;

2ημ--συν--=-ν3 <=> 2συν-συν--=ν3 2 2 2 2 J3 Α-Β r;; Α-Β <::::> 2-συν--=...;3 <::::>συν--=1 . 2 2 2 π Α-Β π , κι επειδή -2 < -2- < 2' συμπαιραινουμε

ότι: Α=Β Οπ, Α Β π ' Γ π Ά ' οτε: = =- αφου =-. ρα η παρα-3 3 σταση Κ = η μΑ + ημΒ + ημΓ γίνεται μέγιστη όταν

Α Β Γ π δ λαδ' ' ' ' , = = = 3 η η οταν το τριγωνο ει ναι ισο-πλευρο.

• ΤΕΧΝΙΚΑ • ΓΙΑ ΤΑ Α.Ε.Ι. • T.E.I. • Ι.Ε.Κ.

�> ΘΠΙΚΩΝ ΚΑΙ ΘΕΩΡΗΠΚΩΝ ΕΠΙΠΗΜΩΝ

ΒΙΒΛΙΟΠΩΛΕΙΟ ΕΚΔΟΣΕΙΣ ΖΗΤΗ Αρμενοπούλου 27 • ΘΕΣΣΑΛΟΝΙΚΗ 546 35 Τηλ. (031)203.720 • Fax (031)211.305

e-mall:[email protected]

ΠΛΗΡΕΙΣ ΣΕΙ

ΕΚΠΑΙΔΕΥΤΙΚΑ �> ΓΙΑ ΤΟ ΛΥΚΕΙΟ (ΓΕΝΙΚΟ • ΚΑΤΕΥΘΥΝΣΕΩΝ)

� ΓΙΑ ΤΟ ΓΥΜΝΑΣΙΟ

ΒΙΒΛΙΟΠΩΛΕΙΟ ΑΘΗΝΩΝ ΕΝΩΣΗ ΕΚΔΟΤΩΝ ΒΙΒΛΙΟΥ ΘΕΣΣΑΛΟΝΙΚΗΣ Στοά του Βιβλίου (Πεσμαζόγλου 5)• ΑΘΗΝΑ 105 64 Τηλ.·Faχ (01)3211097

IKf)N ΒΙΒΛΙΩΝ " · .. , ,

Γ ι Α Τ Ο Λ Υ Κ Ε�Ιό ))1''· ;i''0:Ά.: .rt:� ΙΟ'tΓ:�'οΝ!�Ν (ΓΕΝΙΚΟ·ΚΑΤΕΥ Θ ΥΝΣΕΩΝ) ·, ,ι ' <\, < .. ·

αναλυτικά προγράμματα ' ; , j�•'i';'"�'Q fi ι βλί.c:ι. UOC:'•tlC]· Τ(]"ΙjιD.€ί1Γ€

..,..-------, θ. ΞΕΝΟΣ

ΤΡΑΠΕΖΑ θΕΜΑΤΩΝ ΠΑ ΜΑθΗΜΑτtΚΑ

ΠΡΟΒΛΗΜΑΤΑ MAθHMAfiKQN Β' ΛΥΚΕΙΟΥ

Α' ΛΥΚΕΙΟΥ .. (:ΛΥΚΕΙΟΥ ΚΑΙ ΕΥΚΛ::Ί{ε:��οΜετΡ,�f• •• . ,;:C· ι, ;·\/ � , ....

. ,,· -··

ΕΥΚΛΕΙΔΗΣ Β' λ.ε. τ.l/11

!Σ:IBt!lt !OΓlZ:t(Of !ΥΝι\ΡΤΙΣΒ�

MI.U: 11PAr}lλ?Jt:IJΣ llll'f.\IA.JTΉΣ

Page 14: Ευκλειδης Β 41

Η ιι d.,... μ ιι ι:.., ιt-tJ-1. Δ.., ιι r ω v--ι d μ.(), 1. Η ιtdf<\. μιιι:ιιιιιέ� Ο �41 μf1,ιι��ε�

Επιμέλεια: Α. Φελούρης - Σ. Λουρίδας

ilιslrsis ΙΙ•I•ιι•τιιιιί ιfι•ΥΙJΥΙtΙ'Ι'Ιί 2001 5" Βιιλιιιινιιιiι ••Ι•ιι•τιιιiι IJ λοpιιιίtlιι JI66Jν (Αwιι6J6ίιι}

1r Β•λιιιινιιιiι ••Ι•pιιτιιιiι ΙJλοpιιιίtl• (Β&λιyιίtlι} 4%" Aιslνiιs ••l•pιιτιιιίι IJJopιιιίfllλ (IJDA6ινyη•νJ

ΜeyάΛeς Διe8νeίς enιwχίeς -r;ωv eλλfjvωv μα8ητ;ώv στ;α Μα8ηματ;ικά Στην Κύπρο διεξήχθη η 5η Βαλκανική Μαθηματική Ολυμπιάδα Νέων (για μαθητές μέχρι 15,5 ετών) από 1 7

έως 23 Ιουνίου 2001 . Συμμετείχαν 8 χώρες με εξαμελείς εθνικές ομάδες. Και φέτος όλοι οι έλληνες μαθητές που έλαβαν μέρος και οι οποίοι διακρίθηκαν στους διαγωνισμούς της

Ε.Μ.Ε. κατόρθωσαν να διάκριθούν σ' αυτή τη διοργάνωση. Συγκεκριμένα: Αρετάκης Στέφανος Πειραματικό Γυμ. Πατρών Χρυσό μετάλλιο Γαλάνης Ανδρέας 3° Γυμ. Χαλανδρίου Αργυρό μετάλλιο Κερασιώτης Παρασκευάς 2° Πειρ/κό Γυμ. Αθηνών Αργυρό μετάλλιο Σγουρίτσα Αλκμήνη 2° Γυμ. Χολαργού Αργυρό μετάλλιο Χαλούλος Κων/νος 1° Γυμ. Χολαργού Αργυρό μετάλλιο Ασλανίδου Ιωάννα 2° Λύκειο Ανατόλια Χάλκινο μετάλλιο

Συνοδοί των μαθητών ήταν οι Καθηγητές Μαθηματικών κ.κ. Σωτήρης Λουρίδας (αρχηγός της ομάδας) και Δούναβης Αντώνιος (υπαρχηγός της ομάδας).

Εξάλλου από 3 - 9 Μαϊου πραγματοποιήθηκε στο Βελιγράδι της Νέας Γιουγκοσλαβίας και η 1 8η Βαλκανική Μαθημαnκή Ολυμπιάδα, όπου η Ελληνική ομάδα είχε εξίσου μεγάλες επιτυχίες:

Συμμετείχαν και διακρίθηκαν οι μαθητές μας: Μετσοβίτης Ευστάθιος 2° Λύκειο Πύργου Παπαλάμπρος Μιχαήλ 1° Πειρ/κό Λύκειο Αθηνών Ράπτης Γέωργιος 41° Λύκειο Αθηνών Σταυρόπουλος Κων/νος 1° Λύκειο Αμαλιάδας Πουλόπουλος Δημήτριος 3° Λύκειο Κορυδαλλού Ραυτοπούλου Αθηνά 1° Λύκειο Πρέβεζας

Χάλκινο μετάλλιο Χάλκινο μετάλλιο Χάλκινο μετάλλιο Χάλκινο μετάλλιο

Συνοδοί ήταν ο Καθηγητής του Πανεπιστημίου Ιωαννίνων κ. Θεόδωρος Μπόλης (αρχηγός της ομάδας)και ο καθηγητής Μαθημαnκών κ. Ανδρέας Τριανταφύλλου (υπαρχηγός της ομάδας).

Από 1 έως 14 Ιουλίου 2001 έγινε στην Ουάσινγκον των Η.Π.Α. η 42η Διεθνής Μαθημαnκή Ολυμπιάδα, ό­που η Ελληνική ομάδα κατέλαβε την 28η θέση μεταξύ 83 χωρών.

Συμμετείχαν και διακρίθηκαν οι μαθητές μας: Ράπτης Γέωργιος 41° Λύκειο Αθηνών Δήμου Αναστάσιος Πειρ/κό Λύκειο Πατρών Μετσοβίτης Ευστάθιος 2° Λύκειο Πύργου Πουλόπουλος Δημήτριος 3° Λύκειο Κορυδαλλού Αϊβαζίδης Στέφανος 12° Λύκειο Πειραιά Παπαλάμπρος Μιχαήλ 1 ° Πειρ/κό Λύκειο Αθηνών

(Αργυρό μετάλλιο), (Χάλκινο μετάλλιο), (Χάλκινο μετάλλιο), (Χάλκινο μετάλλιο), (Εύφημη μνεία),

Τους μαθητές συνόδεψαν ο Επίκουρος Καθηγητής του Εθνικού Μετσοβείου Πολυτεχνείου κ. Ανάργυρος Φελλούρης (αρχηγός της ομάδας) ο καθηγητής Μαθημαnκών κ. Σωτήρης Λουρίδας (υπαρχηγός της ομάδας) και ως παρατηρητές της διοργάνωσης ο Πρόεδρος της Ε.Μ.Ε. κ. Νικόλαος Αλεξανδρής (Καθηγητής Πανεπιστημίου Πειραιά) και ο Αντιπρόεδρος της Ε.Μ.Ε. κ. Γεώργιος Δημάκος (Λέκτορας Πανεπιστημίου Αθηνών).

Το περιοδικό μας συγχαίρει τους μαθητές αυτούς και τους εύχεται ολόψυχα καλή επιτυχία για το μtλλον. Οι επιδόσεις αυτές των Ελλήνων μαθητών δημιουργούν τις καλύτερες προϋποθέσεις για μια επιτυχημένη εμ­

φάνιση της Ελλάδας στη Διεθνή Μαθηματική Ολυμπιάδα του 2004, που θα διοργανωθεί στην Αθήνα από την Ελληνική Μαθηματική Εταιρεία στο πλαίσιο της Πολιτισnκής Ολυμπιάδας 2004.

ΕΥΚΛΕΙΔΗΣ Β' λ.ε. τ.l/12

Page 15: Ευκλειδης Β 41

Μαθηματικοί Διαγωνισμοί - Μαθητικές Ολυμπιάδες

Τόπος Διοργάνωσης : Η.Π.Α. - Ουάσινγκτον ΠΡΟΒΛΗΜΑ Ι Έστω οξυγώνιο τρίΎωνο ABC και Ο το κέ­

ντρο του περιγqραμμένου κύκλου του. Έστω ΑΡ το ύψος από την κορυφή Α με το σημείο Ρ πάνω στην πλευρά BC.

Δίνεται ότι C � Β + 30° . Α Λ

Να αποδείξετε ότι: Α+ COP < 90° . Λύση

Λ Οι γωνίες Α και BOC είναι εγγεγραμμένη

και επίκεντρη, αντιστοίχως, στο τόξο BC του περι­γεγραμμένου κύκλου, οπότε Α Λ Λ

2Α = BOC = ιgοο - 2 · PCO Α Λ

ή ισοδύναμα Α = 90° - PCO (ι) Επομένως αρκεί να αποδείξουμε ότι

Λ Λ PCO > COP , ή ισοδύναμα ΟΡ > PC .

Παρατηρούμε ότι ΒΡ = ΑΒσυνΒ = 2RημCσυνΒ και

PC = ΑCσυνC = 2RημΒσυνC , οπότε ΒΡ - PC = 2R ( ημCσυνΒ -ημΒσυνC) =

= 2Rημ (C - Β) (2) Όμως έχουμε από την υπόθεση ότι το τρίγωνο

ABC είναι οξυγώνιο και C-Β � 30° , οπότε θα εί-ναι 30° � C -B<90°

.!_� ημ(C-Β)<ι (3) 2 Λόγω της (3), από την σχέση (2) προκύπτει ότι

BP -PC � 2R ·.!_ = R (4) 2 Επομένως έχουμε

OP >PC<:=>OP+R >PC+R <:=> OP+OB > PC+R ή αρκεί ΟΡ+ΟΒ > ΒΡ, [λόγω της (4)] η οποία ισχύει, λόγω της τριγωνικής ανισότητας στο τρίγωνο ΟΒΡ.

Έτσι από το τρίγωνο OPC με ΟΡ > PC προ-Λ Λ

κύπτει ότι PCO > POC , οπότε από την (ι) προκύ-Α Λ Λ

πτει ότι Α = 90° - PCO < 90° -POC .

2°ς τρόπος

Λ Λ Λ Επειδή C-Β = ΟΑΡ όπου ΑΡ το ύψος του

τριγώνου ABC , από το τρίγωνο OAD έχουμε: � R ΜΡ = ΟD = 0Αημ0ΑD > Rημ30ο = 2 (ι). Από το ισοσκελές τρίγωνο BOG έχουμε:

OB+OC > BC = 2MC 2R > 2MC

R > MC (2). Επειδή υποθέσαμε ότι C � 90° θα είναι

MP+PC =MC και από τις (ι) και (2) έχουμε: R R > MC = MP+PC > -+PC 2

R - > PC 2 ' οπότε λόγω της (ι) προκύπτει ότι

MP > PC

(3)

(4) Από το ορθογώνιο τρίγωνο ΟΜΡ έχουμε ακό­

μη ότι ΟΡ � MR (η ισότητα ισχύει όταν Ο = Μ ), οπότε λόγω της (4) λαμβάνουμε

OP > PC Λ Λ

OCP > POC ; Λ Λ Λ

Άρα έχουμε Α = 90° -OCP <90° -POC . 3°ς τρόπος Όπως και στις δύο προηγούμενες λύσεις, αρ­

κεί να δείξουμε ότι PC < ΟΡ. Έχουμε PC2 < ΟΡ2 <=> PC2 < ΟΜ2 + ΜΡ2

<=> PC2 < ΟΜ2 + (CM -PC)2 <=> 0<0Μ2 + CM2 -2CM · PC <=> Ο< R 2 -BC · PC

Λ Λ Λ <=>0<R2 -(2RσυνΑ)(2RημΒσυνC) <=> 4ημΑημΒσυνC <ι

Λ Λ Λ Λ <=> 2ημΑ[ ημ(C+ Β) - ημ(C-Β)]< ι

η οποία αληθεύει γιατί Λ Λ ι ημΑ <ι και ημ(C- Β) � ημ30ο = 2.

ΕΥΚΛΕΙΔΗΣ Β' λ.ε. τ.l/13

Page 16: Ευκλειδης Β 41

Μαθηματικοί Διαγωνισμοί - Μαθητικές Ολυμπιάδες

ΠΡΟΒΛΗΜΑ 2 Ν α αποδείξετε ότι

a b c > l -Ja2 + 8bc

+ -Jb2 + 8ca

+ -Jc2 + 8ab -

για όλους τους θετικούς πραγματικούς α­ριθμούς a, b και c.

Λύση Επειδή η παράσταση του πρώτου μέλους της

ανισότητας είναι κυκλικά συμμετρική ως προς a, b, c, αρκεί να αποδείξουμε ότι υπάρχει k Ε � έτσι

k , , a > a (l ) ωστε να ισχυει 1 - k k k "a 2 + 8bc a + b + c για όλους τους θετικούς πραγματικούς αριθμούς a, b και c.

Με ύψωση στο τετράγωνο και πράξεις, η ( 1) γίνεται ισοδύναμη προς την

a2 (ak + bk +ck )2 � a2k (a2 +8bc)

<=> (ak + bk + ck )2 � a2k +8a2k-2bc <=> (ak + bk + ck )2 - (ak )2 � 8a2k-2bc <::::> (bk +ck )(ak + bk +ck +ak ) � 8a2k-2bc (2) Με χρήση της ανισότητας αριθμητικού-

γεωμετρικού μέσου δύο φορές στο πρώτο μέλος η (2) αληθεύει, αν αληθεύει η ανισότητα

2bYzcYz · 4aXbXcXaX � 8a 2k-2bc k/ 3k/ 3k/ <=> 8a72b Ι4c 74 � 8a 2k-2bc,

k η οποία αληθεύει (ως ισότητα) όταν -= 2k - 2

2

και 3k = 1 δηλαδή όταν k = i . 4 3

'Ετσι από την ( 1) για k = ! θα έχουμε: 3 a ak

�====

= >�

--�

-Ja2 +8bc - ak + bk +ck

b bk -r===== > �--�� -Jb2 +8ca - ak + bk +ck

c ck �=== > --;----:---. -Jc2 +8ab - ak +bk +ck '

από τις οποίες με πρόσθεση κατά μέλη προκύπτει η ζητούμενη ανισότητα.

2°ς τρόπος Θεωρούμε τη συνάρτηση

a b c f ( a, b, c) = + + 1 , -Ja2 +8bc -Jb2 +8ca -Jc2 +8ab

με a, b, c > Ο , οπότε η ανισότητα γίνεται: f(a,b,c ) � Ο .

Επειδή για κάθε t > Ο έχουμε f( ta, tb, tc) = f(a, b,c) = t0f(a, b,c) ,

η συνάρτηση f, άρα και η ανισότητα, είναι ομογε­νής μηδενικού βαθμού, οπότε με κατάλληλη επι-

λ ' 1 ' βλ ' ογη του t = 1 ,.--;-- το γινομενο των μετα ητων ιιabc

a'=ta, b '=tb, c'=tc ισούται με 1. Έτσι αρκεί να α­ποδείξουμε την ανισότητα f(a',b', c') � O με a', b', c' = 1 . Επομένως, χωρίς βλάβη της γενικότη­τας μπορούμε να θεωρήσουμε ότι για τους αριθ­μούς a, b, c ισχύει η ισότητα abc = 1 .

Η ανισότητα γράφεται ισοδύναμα ως

�+�+� � ι . (Ι) l + 8ca l + 8ab

b2 c2 και επειδή υποθέσαμε ότι abc = 1 καταλήγουμε στην ισοδύναμη ανισότητα

4+4+4 � � .

1+_! 1+_! b3 c3

(2)

από την οποία, αν θέσουμε χ = a -3 , y = b -3 , z = c -3 , λαμβάνουμε προς απόδειξη την ανισότητα

1 + 1 + 1 > 1 (3) .)ι+ 8χ -Jι + 8y .Jι + 8z '

για όλους τους θετικούς πραγματικούς αριθμούς χ, y και z με xyz = 1 .

Επομένως, αρκεί να βρούμε κάποιο k Ε � τέ­τοιο ώστε να ισχύει

1 xk --== > (4) ..f1+8x xk +yk +zk

για όλους tους θετικούς πραγματικούς αριθμούς χ, y, z με xyz = 1 .

Γ δ δ ' ' 1 ' ια ε ομενο χ το γινομενο yz =- ειναι στα-

χ θερό, οπότε το ελάχιστο του αθροίσματος yk + zk

λ β ' ' 1 ( ' ' αμ ανεται οταν y = z = .J;. απο ανισοτητα α-

ριθμητικού-γεωμετρικού μέσου) και είναι

min(yk + zk ) = 2χ -Yz . Επομένως αρκεί να αποδείξουμε ότι για κάθε

χ > Ο αληθεύει η ανισότητα 1 xk 1 1

--=== > -- ---...,-

../1+8χ - xk +2x-Yz-

1+2x-3Yz - 1+2xd '

ή ισοδύναμα 1 + 8χ � (ι+ 2xd )2 (5) xd + x2d <::::> 1+8x� 1+4xd+4x2d <::::> χ� ----

2

ΕΥΚΛΕΙΔΗΣ Β' λ.ε. τ.l/14

Page 17: Ευκλειδης Β 41

Μαθηματικοί Διιryωνισμοί - Μαθητικές Ολυμπιάδες ή αρκεί χ � .Jxdx2d = χ3% , η οποία αληθεύει για

2 d = - . 3 Στη συνέχεια εργαζόμαστε μέσω της (4), όπως

και στον πρώτο τρόπο. 3ος τρόπος Θα δώσουμε μία ακόμη λύση με τη βοήθεια

της Ανάλυσης, η οποία οφείλεται στον αρχηγό της Πολωνικής ομάδας Μ. Cuczma.

Λόγω ομογένειας, υποθέτουμε όn abc = ι , ο-

πότε έzουιu: J • = R = (ι + Sa _, )-Υ, a2 + 8bc a2 + !

. a και ομοίως

b (ι +sb-η-Yz c (ι+8c-η-Υz , .Jb2 + 8ca ' .J c2 + 8ab

Θέτουμε a -3 = ex b -3 = eY και c -3 = e2 οπό-' '

τε λόγω της σχέσης abc = ι προκύπτει όn χ + y + z = Ο . Θεωρούμε τη συνάρτηση

f{t) = (ι + 8e1 Γ Yz , t e JR . 'Ετσι αρκεί να αποδεί­ξουμε όn f(x) + f (y)+f (z) � ι .

Η συνάρτηση αυτή έχει δεύτερη παράγωγο

f"(t) = 4e1 (ι + 8e1 Γ% (4e1 - ι) , οπότε είναι 1αιρτή στο διάστημα [ -ln 4, +οο) .

Διακρίνουμε nς περιπτώσεις: Ι. Έστω x,y,z � -ln4 .

Τότε από την ανισότητα του Jensen προκύπτει ότι f (x) + f (y)+f (z) � 3f( x +�+ z )= 3f (O) = l .

1 1 . Έστω x < -ln4 , y < -ln4

Τότε f (x) > �, f(y) > � και

2 f (x) + f(y)+ f(z) > f(x) + f(y) > J3 > ι .

ΙΠ.'Εστω χ < -ln4 � y,z Τότε έχουμε

f (x) + f (y) + f (z) � f (x) + 2f( y;z )=

= f (x) + 2f(-;}

οπότε αρκεί να αποδείξουμε ότι

f(x)+ 2f(-; )� ι , για χ < -ln4 .

Αν θέσουμε u = e-� , τότε u > 2 και αρκεί πλέον να αποδείξουμε την ανισότητα

u 2 � + � � l , για u > 2 νu- + δ '\/8U + ι

u 2 <=> � � ι - ι;;--:-;- , για u > 2 νu2 + 8 ν8u + ι

u2 4 4 <=> --� ι +--- , για u > 2 u2 + 8 8u + ι .Jsu + l

ι > u2 + 16u + 10 2 <=> � _ ( ) r;;--:-: , για u >

'\/8U + l u2 + 8 '\/8U + l η οποία με ύψωση στο τετράγωνο είναι ισοδύναμη προς την ανισότητα

(u - ι )2 (2u3 + 4u2 + 30u -9) � Ο , η οποία αληθεύει για u > 2 .

4°ς τρόπος [Σ. Λουρίδα] Θα χρησιμοποιήσουμε, κατά κύριο λόγο, την

γνωστή ανισότητα Cauchy-Schwarz-Buniakowski (C.S.B.)

(χ� + χ� + · . . + x� )(y� + y; + . . · + y� )�

(χΙ yΙ + Χ2Υ2 + · . . + ΧπΥπ )2 ·

Έστω Α a b c = + +��== .Ja2 +8bc .Jb2 +8ac .Jc2 + 8ab

και Β = a.Ja2 + 8bc + b.Jb2 +8ac + c.Jc2 + 8ab , ο­πότε από την ανισότητα (C.S.B.) θα έχουμε

Α· Β=[( #a':SOO J +( #b�Sac J +( #c':Sab η [ ..[a(�a2 +8bc)2 +.Jb(�b2 +8ac)2 +..fc(�c2 +8ab)2 ] � (a + b + c)2 . δηλαδή

Αρκεί B � (a+b+ c)2 <=> Β2 � (a +b+c)4 • Γνωρίζουμε όn

(ι)

3 .

(a + b+c) = a3 + b3 + c3 + 3(a + b)(b+ cXc +a) �

a3 + b3 +c3 + 24abc <*> Ταυτόχρονα:

� =(.Jiι.Ja3 +8abc +.Jb.Jb3 +8abc +..fc.Jc3 +8abc)2 �

Β2 � ((J;} + (Jb)2 + (..fc)2 ) ( (.Ja3 +8abc }2

+(.Jb3 + 8abc }2 + (.Jc3 + 8abc }2 ] = (a+b+c) ( a3+ +b3 + c3 + 24abc ), από την ανισότητα (C.S.B.).

Άρα B2 � (a +b + c)(a +b + c)3 δηλ. B2 � (a +b + c)4 δηλ. Β � (a + b + c)2 (2). Με βάση τις σχέσεις

ι•> Αφού "tx, y e JR; => x + y

� FY => χ+ y � 2,J;Y 2

ΕΥΚΛΕΙΔΗΣ Β' λ.ε. τ.Ι/15

Page 18: Ευκλειδης Β 41

Μαθηματικοί Διαγωνισμοί - Μαθητικές Ολυμπιάδες

( 1 ), (2) έχουμε: A·B�(a+b+c)2 ::::} ::::} A(a+b+c)2 � (a+b+c)2 ::::} Α � 1 .

ΠΡΟΒΛΗΜΑ 3 Εικοσιένα κορίτσια και εικοσιένα αγόρια

πήραν μέρος σε έναν μαθηματικό διαγωνισμό. • Κάθε διαγωνιζόμενος/η έλυσε το πολύ έξι

προβλήματα. • Για κάθε ζευγάρι από ένα κορίτσι και ένα α­

γόρι υπήρξε ένα τουλάχιστον πρόβλημα το οποίο λύθηκε και από το κορίτσι και από το αγόρι. Να αποδείξετε ότι υπήρξε ένα πρόβλημα το

οποίο λύθηκε από τρία τουλάχιστον κορίτσια και από τρία τουλάχιστον αγόρια.

Λύση Κατ' αρχήν χρησιμοποιούμε τους παρακάτω

συμβολισμούς: G = το σύνολο των κοριτσιών που πήραν μέ­

ρος στο διαγωνισμό, Β = το σύνολο των αγοριών που πήραν μέ­

ρος στο διαγωνισμό, P (g) = το σύνολο των προβλημάτων που λύθη­

καν από το g ε G , P (b) = το σύνολο των προβλημάτων που λύθη­

καν από το b ε Β , G (ρ) = το σύνολο των κοριτσιών που έλυσαν το

πρόβλημα ρ. Β (ρ) = το σύνολο των αγοριών που έλυσαν το

πρόβλημα ρ. Σύμφωνα με τα δεδομένα του προβλήματος,

για κάθε gε G και b ε Β , θα έχουμε:

IP (g)l ::; 6 , IP (b)l ::; 6 και P (g)nP (b) :;t: 0 , όπου με IP (g)l συμβολίζουμε το πλήθος των στοι­

χείων του συνόλου P(g) κλπ. Πρέπει να αποδείξουμε ότι για κάποιο πρό­

βλημα p ε Ρ ισχύουν ταυτόχρονα οι ανισότητες

IG (ρ )I � 3 και IB (ρ )I � 3 . Θα υποθέσουμε ότι δεν ισχύει το ζητούμενο

και θα προσπαθήσουμε να καταλήξουμε σε άτοπο aπαριθμώντας με δύο τρόπους όλες τις διατεταγ­μένες τριάδες (p, g, b) με ρε P(g)nP (b) .

Έτσι, αν θεωρήσουμε το σύνολο T ={(p, g, b) : p ε P (g)nP (b)} , η δεύτερη συνθή­

κη του προβλήματος μας δίνει τη σχέση ITI = Σ ΣIP(g)nP (b)I � IGI · IBI = 212 (1 )

Υποθέτουμε ότι δεν υπάρχει ρε Ρ που να ι-

κανοποιεί ταυτόχρονα τις ανισότητες IG (ρ )I � 3 και IB (p)l � 3 .

Κατ' αρχήν παρατηρούμε ότι

ΣIG(p )I = Σ IP(g)l :::; 6IGI και ΣIB(p )I :::; 6 IBI ,(2) peP geG peP

όπου η ισότητα των δύο πρώτων αθροισμάτων της (2) προκύπτει μέσω της συνάρτησης

χ ( ) = {1, αν το g έλυσε το πρόβλημα p g, Ρ Ο, αν το g δεν έλυσε το πρόβλημα p,

και των ισοτήτων ΣIG(p)l = ΣΣχ(g,p) = ΣΣχ(g,p) = ΣIP(g)l peP geG peP geG

Θεωρούμε και τα σύνολα Ρ+ = {ρε P : la (p)l � 3} Ρ_ = {ρε P : IG (p)l ::; 2} . Θα αποδείξουμε ότι

Σ IG (p )I � IGI και Σ IB (p )I � IBI , (3) peP_

οπότε αμέσως προκύπτουν οι ανισότητες Σ IG(p)I ::; 5 IGI και Σ IB(p)I ::; 5 IBI . pE� pE �

Πράγματι, αν θεωρήσουμε τυχαίο g ε G , τότε σύμφωνα με την αρχή της περιστεροφωλιάς, οι δύο συνθήκες του προβλήματος δίνουν ότι το κο­ρίτσι g έλυσε κάποιο πρόβλημα p το οποίο λύθηκε

' λ ' 2 1 ' δ λαδ' ' λ ' απο του αχιστον - αγορια, η η απο του α-

6 χιστον 4 αγόρια, δηλαδή IB (p )I � 4 , οπότε ρε Ρ_ . Επομένως κάθε κορίτσι έλυσε τουλάχιστον ένα πρόβλημα του συνόλου Ρ_ και έτσι

Σ IG (ρ )I � IGI . peP_

Από τις (2) και (4) προκύπτει ότι Σ IG(p)l = ΣG(p)- Σ IG (p)I ::; 5 IGI . pE � peP pE �

(4)

Ομοίως κάθε αγόρι έλυσε ένα πρόβλημα που λύθηκε από τουλάχιστον 4 κορίτσια, οπότε κάθε αγόρι έλυσε ένα τουλάχιστον πρόβλημα του συνό­λου Ρ+ . Έτσι θα έχουμε τις σχέσεις

Σ IB(p)I � IBI και Σ IB(p)I � IBI . (5) peP_

Χρησιμοποιώντας τις (4) και (5) υπολογίζουμε ITI = Σ IG (p)I · IB(p)l

peP_

:::; 2 Σ IG(p)l +2 ΣIB(p)l ::; 1oiGI +IOIBI =20 · 21 , peP. peP_

ΕΥΚΛΕΙΔΗΣ Β' λ.ε. τ.l/16

Page 19: Ευκλειδης Β 41

Μαθηματικοί ΔιαΎωνισμοί - Μαθητικές Ολυμπιάδες

το οποίο είναι αντίθετο προς τη σχέση ( 1 ), όπου είναι ιτι � 2 12 •

2°ς τρόπος Χρησιμοποιούμε τους συμβολισμούς της πρώ­

της λύσης και υποθέτουμε ότι για κάθε p ε Ρ ι-

σχύει IG (p )I ::;2 ή IB(p )I � 2 . Για κάθε p ε Ρ , χρωματίζουμε το p κόκκινο,

αν Ι α (ρ )I � 2 , ενώ το χρωματίζουμε μαύρο, αν

IG (ρ )I � 3 . Όμως σύμφωνα με την υπόθεση που

κάναμε, αν Ι α (p )I � 3 , τότε ισοδύναμα θα ισχύει

IB(p)l � 2 , οπότε αν με X(p) συμβολίσουμε το

χρώμα του προβλήματος p, τότε

X(p) = {κόκκινο, αν Ι α (p )I � 2 μαύρο, αν IB (ρ )I � 2.

Θεωρούμε έναν πίνακα με 2 1 γραμμές, που η καθεμία αντιστοιχεί σε ένα κορίτσι, και με 2 1 στή­λες, που η καθεμία αντιστοιχεί σε ένα αγόρι.

Για κάθε g Ε G και b Ε Β χρωματίζουμε το τε-

τράγωνο που αντιστοιχεί στο ζευγάρι (g, b) ως εξής:

Θεωρούμε πρόβλημα ρε P (g)nP(b) και a­

ντιστοιχούμε στο τετράγωνο (g, b) το χρώμα του προβλήματος p, (υπάρχει ένα τουλάχιστον τέτοιο p, λόγω της δεύτερης συνθήκης του προβλήματος). Σύμφωνα με την αρχή της περιστεροφωλιάς, ένα από τα δύο χρώματα aντιστοιχίζεται τουλάχιστον

σε 441

τετράγωνα, δηλαδή τουλάχιστον σε 221 2 τετράγωνα. Έτσι κάποια γραμμή θα έχει τουλάχι-

221 ' ' δ λ δ ' θ ' στον 2ι μαυρα τετραγωνα, η α η α εχει του-

λάχιστον 1 1 μαύρα τετράγωνα ή κάποια στήλη θα έχει τουλάχιστον 1 1 κόκκινα τετράγωνα.

Υποθέτουμε όn η γραμμή που αντιστοιχεί στο κορίτσι g ε G έχει τουλάχιστον 1 1 μαύρα τετρά-γωνα. Τότε για καθένα από αυτά τα 1 1 τετράγωνα το μαύρο πρόβλημα που χρησιμοποιήθηκε για τον χρωματισμό του τετραγώνου λύθηκε το πολύ από 2 αγόρια. Έτσι το κορίτσι g πρέπει να έχει λύσει

τουλάχιστον .!...!. διαφορετικά προβλήματα, δηλαδή 2 τουλάχιστον 6 διαφορετικά προβλήματα. Λόγω της πρώτης συνθήκης του προβλήματος, το κορί­τσι g έλυσε ακριβώς αυτά τα 6 προβλήματα. Τότε όμως το πολύ 12 αγόρια έλυσαν κάποιο πρόβλημα που λύθηκε από το κορίτσι g, που είναι αντίθετο με τη δεύτερη συνθήκη του προβλήματος.

Ομοίως καταλήγουμε σε άτοπο, αν υποθέσου­με ότι κάποια στήλη έχει τουλάχιστον 1 1 κόκκινα

τετράγωνα. Άρα για κάποιο πρόβλημα p ε Ρ θα ισχύει

IG (p)l � 3 και IB(p)l � 3 . 3°ς τρόπος Θεωρούμε έναν πίνακα 2 1 χ 2 1 και στο τετρά­

γωνο που ανήκει στη i - και στην στήλη στήλη το­ποθετούμε ένα από τα προβλήματα που επιλύθη­καν και από το αγόρι i και από το κορίτσι j .

Σύμφωνα με την πρώτη συνθήκη του προβλή­ματος, σε κάθε γραμμή του πίνακα θα υπάρχουν το πολύ 6 διαφορετικά προβλήματα. Εύκολα διαπι­στώνουμε όn το πολύ 5 προβλήματα μπορούν να εμφανίζονται σε μια γραμμή το πολύ 2 φορές, για­τί διαφορεnκά θα συμπληρώναμε το πολύ 6χ2 = 12 από τα 2 1 τετράγωνα της γραμμής, που αντίκειται στη δεύτερη συνθήκη του προβλήματος. Επομένως τουλάχιστον 1 1 , δηλαδή περισσότερα από τα μισά τετράγωνα κάθε γραμμής θα συμπλη­ρώνονται με προβλήματα που εμφανίζονται στη γραμμή τουλάχιστον 3 φορές.

Το ίδιο ακριβώς συμπέρασμα μπορεί να προ­κύψει και για κάθε στήλη του πίνακα.

Επομένως υπάρχει ένα τουλάχιστον τετράγω­νο του πίνακα που εμφανίζεται 3 τουλάχιστον φο­ρές στη γραμμή του και 3 τουλάχιστον φορές στη στήλη του.

ΠΡΟΒΛΗΜΑ 4 Έστω n ένας περιττός ακέραιος μqαλύτε­

ρος του 1, και έστω k1 , k2 , ••• , kn δεδομένοι α­κέραιοL Για καθεμιά από τις n ! μεταθέσεις a = (a1 ,a2 , . . . ,a0 ) του συνόλου {1,2, . . . ,n} ορί-

Ω

ζουμε S(a) = Σk1a1 • 1=1

Να αποδείξετε ότι υπάρχουν δύο μεταθέσεις b και c με b :;t: c , έτσι ώστε ο αριθμός n ! να εί-ναι διαιρέτης του S(b )-S (c) .

Λύση Θα χρησιμοποιήσουμε απαγωγή σε άτοπο, η

οποία οδηγεί στον υπολογισμό του αθροίσματος Σs(a)(modη !) πάνω στις η ! μεταθέσεις του

συνόλου {1, 2, . . . ,η} . Στη συνέχεια θα υπολογί­

σουμε το άθροισμα Σs(a) και με άλλο τρόπο και

θα προσπαθήσουμε για η περιττό να καταλήξουμε σε άτοπο.

Αν υποθέσουμε ότι ο η! δεν είναι διαιρέτης του S(b) - S(c) για κάθε b :;t: c , τότε κάθε S(a) πρέπει να έχει διαφορετικό υπόλοιπο όταν διαιρεί­ται με το η ! και αυτά πρέπει να είναι ακριβώς οι αριθμοί Ο, 1 , 2, . . . , η !- 1 . Τότε θα έχουμε και ότι

ΕΥΚΛΕΙΔΗΣ Β' λ.ε. τ.l/17

Page 20: Ευκλειδης Β 41

Μαθηματικοί Διαγωνισμοί - Μαθητικές Ολυμπιάδες

Σs(a) = [ο+ 1 + 2 + · · · + (η!-Ο]mοdη !

Σs(a) = (η !-Οη! mοdη ! . ( 1 ) 2 Όμως στο άθροισμα Σs(a) , ο αριθμός k1

πολλαπλασιάζεται με κάθε αριθμό k του συνόλου {1,2, . . . ,η} συνολικά (η -0! φορές, δηλαδή μία φορά για καθεμία από τις (η -0! μεταθέσεις του συνόλου {1, 2, . . . , η} στις οποίες ο αριθμός k έχει τη θέση του aj . Έτσι ο συντελεστής του k1 στο ά-θροισμα Σs(a) είναι

(η - 1) !(1 + 2 + · · · + η) = (η + 1) ! . 2 Το ίδιο ισχύει και για τον συντελεστή του ki ,

i = 2, . . . , η , οπότε

(2) Από τις (1 ) και (2) έχουμε ότι

(η+Ο! (k1 +k2 + · · · +kn ) = (η!-Οη! (mοdη!) . (3) 2 2 Επειδή ο η είναι περιττός, ο η + 1 είναι άρτιος

(η + 1) ! , λλ λά ' , και ο ειναι πο απ σιο του η . , οποτε το 2 αριστερό μέλος της (3) είναι ισοϋπόλοιπο με το Ο modulo η ! , ενώ για η > 1 το δεξιό μέλος της (3) δεν είναι πολλαπλάσιο του η ! αφού ο η !- 1 είναι περιττός. Έτσι για η περιττό και μεγαλύτερο του 1 καταλήγουμε σε άτοπο.

ΠΡΟΒΛΗΜΑ S Σε τρίγωνο ABC, έστω ΑΡ η διχοτόμος της

Λ γωνίας BAC , με το σημείο Ρ πάνω στην πλευρά

Λ BC, και έστω BQ η διχοτόμος της γωνίας ABC , με το σημείο Q πάνω στην πλευρά CA. Δίνεται

Λ ότι BAC = 60° και ότι ΑΒ +ΒΡ = AQ+ QB .

Ποιες είναι οι δυνατές τιμές των γωνιών του τριγώνου ABC;

Λύση (1 ος τρόπος) Α

D Ε Προεκτείνουμε την πλευρά ΑΒ προς το μέρος

του Β κατά τμήμα BD = ΒΡ και την AQ κατά

τμήμα QE = QB . Τότε έχουμε AD=AB+BD=AB+BP= AQ+QB=AQ+QE = AE .

Άρα το τρίγωνο ADE είναι ισοσκελές με Λ

DAE = 60° , οπότε τελικά το ADE είναι ισόπλευρο τρίγωνο.

Επειδή η ΑΡ είναι η διχοτόμος της γωνίας DAE, εύκολα προκύπτει ότι PD = ΡΕ .

Στη συνέχεια θα αποδείξουμε Ε = C , ή ισοδύ­ναμα αρκεί να αποδείξουμε ότι τα σημεία Β, Ρ και Ε είναι συνευθειακά.

Υποθέτουμε ότι τα σημεία Β, Ρ και Ε δεν είναι συνευθειακά, οπότε σχηματίζεται το τρίγωνο ΒΡΕ.

Λ Λ Λ Β Έχουμε ότι PBQ = PDB = PEQ = - . Με τα Ρ, Q 2

να είναι στο ίδιο ημιεπίπεδο ως προς την ΒΕ, όπως στο σχήμα, έχουμε

Λ Λ Λ λ Λ Λ ΡΒΕ = EBQ- PBQ = BEQ- PEQ = ΡΕΒ . Η ίδια ισότητα προκύπτει ανάλογα και στην

περίπτωση που τα Q, Ρ βρίσκονται σε διαφορετικά ημιεπίπεδα ως προς την ΒΕ.

Επομένως θα έχουμε σε κάθε περίπτωση την Λ Λ

ισότητα ΡΒΕ = ΡΕΒ , από την οποία και την ( 1 ) προκύπτει η ισότητα των πλευρών

PB = PE = PD . (2) Επειδή έχουμε θεωρήσει BD = ΡΒ , από την

(2) προκύπτει ότι ΡΒ = BD = PD , οπότε το τρίγω­νο PBD είναι ισόπλευρο, που είναι άτοπο, γιατί

Λ Α τότε θα έχουμε ΑΒΡ = Β = 120° και Α+ Β = 60°+ 120° = 1 80° .

Άρα τα σημεία Β, Ρ και Ε είναι συνευθειακά, οπότε Ε = C . Από το τρίγωνο QBC έχουμε πλέον

ότι Β = C και αφού Β + C = 120° , εύκολα προκύ-2

πτει ότι Β = 80° και C = 40° . 2ος τρόπος (Β. J. Veηkatachala, Ινδία) Θεωρώντας BC = a , CA = b και ΑΒ = c έ-

χουμε ΒΡ = � BQ = 2ca �τ (τ-b) , b + c ' c + a ca AQ cb , , , = -- , οπου τ ειναι η ημιπεριμετρος του τρι-

c + a γώνου ABC. Έτσι η ισότητα ΑΒ + ΒΡ = AQ + QB γίνεται ισοδύναμη προς την ισότητα

a + b + c = __ b_+��τ(τ -b) b + c b + c c + a ca

<=> 4-Jc"τ ( τ·- b) + ab.Jc" = 2-.Γa (b + c)�r---τ(,----τ --b-,-) , η οποία με τον μετασχηματισμό ω = �τ ( τ - b) γί­νεται ισοδύναμη προς την εξίσωση

ΕΥΚΛΕΙΔΗΣ Β' λ.ε. τ.l/18

Page 21: Ευκλειδης Β 41

Μαθηματικοί Διαγωνισμοί - Μαθητικές Ολυμπιάδες

4Fcω2 - 2Ji. (b + c)ω + abFc = Ο , ' ' λ ' J;; ' Ji.b η οποια εχει τις :υσεις ω = -- η ω = Γ 2 2νc

Ι.

Διακρίνουμε τώρα δύο περιπτώσεις: J;; ac ω = - <=> τ(τ -b) = -2 4

<=> a2 + c2 + ac = b2 <=> συν Β = _ _.!._ 2

<=> Β = 120° [αδύνατο, αφού Α = 60° ]

11. ω =� <=:> c(a + c)2 = b2 (a + c) 2νc

<=> b2 = c (c + a) <=> b2 -c2 = ca <=> (2RημΒ )2 - ( 2RημC )2 = 2RημC · 2RημΑ <=> η μ 2Β -η μ 2C = ημCημΑ <=>ημ(Β-C)ημ(Β+C) =ημCημΑ <=:>ψ(Β-C) =ημC <=> Β -C = C [ Β , C γωνίες τριγώνου] <=> Β = 2C , οπότε αφού Β + C = 120° προιcύπτει ότι Β = 80° και C = 40° .

ΠΡΟΒΛΗΜΑ 6 Έστω a, b, c, d ακέραιοι αριθμοί με

a > b > c > d > Ο . Δίνεται ότι ac+ bd = (b +d+a- cHb+ d -a + c) .

Να αποδείξετε ότι ο αριθμός ab + cd δεν εί­ναι πρώτος.

Λύση (1 ος τρόπος), Υ. Chen, Κίνα. Η δεδομένη ισότητα μετά από πράξεις γίνεται a2 - ac + c2 = b2 + bd + d2 (1 ) .

Υποθέτουμε ότι ο αριθμός ab + cd = p είναι πρώτος. Τότε έχουμε a = (p - cd)b-1 και με αντι­κατάσταση στην ( 1) λαμβάνουμε

(p - cd)2 - (p -cd)cb + c2b2 = b2 (b2 + bd + d2 ) <=> p2 - 2cdp -cbp = (b2 -c2 ) (b2 + bd+ d2 ) <=> p(ab - 2cd -cb) = (b2 - c2 ) (b2 + bd+ d2 ) (2)

Επειδή έχουμε ότι 1 < b2 - c2 < ab < p , από την (2) έπεται ότι -Ρ Ι (b2 + bd + d2 ) (3)

Όμως από τις υποθέσεις a > b > c > d > Ο προκύπτει ότι b2 + bd + d2 < 2ab + cd < 2p , οπότε από την (3) λαμβάνουμε ότι

p = b2 +bd + d2 (4)

Τόπος Διοργάνωσης: Γιουγκοσλαβία (Βελιγράδι)

Εξισώνοντας τις εκφράσεις για το p λαμβά-νουμε b (b + d - a) = d(c -d) (5)

Επειδή (b,d) = 1 (διαφορετικά ο ab + cd δεν είναι πρώτος), από την (5) λαμβάνουμε ότι b Ι (c -d ) , που είναι άτοπο, γιατί Ο < c - d < b .

Άρα ο αριθμός ab + cd δεν είναι πρώτος. 2"ς τρόπος Η ισότητα ac+bd= (b+d+·a-c)(b+d-a+c)

είναι ισοδύναμη προς την ισότητα a2 - ac + c2 = b2 + bd + d2 , (1 )

από την οποία προσπαθούμε να βρούμε μια ισότη­τα της μορφής (r + s) (b2 + bd + d2 ) = (ab + cd) t . Για παράδειγμα, έχουμε

(ac + bd) (b2 + bd + d2 ) = = ac (b2 + bd + d2 ) + bd (a2 - ac + c2 ) = ab2c + acd2 + a2bd +b2cd = ab · bc + cd · ad + ab · ad + cd · bc = (ab + cd)(ad + bc) . (2) Στη συνέχεια παρατηρούμε ότι ισχύουν οι α-

νισότητες ab + cd > ac + bd > ad + bc , (3) οι οποίες προκύπτουν εύκολα από τις ανισότητες (a - d)(b - c) > O και (a -b)(c - d) > O .

Υποθέτουμε ότι ο αριθμός ab + cd είναι πρώ­τος. Τότε από την (3) έπεται ότι ( ab + cd, ac + bd) = 1 , οπότε από την (2) προκύπτει ότι (ac + bd) Ι (ad + bc) , 10 οποίο είναι αδύνατο λόγω της (3).

Επομένως ο αριθμός ab + cd δεν είναι πρώτος. Σημείωση: Η σχέση (2) μπορεί να προκύψει από την (1)

και γεωμετρικά, αν θεωρήσουμε εγγράψιμο τετρά­πλευρο ABCD με ΑΒ = a , BC = d , CD = b ,

Λ Λ AD = c , BAD = 60° και BCD = l20° .

Τότε τα δύο μέλη της (1 ) είναι ίσα με BD2 , ε­νώ με εφαρμογή του νόμου των συνημιτόνων στα τρίγωνα ABC και ACD προκύπτει τελικά ότι

AC2 = (ab + cd)(ac + bd) . ad +bc

Έτσι από το θεώρημα του Πτολεμαίου λαμβά­νουμε την ισότητα AC · BD = ab + cd , η οποία με ύψωση στο τετράγωνο και αντικατάσταση της τι­μής των AC2 και BD2 οδηγεί στη σχέση (2).

ΠΡΟΒΛΗΜΑ 1 Έστω η ένας θετικός ακέραιος αριθμός. Α­

ποδείξτε ότι αν a και b είναι ακέραιοι μεγαλύ-ΕΥΚΛΕΙΔΗΣ Β' λ.ε. τ.l/19

Page 22: Ευκλειδης Β 41

Μαθηματικοί Διαγωνισμοί - Μαθητικές Ολυμπιάδες

τεροι του ι τέτοιοι που 28 - ι = ab , τότε 0 αριθ- ριθμοί, τότε αυτό είναι ένα κανονικό πεντά-

μός ab - (a-b) - ι είναι της μορφής k . 22m ό-γωνο.

που k είναι περιττός και m είναι ένας θετικός α- Λύση (lος τρόπος)

κέρα� (Ι" τρ6πος) c"'�:�:--,�:- _ _ _ _ _ _ Α,

Κατ' αρχήν έχουμε: - _ _ _ _ Τ = ab - ( a - b) - Ι = ( a + Ι )(b - Ι) . Α;� _ � Από την ισότητα ab = 2" - Ι προκύπτει ότι οι αριθμοί a, b είναι περιττοί, οπότε οι αριθμοί a+ 1 και b-1 είναι άρτιοι. Επομένως θα έχουμε a + Ι = κ · 2"' και b - 1 =λ · 2"' , (1 )

με κ, λ περιττούς θετικούς ακέραιους και ν 1 , ν2 θε­τικούς ακέραιους. Τότε η ισότητα ab = 2" -1 γίνε-ται: (κ · 2"' - Ι) · (λ · 2"' + 1) = 2" - Ι

Έστω ότι το τετράγωνο Α1Α2Α3Α4Α5 έχει τις δεδομένες ιδιότητες, οπότε όλες οι εσωτερικές γω­νίες του είναι 108°. Έστω ακόμη ότι οι προεκτάσεις της Α1Α5 τέ-μνουν τις Α2Α3 και Α3Α4 στα σημεία C και D, α-

(2) ντιστοίχως. Τότε τα τρίγωνα A1CA2 , A4DA5 και Επίσης από την ισότητα ab = 2" - 1 και τις ( 1 ) λαμβάνουμε:

a = k · 2"' - 1 < 2" - l και b = λ · 2"' + 1 < 2" - l από τις οποίες προκύπτει ότι: ν1 < ν και ν2 < ν . (3)

Αν υποθέσουμε ότι ν1 :;t: ν2 και θέσουμε ω=mίη{ν1 ,ν2} , τότε από τη (2) διαιρώντας με 2ω λαμβάνουμε μια ισότητα της οποίας το πρώτο με­λος είναι περιττός θετικός ακέραιος, ενώ το δεύτε­ρο μέλος της είναι αρτιος θετικός ακέραιος ( άτο­πο). Άρα είναι ν 1 = ν 2 , οπότε

T = (a + l)(b - I) = κλ · 22"' = κ · 22m , αν θέσουμε κ = κλ (περιττός θετικός ακέραιος, ως γινόμενο περιττών θετικών ακεραίων) και m = ν1 •

2°ς τρόπος

Έστω deg2 χ ο μεγαλύτερος φυσικός k. που είναι τέτοιος ώστε 2k Ι χ .

Τότε, αν T = (a + l )(b - 1) , θα έχουμε deg2 τ = deg2 [(a + l ) (ab - 1)] = deg2 [(a + l ) (b - 1 )a

επειδή a περιττός. Επιπλέον έχουμε deg2 τ = deg2 [(a + l ) (ab - a)] =

deg2 [(a + 0(2" - 1 - a ) ]= deg2[c (2" -c )] , όπου c = a + 1 άρτιος και c < 2" .

Έστω deg2 c = k . Τότε: c = 2k λ, λε Ν * και συνεπώς: 2k ::; 2kλ = c < 2n κι επομένως k < n Έ­χουμε ακόμα κάτι: deg2 (2n - c) = k και deg2 τ = deg2 c + deg2 (2" - c) = 2k ο

ΠΡΟΒΛΗΜΑ 2

Ν δείξετε ότι, αν ένα κυρτό πεντάγωνο ικα­νοποιεί τις ιδιότητες που ακολουθούν (ι) όλες οι εσωτερικές του γωνίες είναι ίσες

CA3D είναι ισοσκελή, οπότε θα έχουμε: CA3 = A3D

CA2 + Α2Α3 = Α3Α4 + A4D Α2Α3 - Α3Α4 = A4D -CA2 ( 1 )

Επίσης έχουμε 2CΑ2συν72° = Α1Α2 και 2DΑ4συν72° = Α4Α5 , οπότε:

2συν72° (DA4 -CA2) = Α4Α5 - Α1Α2 και συνεπώς λόγω τις ( 1) έχουμε:

2συν72° (Α2Α3 - Α3Α4) = Α4Α5 - Α1Α2 (2) Αν ήταν Α2Α3 - Α3Α4 ::�; Ο , τότε από τη (2)

προκύπτει ότι αριθμός συν72° είναι ρητός αριθμός, που είναι άτοπο. Επομένως θα είναι Α2Α3 = Α3Α4 και Α4Α5 = Α1Α2 •

Ομοίως αποδεικνύεται η ισότητα ανά δύο όλων των πλευρών του πενταγώνου, οπότε είναι κανονικό. 2ος τρόπος (Σ. Λουρίδα)

Θα χρησιμοποιήσουμε ότι: ί) Το ημ1 8° (Ι) είναι άρρητος.

(Ι) Διότι 3 · 18° = 54°

2 · 1 8° = 36° 54° + 36° = 90°

ημ(3 · Ι 8° ) = συν(2 · 1 8°)

ή 3ημ1 8° - 4ημ3 1 8° = Ι - 2ημ2Ι8°

ή 4ημ3 1 8° - 2ημ2Ι 8° - 3ημ18° +Ι = 0 ο Αν: ημ1 8° = χ

Τότε: 4x3 - 2x2 - 3x + l = O . ( 1 ) k lk,λε Ζ

Έστω ρ= «ρητή ρίζα της ( 1)» Τότε: ρ = -λ λ :;t: Ο (k, λ) = l .

και

(2) τα μήκη όλων των πλευρών είναι ρητοί α-Έχουμε: 4k3 - 2k2λ - 3kλ2 + � = 0

ΕΥΚΛΕΙΔΗΣ Β' λ.ε. τ.l/20

Page 23: Ευκλειδης Β 41

Μαθηματικοί Διαγωνισμοί - Μαθητικές Ολυμπιάδες

ii) Ένα κυρτό πεντάγωνο με ίσες εσωτερικές γω­νίες και με πάνω από δύο πλευρές ίσες είναι κανονικό.

iii) Ένα ισοσκελές τρίγωνο ΑΒΓ (έστω ΑΒ = ΑΓ ) και Α = 36° δεν μπορεί να έχει όλες του τις πλευρές με μέτρα ρητούς αφού τότε το ημl8° θα ήταν ρητός.

ο ΜΓ ημ1 8 = - = . . . . . ΑΓ Λύση Έστω ΑΒΓ ΔΕ το κυρτό πεντάγωνο με

A = B = r = Δ = E = l08° και με τα μήκη των πλευρών ΑΒ, ΒΓ, ΓΔ, ΔΕ, ΕΑ ρητούς αριθμούς. Έστω ΑΒ � ΒΓ , ΑΒ > ΓΔ , ΑΒ > ΔΕ , ΑΒ > ΕΑ . Θεωρούμε το κανονικό πεντάγωνο ΑΒΓ1Δ1Ε1 (m-

θανόν Γ1 = Γ ).

Ει

Δι

Γι

Θεωρούμε Ε,Ε2 1 1 Γ,Δ, οπότε, αν Ε :ι:. Ε, τότε: i) 'Ε = Ε2 = 108° . ii) Οι πλευρές του ισοσκελούς τρ. Ε1ΕΕ2 είναι ρητοί

Α. Β αριθμοί αφού Ε1Ε2 = Ε1Ε = ΑΒ-ΑΕ και ΕΕ2 ρητός, αφού είναι διαφορά ρητών. Αυτό όμως είναι άτοπο από την

Λ παρατήρηση (προφανώς Ε Ει Ε2 = 1 8° ).

Άρα Ε= Ε1 :::::} Δ =Δ, και Γ = Γ, , οπότε το πεντάγωνο ΑΒΓ ΔΕ είναι κανονικό.

ΠΡΟΒΛΗΜΑ 3 Έστω a, b, c οι θετικοί πραγματικοί αριθ­

μοί τέτοιοι που a + b + c � abc . Αποδείξτε ότι:

a2 + b2 + c2 � abc.J3.

Λύση Έστω a 2 + b2 + c2 < J3abc . Τότε θα έχουμε:

3� a 2b2c2 :::; a 2 + b2 + c2 < J3abc , όπου η πρώτη α­νισότητα προκύπτει με εφαρμογή της ανισότητας

και συνεπώς: 4k3 = (2k2 + 3kλ+ λ2 )λ και

ι:: = (3λ2 + 2kλ-4k2 )k .

Απ' την πρώτη παίρνουμε λ/4 [αφού (k3 , λ) = 1 ] Άρα: λ = ±1, ±2, ±4 Απ' τη δεύτερη k = ±1 Συνεπώς: ρ = ±ι,±!,±! . 2 4 Εύκολα διαπιστώνεται ότι για τις τιμές αυτές του ρ η ( 1 ) είναι αδύνατη. (Λύνεται και Γεωμετρικά).

αριθμητικού - γεωμετρικού μέσου. Έτσι έ-.{ουμε 3�a2b2c2 < J3abc , από την οποία προκύπτει αμέ-σως ότι abc > 3J3 (1)

a2b2c2 (a + b + c)2 Επίσης έχουμε -3- :::; 3

, αφού abc :::; a + b + c και a,b,c > Ο . Επειδή (a + b+ c)2

< 2 b2 2 ---- _ a + + c 3

(προκύπτει από την

(a -b)2 + (b- c)2 + (c - a)2 � Ο ) και a2 + b2 + c2 < J3abc, έχουμε τελικά ότι abc < 3J3 , που αντίκειται στη σχέση ( 1 ).

Άρα a 2 + b2 + c2 � J3abc . 2°ς τρόπος (Σ. Λουρίδα): Γνωρίζουμε ότι για κάθε χ, y, w ε IR ισχύει :

χ2 + y2 +ω2 � χy + yω+ωχ, ( 1) από την οποία προκύπτει και η ανισότητα

(x + y+w )2 � 3(xy + yw + wx). (2) Έτσι έχουμε:

a2 + b2 + c2 � abc.J3 � (a2 + b2 + c2 )2 � 3a2b2c2 ή λόγω της (2), αρκεί 3(a2b2 + b2c2 +c2a2 ) � 3a2b2c2 ή λόγω της ( 1 ), αρκεί ab2c + a2bc + abc2 � a2b2c2

<=> abc (b + a + c) � a2b2c2 <=> a+b+ c � abc ,

η οποία αληθεύει από την υπόθεση.

ΠΡΟΒΛΗΜΑ 4 Ένας κύβος διαστάσεων 3 χ 3 χ 3 υποδιαφεί-

ται σε 27 ίσα μοναδιαία κυβικά κύτταρα. Ένα από αυτά τα κύτταρα είναι κενό και τα υπόλοιπα γεμίζο­νται με μοναδιαίους κύβους που μαρκάρονται με τους αριθμούς 1, 2, ... , 26 κατ' αυθαίρετο τρόπο. Μία αποδεκτή κίνηση είναι μία μετακίνηση ενός μονα­διαίου κύβου σ' ένα διπλανό κενό κύτταρο. Υπάρχει πεπερασμένη ακολουθία αποδεκτών κινήσεων μετά από την οποία ο μοναδιαίος κύβος μαρκαρισμένος με τον αριθμό κ και ο μοναδιαίος κύβος μαρκαρι­σμένος με τον αριθμό 27 - κ ανταλλάσσουν θέσεις για κάθε κ = 1,2, . . . ,13 ; [Δύο κύτταρα θεωρούνται

διπλανά, αν έχουν κοινή έδρα].

Λύση Χρωματίζουμε τους μοναδιαίους κύβους μαύρους

ή άσπρους, έτσι ώστε οι δutλανοί κύβοι να έχουν δια­φορετικά χρώματα. Μαρκάρουμε τα μοναδιαία κυβικά κύτταρα με τους αριθμούς 1 , 2, ... , 27 έτσι ώστε διαδο­χικοί αριθμοί να ανnστοιχούν σε διπλανά κύτταρα. Για παράδειγμα, χωρίζουμε τον κύβο σε τρεις ορόφους και ανnστοιχούμε στον τρίτο όροφο τους αριθμούς 1-9, στον δεύτερο όροφο τους αριθμούς 10-18 με το 10 α­κριβώς κάτω από το 9 και στον πρώτο όροφο τους α-

ΕΥΚΛΕΙΔΗΣ Β' λ.ε. τ.l/21

Page 24: Ευκλειδης Β 41

Μαθηματικοί Διαγωνισμοί - Μαθητικές Ολυμπιάδες

ριθμούς 19-27 με το 19 ακριβώς κάτω από το 18 (βλ. σχήμα 2).

Σχήμα 2 Ονομάζουμε το ζευγάρι των κύβων { m, η} ανε­

στραμμένο, αν ο κύβος με το μεγαλύτερο αριθμό είναι στο κυβικό κύτταρο με το μικρότερο αριθμό. Σημειώ­νουμε με Ι (κ) το συνολικό αριθμό των ανεστραμμέ­νων μη διατεταγμένων ζευγών των κύβων μετά από κ > Ο αποδεκτών κινήσεων. Ένας μετακινούμενος ιώ­βος τοποθετείται πάντα σε ένα κυβικό κύτταρο με δια­φορετικό χρώμα από το δικό του. Έτm κατά τη διάταξη των μοναδιαiων κύβων διέρχεται από έναν άρτιο αριθ­μό (mθανώς Ο) όJJ..mν μοναδιαiων κύβων. Το πέρασμα ενός κύβου WJ.iJ[,ει τον αριθμό Ι ( • ) κατά ένα, οπότε κάθε αποδεκτή κίνηση διατηρεί το άρτιο ή περιττό του αριθμού Ι ( •) .

Υποθέτουμε ότι έχουμε αναδιατάξει τους μονα­διαίους κύβους, όπως ζητείταt, σε λ κινήσεις. Οι 26 κύ­βοι σχηματίζουν 25 · 1 3 = 325 μη διατεταγμένα ζεύγη. Ένα ζευγάρι κύβων σε δύο κύτταρα είναι ανεστραμμέ­νο στην τελική διάταξη, αν, και μόνο αν, το ζευγάρι των κύβων στα δύο αυτά κύτταρα δεν είναι ανεστραμμένο κατά την αρχική διάταξη. Έτm θα έχουμε Ι (λ) = 325 - Ι (Ο) , που αντίκειται στο ότι οι Ι (λ) και Ι (Ο) είναι και οι δύο άρτιοι ή και οι δύο περιττοί.

Άρα είναι αδύνατο να αναδιατάξουμε τους κύβους όπως ζητείται.

2ος τρόπος Χρωματίζουμε τα (μοναδιαία) κυβικά κύτταρα

μαύρα ή άσπρα, εναλλάξ έτm ώστε γειτονικά κύτταρα να έχουν διαφορετικά χρώματα.

Σε κάθε μοναδιαίο κυβικό κύτταρο αντιστοιχού­με τον αριθμό του μοναδιαίου κύβου ο οποίος αρχικά τοποθετείται στο κύτταρο αυτό και αυτός ο αριθμός παραμένει ως αριθμός του κυττάρου. Υπάρχει και ένα άδειο κυβικό κύτταρο στο οποίο αντιστοιχούμε το σύμ-

. βολο *. Μια επιτρεπτή κίνηση είναι μiα αντιμετάθεση (1 2 . . . i . . . 26 *}

της μορφής: Pi = *

. 1 ::; i ::; 26 . 1 2 . . . . . . 26 ι

Η αντιμετάθεση Pi είναι στοιχείο του συνόλου S27 των μεταθέσεων 27 στοιχείων και είναι περιττή με­τάθεση. ('fo σύνολο s27 είναι ομάδα ως προς τη σύν­θεση μεταθέσεων).

Για να ισχύει το ζητούμενο θα πρέπει να βρούμε έναν πεπερασμένο αριθμό αντιμεταθέσεων Ρι , Ρ2 , ... , Ρ ι έτm ώστε να ισχύει: (1 2 . . . 25 26 * )

ΡιΡι-ι · · · Ρ2Ρι =

26 25 . . . 2 1 * = q ·

Επειδή η μετάθεση q έχει το σύμβολο * ως στα­θερό σημείο αυτή αναλύεται σε ένα πεπερασμένο αριθ­μό αντιμεταθέσεων ως εξής: q = (1, 26)(2, 25) · · · (13, 14)(*) . Επομένως η μετάθε­ση q είναι περιττή.

Κάθε μετάθεση της μορφής Pi WJ.iJ[,ει το χρώ­μα του κυβικού κυττάρου που γεμίζεται από τον κύβο *. Για να λάβουμε τη μετάθεση q θα πρέπει να χρηm­μοποιήσουμε άρτιο αριθμό μεταθέσεων της μορφής Pi , οπότε η μετάθεση q θα πρέπει να είναι άρτια, που είναι άτοπο.

r ΕΙΣιΙΤDΤΗ ΣΤΗ IED'Piιl TPιlf/JHMιiTQN Η θεωρία γραφημάτων είναι ένας κλάδος των

μαθηματικών του οποίου η «γένεση)) ανάγεται στο 1 736 όταν ο μεγάλος Ελβετός μαθηματικός L. Euler (1707-1782) δημοσίευσε μiα, από τις περίπου χίλιες εργασίες του, την "solutio problematis ad geometήam situs pertiηeηtis (η λύση προβλήματος που αναφέρεται στην γεωμετρiα της θέσης)". Μετά από 200 χρόνια ο Ούγγρος μαθηματικός D. Κφηig (1884-1944) δημοσίευσε το 1936 το πρώτο βιβλίο θεωρίας γραφημάτων. Έκτοτε η θεωρία γνώρισε μια μεγάλη ανάπτυξη η οποία έγινε ακόμα εντονότερη τα μεταπολεμικά χρόνια ίσως και εξαιτίας των εφαρμο-

του Α. Παπαϊωάννου

γών της θεωρίας στην πληροφορική, στους σχεδια­σμούς, στους κώδικες κ.ο.κ . .

Ένα γράφημα G αποτελείται από ένα μη κενό, συνήθως πεπερασμένο, σύνολο η σημείων ν(G) που ονομάζονται κορυφές του γραφήματος, από ένα σύ­νολο E(G), υποσύνολο του καρτεσιανού γινομένου ν ( G) χ ν ( G) , τα m στοιχεία του οποίου ονομάζο­νται πλευρές του γραφήματος G, και από έναν κανό­να που ορiζει αν δύο διαφορετικές κορυφές, έστω οι χ και y, ενώνονται ή όχι με την πλευρά xy (ή και yx).

Δύο κορυφές χ και y που ενώνονται με την πλευρά xy (ή και yx) λέγονται γειτονικές, ενώ η

ΕΥΚΛΕΙΔΗΣ Β' λ.ε. τ.l/22

Page 25: Ευκλειδης Β 41

Μαθηματικοί Διαγωνισμοί - Μαθητικές Ολυμπιάδες

πλευρά xy προσπίπτει στην κορυφή χ ή διέρχεται από την κορυφή χ. Οι κορυφές χ και y είναι τα άκρα της πλευράς xy. Οι πλευρές xy και xz που διέρχονται και οι δύο από την κορυφή χ ονομάζονται γειτονι­κές.

Από τον ορισμό μία πλευρά xy ενώνει ακριβώς δύο διαφορετικές κορυφές τις χ και y. Στην πράξη ό­μως εμφανίζονται περιπτώσεις όπου οι δύο κορυφές συμπίπτουν, οπότε μία τέτοια πλευρά 'λiγεται βρό­χος. Επίσης πάλι στην πράξη εμφανίζονται περιπτώ­σεις όπου οι διαφορετικές κορυφές χ και y ενώνονται με περισσότερες από μία πλευρές οι οποίες 'λiγονται πολλαπλές πλευρές. Κατά την εmκρατέστερη ορο­λογία ένα γράφημα δεν περιέχει βρόχους και ποJ..J..n.­π'λiς πλευρές, ενώ σ' ένα πολυγράφημα επιτρέπο­νται οι βρόχοι και οι πο'λλωrλές πλευρές. Το διά­γραμμα a του σχήματος 1 απεικονίζει ένα γράφημα G με V(G) = {1, 2, 3,4} , E(G) = {12,23,34,41, 24} ενώ το διάγραμμα: b ένα πολυγράφημα όπου οι κο­ρυφές 1 και 4 ενώνονται με 3 πολλαπλές πλευρές, και από έναν βρόχο στις κορυφές 2 και 3.

σ ι[2J' 4 3 � � � Σχήμα 1

Τονίζουμε ότι ένα γράφημα μας πληροφορεί μόνο εάν οι κορυφές χ και y ενώνονται ή όχι με πλευρά. Δεν έχουμε καμία πληραρορία για την από­σταση των δύο κορυφών, το είδος της σύνδεσής τους, την γωνία των πλευρών xy και xz και όJ.λα στοιχεία βασικά για την Γεωμετρία.

Ονομάζουμε δρόμο μία ακολουθία κορυφών και πλευρών της μορφής ν0 , eι , νι , ez , ν2 , . . . , νη-ι • eη , ν η , όπου η πλευρά ei ενώνει τις κορυφές νί-ι και νi ι � i � η . Το μήκος ενός δρόμου είναι το πλήθος των πλευρών του. Ένας δρόμος με όλες τις πλευρές του διαφορετικές (όχι όμως και όλες τις κο­ρυφές) 'λiγεται δρομίσκος. Αν επιπλέον όλες οι κο­ρυφές είναι διαφορετικές τότε έχομε το μονοπάτι ν ι , . . . , ν η που αποτελείται από τις η - 1 πλευρές ν ι ν 2 , ν2ν3 , .. . , νη-ινη δηλαδή έχει μήκος η - 1 και συμ­βολίζεται με Ρη .

Α ν η πρώτη κορυφή ν 0 και η τελευταία ν η σ' έναν δρόμο συμπίπτουν ο δρόμος 'λiγεται κλειστός· παρόμοια ορίζεται και ο κλειστός δρομίσκος. Όμως ένα κλειστό μονοπάτι 'λiγεται κύκλος. Ο κύκλος νι , ν 2 , .. . , ν η αποτελείται από τις η πλευρές ν ι ν z ,

ν2ν3 , . .. , νη_ινη , νηνι άρα έχει μήκος η και συμβο­λίζεται με cη ο

Το σχήμα 2 διευκρινίζει τις έννοιες αυτές.

Η ακολουθία ι , 2, 5, 2, 3, 4, 5, 6 είναι ένας δρό­μος μήκους 7 με άκρα τις κορυφές ι και 6. Η προ­σθήκη της πλευράς ι6 μας δίνει έναν κλειστό δρόμο μήκους 8. ο δρομίσκος ι , 2, 5, 3, 4, 5, 6 έχει μήκος 6 και άκρα τις κορυφές ι και 6. Η προσθήκη της πλευ­ράς 16 μας δίνει έναν κλειστό δρομίσκο μήκους 7. Το μονοπάτι ι, 2, 3, 4, 5, 6 μήκους 5 κλείνει αν προ­σθέσουμε την πλευρά ι6 σε κύκλο μήκους 6. Ο κύ­κλος ι , 2, 3, 4, 9, 10, ι ι , 7, 8, 5, 6, ι έχει την ιδιότητα ότι διέρχεται (προφανώς μία μόνο φορά) και από τις ι ι κορυφές του γραφήματος G. Ο κύκλος αυτός 'λi­γεται κύκλος-Ηamiltοη, το δε γράφημα G που έχει κύκλο-Ηami1tοη 'λiγεται γράφημα-Ηamiltοη.

2

1 1 10 Σχήμα 2

Ένα γράφημα 'λiγεται συνεκτικό αν οποιεσδή­ποτε δύο κορυφές του ενώνονται με κάποω μονοπά­τι. Ένα μη συνεκτικό γράφημα αποτελείται από δύο ή περισσότερες συνεκτικές συνιστώσες. Στο σχήμα 3a έχουμε το συνεκτικό γράφημα G με 4 κορυφές και 5 πλευρές ενώ στο σχήμα 3b το μη συνεκτικό γρά­φημα Η με 13 κορυφές 1 1 πλευρές και 3 συνεκτικές συνιστώσες.

a -

b - K D Y

Η Σχήμα 3

Ο βαθμός d(ν) της κορυφής ν είναι το πλήθος των πλευρών που διέρχονται από την κορυφή ν. Στο σχήμα 3a οι βαθμοί είναι: d( ν 2 ) = d( ν 4) = 3 και d( ν ι ) = d( ν 3 ) = 2 .

Παρατηρούμε ότι το άθροισμα των βαθμών των κορυφών κάθε γραφήματος ισούται με το διπλάσιο των πλευρών του γραφήματος πράγματι κάθε πλευ­ρά αφού διέρχεται από δύο κορυφές συνεισφέρει δύο μονάδες στο άθροισμα αυτό. Συμβολικά για το γρά­φημα G με η κορυφές και e πλευρές ισχύει η

Σd(νί ) = 2e . Ο αριθμός η (πλήθος κορυφών) ονο-i=ι

μάζεται τάξη του γραφήματος G και ο αριθμός e (πλήθος πλευρών) ονομάζεται μέγεθος του γραφή­ματος G. Το γράφημα G τάξης η και μεγέθους e συμβολίζεται και με G(η,e) .

ΕΥΚΛΕΙΔΗΣ Β' λ.ε. τ.l/23

Page 26: Ευκλειδης Β 41

Μαθηματικοί Διαγωνισμοί - Μαθητικές Ολυμπιάδες

Άμεση συνέπεια της σχέσης Σ, d ( νί ) = 2e , αν μών 4, 3, 3, 3, 2 όμως δεν είναι ισόμορφα.

i=l χωρίσουμε τις η κορυφές του G σε «άρτtε9> κορυφές (δηλαδή κορυφές άρτιου βαθμού) και σε «περιττές» κορυφές (κορυφές περιττού βαθμού) και θυμηθούμε ότι αν ένα άθροισμα περιττών αριθμών είναι άρτιο τότε θα έχει άρτιο πλήθος προσθετέων, είναι το λε­γόμενο «λήμμα των χεφαψιών»:

«Σε κάθε γράφημα το πλήθος των κορυφών με περιττό βαθμό είναι άρτιος αριθμό9> όπου το Ο θεω­ρείται άρτιος αριθμός.

Μια κορυφή ν με d( ν) = Ο ονομάζεται μεμο-

νωμένη κορυφή. Έστω το γράφημα G(ν,Ε) . Το γράφημα

G'(ν',Ε') με ν' ς ν και Ε' ς Ε ονομάζεται υπο­

γράφημα του G. Στην ειδική περίπτωση που ν' = ν το G' λέγεται παράγον υπογράφημα του G. Το υπο­γράφημα Η του G που έχει ν' c ν και περιέχει όλες τις πλευρές του G που διέρχονται από τις κορυφές του ν' ονομάζεται υπογράφημα του G παραγόμενο

από τις κορυφές του ν' . Στο σχήμα 4 έχουμε το G(6,8) το υπογράφημα Η1 (5,4) το παράγον υπο-γράφημα Η2 ( 6, 7) και το παραγόμενο υπογράφημα Η3 του G από τις κορυφές {3,4,5,6} .

.-----.2

6 1-----.1

5 "---_J4 G Η, •

Σχήμα 4 Η ακολουθία βαθμών ενός γραφήματος G προ­

κύπτει αν αναγράψουμε τους βαθμούς των κορυφών του G σε φθίνουσα (μη αύξουσα) σεφά. Η ακολου­θία των βαθμών του γραφήματος G του σχήματος 4 είναι 4, 3, 3, 2, 2, 2.

Δύο γραφήματα G(ν,Ε) και Η(ν',Ε') λέγο-νται ισόμορφα αν υπάρχει μία 1-1 αντιστοιχία φ των κορυφών τους η οποία να διατηρεί τις γειτνιάσεις και τις μη γειτνιάσεις.

Αναλυτικότερα αν a,bε ν(G) και abε E(G) τότε η πλευρά φ(a)φ(b)ε Ε'(Η) . Παρόμοια αν γ,δε V(G) και γδe E(G) τότε η φ(γ)φ(δ) δεν εί­ναι πλευρά του Η ήτοι φ(γ)φ(δ) e Ε'(Η) . Παρατη­ρούμε ότι αν δύο γραφήματα είναι ισόμορφα τότε έ­χουν ίδια τάξη, ίδω μέγεθος και ίδια ακολουθία βαθ­μών. Όμως το αντίστροφο δεν ισχύει όπως φαίνεται από τα δύο γραφήματα του σχήματος 5 που έχουν ί­δια τάξη 6, ίδω μέγεθος 9 και ίδια ακολουθία βαθ-

Σχήμα 5 Πράγματι οι δύο γείτονες τις κορυφής u ε G έ­

χουν βαθμούς 3 και 3. Στο Η οι δύο γείτονες της ν (μοναδική κορυφή βαθμού 2 στο Η άρα εικόνα της u αν υπήρχε η 1-1 αντιστοιχία φ) έχουν βαθμούς 3 και 4. Τα δύο γραφήματα G και Η έχουν διαφορετική δομή άρα δεν είναι ισόμορφα.

Ένα γράφημα με η κορυφές λέγεται πλήρες και

μβολ�r κ , (η ] η (η - 1) συ �εται με n αν εχει και τις 2 =

2

δυνατές πλευρές. Το κενό γράφημα Ε0 έχει η κορυφές και καμία πλευρά, δηλαδή και οι η κορυφές του είναι μεμονωμένες. Ένα γράφημα G(ν,Ε) λέγεται διαμε-ρισμένο ή διμερές αν το σύνολο των κορυφών του δια­μερiζεται σε δύο μη κενά υποσύνολα ν1 , ν2 ξένα με­ταξύ τους τέτοια ώστε κάθε πλευρά του G να ενώνει κάποια κορυφή του V1 με κάποια κορυφή του ν2 . Ένα διμερές γράφημα λοtπόν δεν περιέχει πλευρά που να έ­χει και τα δύο άκρα της στο ν1 ή στο ν2 . Το διμερές γράφημα G λέγεται πλήρες διμερές γράφημα και συμ­βολίζεται κm,n αν είναι διμερές με Ι νι Ι = m , ιν2 Ι = η και περιέχει όλες τις πλευρές με το ένα άκρο στο V1 και το ω.ΝJ στο ν2 . Προφανώς το Κ είναι ένα m,n ( m + η , m · η ) γράφημα. Στο σχήμα 6 έχουμε το Κ5 ,

το Κ3,3 και ένα μη πλήρες διμερές γράφημα με

Ι νι Ι = 4 , ιν2 Ι = 3 και s πλευρές.

Σχήμα 6 Ανάλογα μπορούμε να ορίσουμε τα τριμερή και

εν γένει τα q-μερή γραφήματα. Ένα δέντρο είναι ένα συνεκτικό γράφημα χωρίς

κύκλους. Ένα δάσος είναι ένα μη συνεκτικό γράφη­μα χωρίς κύκλους προφανώς οι συνεκτικές συνι­στώσες του δάσους είναι δέντρα. Στο σχήμα 7 έχου­με τρία δέντρα με 7 κορυφές και 6 πλευρές. Το πρώ­το είναι το Ρ7 (το μονοπάτι με 7 κορυφές), το δεότε-ρο είναι το Κ1,6 , το τρίτο μοιάζει να μην είναι διμε­ρές όμως είναι αν πάρουμε σαν ν1 = { a, b, c} και

ΕΥΚΛΕΙΔΗΣ Β' λ.ε. τ.l/24

Page 27: Ευκλειδης Β 41

Μαθηματικοί ΔιαΎωνισμοί - Μαθητικές Ολυμπιάδες

V2 = {1,2,3,4} .

Σχήμα 7 b 2 3 4

Για η = 7 έχουμε 1 γράφημα με Ο πλευρές (το

Ε, ) και ένα με ( � )= 21 ιrορυφές (το κ, ). Τα πρώ­

τα συνεκτικά γραφήματα με 7 κορυφές έχουν ό'λn 6 πλευρές και είναι ό'λn δέντρα. Τ ρίά από αυτά φαίνο­νται στο σχήμα 7.

Θα δείξουμε τώρα ότι κάθε δέντρο με η κορυφές έχει η - 1 πλευρές. Η απόδειξη γίνεται επαγωγικά. Προτάσσουμε δύο λήμματα.

Λήμμα 1. Αν στο γράφημα G ο βαθμός κάθε κορυφής είναι τουλάχιστον 2 τότε το G περιέχει κά­ποιο κύκλο.

Απόδειξη: Έστω ν κάποια κορυφή του G. Κα­τασκευάζουμε μία ακολουθία κορυφών ν, ν1 , ν2 , ••• Επαγωγικά εκλέyοντας την ν 1 γειτονική της ν και για i ;::: 1 την ν;+ι γειτονική της ν; και διαφορετική από την ν;_1 • Η υπόθεση μας εξασφαλίζει την ύπαρ­ξη της ν;+ι . Δεδομένου ότι το G έχει πεπερασμένο αριθμό κορυφών θα εκλέξουμε σε κάποια φάση κά­ποια κορυφή, η οποία έχει ήδη εκλεγεί. Έστω ν" η πρώτη τέτοια κορυφή. Τότε το τμήμα της ακολουθί­ας κορυφών που βρίσκεται μεταξύ των δύο διαδοχι­κών εκλογών της ν" αποτελεί τον ζητούμενο κύκλο.

Λήμμα 2: Ένα δέντρο Τ έχει τουλάχιστον 1 κο­ρυφή βαθμού 1 .

Απόδειξη: Α ν όλες οι κορυφές του Τ είχαν βαθμό ;::: 2 από το προηγούμενο Λήμμα θα υπήρχε κάποιος κύκλος C και το γράφημα δεν θα ήταν δέ­ντρο.

Θεώρημα 3: Ένα δέντρο με η κορυφές έχει η - 1 πλευρές.

Απόδειξη: Κάνουμε επαγωγή στο πλήθος των κορυφών η. Για η = 2 το μοναδικό δέντρο είναι το • • (πρόκειται για το γράφημα Κ2 ) που έχει 1

πλευρά. Για η = 3 το μοναδικό δέντρο είναι το • • • (πρόκειται για το μονοπάτι Ρ3 ) που έχει 2 πλευρές. Άρα το θεώρημα ισχύει για η = 2, 3 . Δεχόμαστε ότι ισχύει για όλες τις τιμές τις μικρότε­ρες του η και θεωρούμε ένα δέντρο Τ με η κορυφές. Θα δείξουμε ότι τούτο έχει η - 1 πλευρές. Από το Λήμμα 2 το Τ έχει τουλάχιστον 1 κορυφή βαθμού 1 , έστω την ν . Αφαφούμε την κορυφή αυτή ν και την μοναδική πλευρά vw που διέρχεται από την ν οπότε έχουμε ένα δέντρο Τ' με η - 1 κορυφές. Από την ε-

παγωγική υπόθεση e(τ') = η - 2 οπότε το Τ αφού έ­χει τις πλευρές του Τ' και την vw που αφαφέθηκε θα έχει η - 2 + 1 = η - 1 πλευρές.

Πόρισμα 4: Ένα δέντρο Τ με η ;::: 2 κορυφές έ­χει τουλάχιστον 2 κορυφές βαθμού 1 .

Απόδειξη: Έστω d, � d2 � • • • � dη η ακολουθία βαθμών του Τ. Αν το Τ έχει ακριβώς μία κορυφή βαθμού 1 (την κορυφή 1 ) τότε οι υπόλοιπες η - 1 κορυφές του θα έχουν βαθμό ;::: 2 και ισχύει d2 + d3 + · · · + dη ;::: 2 {η - 1) .

η Η σχέση Σ d (ν; ) = 2e γίνεται λοιπόν

i=l η

2(η - 0 = 1 + Σd(ν; } ;::: 1 + 2(η -Ο ήτοι i=2

2η - 2 ;::: 2η - 1 άτοπο. Ένα γράφημα έχει ονομασία όταν έχουμε αντι­

στοιχίσει στις η κορυφές του ονόματα όπως ν 1 , ν 2 , ... , ν η . Ένα από τα πρώτα αποτελέσματα της θεωρί­ας γραφημάτων απεδείχθη (η απόδειξη μάλιστα ήταν ελλιπής) από τον Α. Cayley (1 821- 1895) στην προ­σπάθειά του να απαριθμήσει τους υδρογονάνθρακες. Το αποτέλεσμα αυτό είναι γνωστό σαν τύπος του Cayley. Η απόδειξη που θα δώσουμε οφείλεται στον Η. Prόfer (1896-1934) μαθητή του I. Schur.

Τύπος του Cayley: Το πλήθος των δέντρων με ονομασία με η κορυφές είναι η η-2

• Απόδειξη (κώδικας του Prόfer) Θα αντιστοιχήσουμε σε κάθε δέντρο με ονομα­

σία με η κορυφές μία διατεταγμένη (η - 2) -άδα από

τους αριθμούς {1,2, . . . , η} . Εφόσον το πλήθος των

(η - 2) -άδων είναι ηη-2 ο τύπος απεδείχθη. Θεωρούμε λοιπόν ένα δέντρο με ονομασία με η

κορυφές και η - 1 πλευρές και την κορυφή 1 . Α ν αυ­τή έχει βαθμό 1 την αφαφούμε μαζί με την πλευρά που προσπίπτει σ' αυτήν και σαν πρώτο στοιχείο του κώδικα τοποθετούμε την ονομασία της γειτονικής κορυφής της 1 . Συνεχίζουμε παρόμοια με την κορυ­φή 2 κ.ο.κ. έως ότου παραμείνει μία μόνο πλευρά (και οι δύο κορυφές που είναι τα άκρα της). Ο κώδι­κας θα έχει τότε η -2 στοιχεία και ο αλγόριθμος τε­λειώνει.

Να τονίσουμε όμως ότι αν κατά την διάρκεια του αλγόριθμου κάποια κορυφή που είχε παραλει­φθεί διότι αρχικά είχε βαθμό μεγαλύτερο της μονά­δας, αποκτήσει μετά τις αφαφέσεις των πλευρών βαθμό ίσο με τη μονάδα, τότε κατά την εφαρμογή του αμέσως επόμενου αλγοριθμικού βήματος αυτή θα προταχθεί.

Παρατηρούμε ότι στον κώδικα εμφανίζονται μόνο οι κορυφές που έχουν d( ν) > 1 και μάλιστα η

ΕΥΚΛΕΙΔΗΣ Β' λ.ε. τ.l/25

Page 28: Ευκλειδης Β 41

Μαθηματικοί Διαγωνισμοί - Μαθητικές Ολυμπιάδες

καθεμία τους εμφανίζεται d - 1 φορές. του δέντρου διαγράφουμε την 3 και ο νέος κώδικας Παράδειγμα 4

5 6 1 7

10

1 ο βήμα: Φεύγει η κορυφή 1 η πλευρά 19 και ο κώδικας αρχiζει με 9. 2° βήμα: Η κορυφή 2 με βαθμό 2 > 1 παραλεί­πεται και εκλέγουμε την 3. Φεύγει η 3, η 32 και ο κώδικας γίνεται 92. 3° βήμα: Η 2 πάλι παραλείπεται διότι έχει βαθ­μό >1 και εκλέγεται η 4. Φεύγει η 4, η 42 και ο κώ­δικας γίνεται 922. 4° βήμα: Δεν εκλέyεται η 5 που έχει βαθμό 1 διότι προηγείται η 2 που απέκτησε τώρα βαθμό 1 . Φεύγει η 2, η 23 και ο κώδικας γίνεται 9228. 5° βήμα: Φεύγει η 5, η 58 και ο κώδικας γίνεται 92288. 6° βήμα: Φεύγει η 6, η 68 και ο κώδικας γίνεται 922888. 7' βήμα: Φεύγει η 7, η 79 και ο κώδικας γίνεται 9228889. 8° βήμα: Στο τελευταίο βήμα φεύγει η 8, η 89 και ο τελικός κώδικας είναι 92288899. Παραμένει η πλευρά 910. Θα περιγράψουμε τώρα την αντίστροφη διαδι­κασία. Θα εργαστούμε με ένα παράδειγμα WJ.i:ι. η γενική διαδικασία ακολουθεί την διαδικασία του πα­ραδείγματος. Αν δοθεί ένας κώδικας με ή στοιχεία να κατα­σκευαστεί το μοναδικό δέντρο που αντιστοιχεί σ' αυτόν. Πράγματι έστω ένας κώδικας με η στοιχεία ό­που προφανώς όλοι οι αριθμοί είναι � η . Σ' αυτόν αντιστοιχεί ένα δέντρο με ονομασία με η + 2 κορυ­φές και η + 1 πλευρές. Θεωρούμε τον κώδικα 4671 8825. Θα μας δώσει ένα δέντρο με 1 Ο κορυφές και 9 πλευρές. Η κορυφή με το όνομα 4 θα έχει βαθμό 2 διότι απαντά 1 φορά στον κώδικα. Παρόμοια οι κορυφές 6, 7, 1 , 2, 5 θα έ-χουν βαθμό 2, η κορυφή 8 θα έχει βαθμό 3 αφού α-παντάται δύο φορές στον κώδικα. Τέλος οι κορυφές 3, 9, 10 που δεν απαντώνται στον κώδικα θα έχουν βαθμό 1 στο δέντρο. 1 ο βήμα: Η πρώτη κορυφή που δεν εμφανίζεται στον κώδικα (n3) ενώνεται με πλευρά με την πρώτη κορυφή του κώδικα (η4). Δημιουργείται η πλευρά 34

είναι ο 6718825. 2° βήμα: Η πρώτη κορυφή που δεν εμφανίζεται στον κώδικα και δεν είναι διαγραμμένη (η4) ενώνε­ται με την πρώτη κορυφή του νέου κώδικα (η6). Δη­μωυργείται η πλευρά 46 διαγράφεται η 4 και ο νέος κώδικας είναι ο 718825. Το δέντρο έχει τις πλευρές 34 και 46. 3° βήμα: Η πρώτη κορυφή που δεν εμφανίζεται στον κώδικα και δεν έχει διαγραφεί (η6) ενώνεται με την 7. Δημιουργείται η πλευρά 67, διαγράφεται η 6 και ο νέος κώδικας είναι ο 18825. 4° βήμα: Η πρώτη μη εμφανιζόμενη και μη δια­γραμμένη κορυφή (η7) ενώνεται με την 1 . Δημωυρ­γείται η πλευρά 71 , διαγράφεται η 7, ο νέος κώδικας είναι 8825. 5° βήμα: Η κορυφή 1 ενώνεται με την 8 και δη­μιουργείται η 18. Διαγράφεται η κορυφή 1. Ο νέος κώδικας είναι 825. 6° βήμα: Η 9 ενώνεται με την 8 και δημιουργεί­ται η 89. Διαγράφεται η κορυφή 9. Ο νέος κώδικας είναι 25. 7° βήμα: Η 8 ενώνεται με την 2 και δημιουργεί­ται η 28. Διαγράφεται η κορυφή 8. Ο νέος κώδικας είναι 5. 8° βήμα: Η κορυφή 2 ενώνεται με την 5. Δια­γράφεται η 2. Υπάρχει ακόμα η πλευρά 51 Ο που σχηματίζεται από τις δύο κορυφές που δεν έχουν διαγραφεί. Το δέντρο φαίνεται στο σχήμα.

8 2 5 10 • • • • • Ι • • • 3 4 6 7

9

Το Άρθρο που αναφέρεται στους μετα­σχηματισμούς

ΟΜΟΙΟΘΕΣΙΑ - ΑΝΤJΣΤΡΟΦΗ

του Σ. Λουρίδα θα δημοσιευθεί στο επό-μενο τεύχος

�THII Ε,Μ.Ε.

ΕΥΚΛΕΙΔΗΣ Β' λ.ε. τ.l/26

Page 29: Ευκλειδης Β 41

ΚΥΚΛΟΦΟΡΗΣΑΝ από την Επιτροπή Ερευνών της Ακαδημίας Αθηνών

τα βιβλία του Ακαδημαϊκού

Νικολάου Κ. Αρτεμιάδη 1 . ΙΣΤΟΡΙΑ ΤΩΝ ΜΑΘΗΜΑΤΙΚΩΝ

(από της σκοπιάς του Μαθηματικού) [Σελίδες 764, Δρχ. 8500]

2. ΛΟΓΟΙ ΑΠΟ ΤΟΥ ΒΗΜΑΤΟΣ ΤΗΣ ΑΚΑΔΗΜΙΑΣ ΑΘΗΝΩΝ

(και άλλες μονογραφίες) [Σελίδες 598, Δρχ. 7500]

ΑΚΑΔΗΝΙΑ ΑθΗΝQΝ ED ITPOD H Ε Ρ Ε Υ Ν ι;) Ν

ΔΗΜΟΣΙΕΥUλΤΑ: λl'lθΜΟΣ 6

Ιστορία των Μαθηματικών (λπό της � - ..,....τικού)

ΝIΚΟΛΑΟΣ Κ. ΑΡΊΕΜΙΑΔΗΣ της Αχαδτιμιας ΑθιιΥCίι,.

ΑΘΗΝΑ 2000

Το σύγγραμμα απευθύνεται σε όσους ασχολούνται με τα μαθηματικά, κυρίως όμως στους μαθηματικούς που βρίσκονται στην αρχή της επιστημονικής τους σταδιοδρομίας. Στα 40 κεφάλαια του έρ'yου γίνεται μία σύντομη αναδρομή στην ιστορία των μαθηματικών περίπου από το 500 π.χ. Ελπίζεται ότι οι νέοι μαθηματικοί θα βοηθηθούν στο να προσανατολισθούν προς τις κατευθύνσεις εκείνες που ανταποκρίνονται στην ατομική τους ερευνητική ιδιοσυγκρασία και στα δικά τους ενδιαφέροντα.

ΑΚΑΔΗΜΙΑ ΑθΗΝΩΝ Ε Π Ι τ Ρ Ο Π Η Ε Ρ Ε Υ ΝΩΝ

ΔΗΜΟΣΙΕΥΜΑΤΑ: ΑΡIΘΜΟΣ 7

ΛΟΓΟΙ ΑΠΟ ΤΟΥ ΒΗΜΑΤΟΣ ΤΗΣ ΑΚΑΔΗΜΙΑΣ ΑθΗΝΩΝ (χαι άλλει; επιστημονιχέι; μονοyοαφιει;)

ΝΙΚΟΜΟΣ Κ. ΑΡ'fΕΜΙΑΔΗΣ τ�ς Αχαδ�μ.ιας Αθ�νών

ΑΘΗΝΑ 2001

Το βιβλίο περιλαμβάνει λόγους τους οποίους ο · συγγραφέας εξεφώνησε από του βήματος της Ακαδημίας Αθηνών κατά την τελευταία δεκαπενταετία. Επιπλέον περιλαμβάνει μονογραφίες μαθηματικού περιεχομένου καθώς και θέματα παιδείας και εκπαίδευσης. Τα περιεχόμενα του τόμου διακρίνονται στις ακόλουθες ενότητες: Μαθηματικά και καλλιτεχνία - Ήθος, Επιστήμη - Εφαρμογές των Μαθηματικών (Νέες Ανακαλύψεις) - Ιστορία -Διδασκαλία των Μαθηματικών (Εκπαιδευτικά Συστήματα). Το βιβλίο λόγω του περιεχομένου του θεωρείται συνοδεύων τόμος της Ιστορίας των Μαθηματικών (1 ).

ΤΑ ΒΙΒΛΙΑ ΔΙΑΤΙΘΕΝΤΑΙ ΣΕ ΟΛΑ ΤΑ ΒΙΒΛΙΟΠΩΛΕΙΑ ΚΕΝΤΡΙΚΗ ΔΙΆΘΕΣΗ: ΣΟΛΩΝΟΣ 84, ΑΘΗΝΑ, ΤΗΛ. 3603028

Page 30: Ευκλειδης Β 41

l•r

Σ το τεύχος 40 - Απρίλιος, Μάιος, Ιούνιος 200 Ι του περιοδικού είχε τεθεί το θέμα να δειχτεί με στοιχειώδη τρόπο ότι η συνάρτηση με τύπο g (x) = συν! δεν έχει όριο καθώς ο χ προσεγγίζει το χ

Ο. Απ' την ομάδα μαθηματικών των Εκπαιδευτηρίων «Γείτονα» που συντονίζει ο συνάδελφος Πολύκαρ­πος Λόης πήραμε την παρακάτω εργασία. ΠΡΟΤΑΣΗ Ι

Ν α αποδειχθεί ότι δεν υπάρχουν στο σύνολο JR υ{-οο, +οο} τα: Iim ημχ .

χ-+±-

Απόδειξη: Αποδεικνύουμε πρώτα ότι δεν υπάρχει στο

σύνολο JR υ{-οο, +=} το lim ημχ . χ-Η<»

Εργαζόμαστε με τη μ'θοδο της επαγωγής σε άτοπο. Έστω ότι υπάρχει στο JR υ { -οο, +=} το lim ημχ .

χ--++οο Δεδομένου ότι η συνάρτηση ημχ Ι JR είναι φρ«Ύμένη, θα είναι lim ημχ * ±= .

χ--++οο

Συνεπώς (από υπόθεση) υπάρχει πρ«Ύματι­κός αριθμός α έτσι, ώστε:

lim ημχ = α . (1) Χ--++οο

Από την (1) και το θεώρημα του ορίου της σύνθεσης των συναρτήσεων προκύπτει: lim ημ(χ -π) = α . (2)

χ--++οο

Εφόσον εκ ταυτότητος στο JR αληθεύει ότι ημχ = -ημ(χ -π) , από τις (1) και (2) λαμβάνου-με:

lim ημχ = Ο . (3) χ--++οο

Από την (3) και το θεώρημα του ορίου της σύνθεσης των συναρτήσεων προκύπτει:

lim ημ(χ -�)= ο . χ--++οο 2

(4)

Εφόσον εκ ταυτότητος στο JR αληθεύει ότι συνχ = -η μ( χ -; ). από την ( 4) λαμβάνουμε:

lim συνχ = Ο . (5) χ--++οο

Από τις (3) και (5) καταλήΎουμε σε άτοπο ε­φόσον: Ι = lim (ημ2χ +συν2χ)= lim ημ2χ + lim συν2χ =0 .

χ ----Ηοο χ -+ι-οο χ -+ι-οο

Τέλος, εφόσον ημχ = -ημ(-χ) , άμεσα απο-δεικνύεται ότι:

Δεν υπάρχει στο σύνολο JR υ { -οο, +=} ούτε το: lim ημχ .

χ-+�

Η όλη εργασία βασίζεται ουσιαστικά στο θεώ­ρημα:

lim g (x) = Υ ο } χ--+χο lim f (g (χ))= f . lim f (χ) = f και χ--+χο

x--+yo

g( χ) * y 0 , για όλους τους χ που είναι «κοντά» στο

ΠΡΟΤΑΣΗ ΙΙ Ν α αποδειχθεί ότι δεν υπάρχουν στο σύνολο

JR υ{-οο, +οο} τα: lim ημ(.!.) . χ--+0± χ

Απόδειξη: Προκύπτει αμέσως από την ΠΡΟΤ ΑΣΗ Ι,

διότι: Αν υπήρχε στο JR υ{-οο, +οο} ένα από τα

lim ημ(.!.)= α χ--+0± χ

από το θεώρημα του ορίου της σύνθεσης των συ­ναρτήσεων θα ήταν:

α = lim ημ(.!.)= lim ημt : άτοπο χ --+0± χ t --+±οο

ΕΥΚΛΕΙΔΗΣ Β' λ.ε. τ.l/28

Page 31: Ευκλειδης Β 41

Η στήλη της Αλληλογραφίας

ΠΡΟΤΑΣΗ1 111

Να δειχθεί ότι: δεν υπάρχει στο IR το όριο κο­ντά στο Ο της συνάρτησης g με g(x) = συν.!. χ

Απόδειξη: Προκύπτει άμεσα απ' την πρόταση I. Έστω

lim συν_!_ = .e, .e ε IR lim συν_!_ = .e. χ�ο· χ χ�ο· χ

Οπότε lim συw = .e.

Και συνεπώς: lim συν(u - π )= ι Οπότε: u�- 2

Iim ημu = .e. άτοπο. u�-

Προφανώς η g(x) ως φραγμένη Δε μπορεί να aπειρίζεται ούτε θετικά ούτε αρνητικά.

ΣΧΟΛΙΑ: • Για τις (σε διάστημα ορισμένες) παραγώγους

συναρτήσεων ισχύει ένα θεώρημα ενδιάμεσων τιμών (θεώρημα Darboux), μέσω του οποίου εξασφαλίζεται ότι: Η aσυνέχεια που μπορεί να παρουσιάζει σε σημείο χ0 ενός διαστήματος μια παράγωγος συνάρτηση είναι ουσιώδης, με την έννοια ότι δεν υπάρχουν στο IR u { -οο, +οο} τα πλευρι-κά όρια της παραγώγου συνάρτησης κοντά στο χ0 •

• Το παρακάτω προτεινόμενο θέμα είναι ένα εν­δεικτικό παράδειγμα (τουλάχιστον) «περίερ­γης» παραγώγου συνάρτησης: Ν α αποδειχθεί ότι η συνάρτηση

είναι:

f (x) = {O, αν χε Q χ2 , αν χε ΙR \ Q

(ί) Παντού ασυνεχής στο IR • . (ii) Παραγωγίσιμη στο Ο.

Από τον συνάδελφο Κώστα Γκολφινόπουλο πήραμε το παρακάτω γράμμα:

Θέλω αρχικά να εκφράσω την ικανοποίησή μου, αφού στο τεύχος 40 του περιοδικού «ΕΥΚΛΕΙΔΗΣ Β'>> - ΑΠΡΙΛΙΟΣ ΜΑΙΟΣ ΙΟΥΝΙΟΣ 2001 - υπήρξαν κάποιες απαντήσεις στα ερωτήματα που έθεσα προς το περιοδικό σας

1 Προσθήκη απ' τη Σ.Ε.

σχετικά με το 1 ο θέμα (Β 1 -α) της Γ' Λυκείου στα Μαθηματικά Θετ. Κατεύθυνσης, κατά τις εξετά­σεις του 2000. Παράλληλα θα ήθελα να κάνω και μερικές παρατηρήσεις γιατί θεωρώ ότι ουσιαστικά δεν υπήρξε απάντηση.

1. Ο «ΕΥΚΛΕΙΔΗΣ» απάντησε: «Στο σχ. βι­βλίο, που διδάσκεται τώρα, δεν υπάρχει ούτε υπαι­νιγμός πάνω στο ζήτημα αυτό και συνεπώς δεν έ­πρεπε να τεθεί στις εξετάσεφ>.

Το ότι στο σχ. βιβλίο δεν υπάρχει ούτε υπαι­νιγμός δεν είναι επαρκής λόγος για να μην τεθεί έ­να θέμα στις εξετάσεις. Στην προκειμένη περί­πτωση υπάρχει επαρκής λόγος, διότι εκτός από το γεγονός ότι δεν αναφέρεται πουθενά στο βιβλίο, ταυτόχρονα δε διαφαίνεται και κάποιος τρόπος να το αντιμετωπίσει ένας τελειόφοιτος του Λυκείου με τις γνώσεις που επίσημα μπορεί να κατέχει, έτσι ώστε η απάντηση στην ερώτηση ΣΩΣΤΟ-ΛΑΘΟΣ · να μην δοθεί διαισθητικά ή ολότελα τυχαία.

Παρόλα αυτά η παραδοχή εκ μέρους του «ΕΥΚΛΕΙΔΗ» ότι « . . . δεν έπρεπε να τεθεί στις ε­ξετάσεις» είναι κατά ένα μέρος, το σημαντικότερο ίσως, ικανοποιητική.

2. Ο «ΕΥΚΛΕΙΔΗΣ» απάντησε: «Το παρά­δειγμα που αναφέρεται στο υστερόγραφο 2, μπορεί να αποδειχθεί απλά με τη βοήθεια του ορισμού του ορίου . . . (και ακολουθεί η απόδειξη - 20 σειρές)».

Το «απλά» δεν θα το σχολιάσω διότι δεν κα­ταλαβαίνω το νόημά του. Ο ορισμός του ορίου συνάρτησης είναι ένα θεωρητικό θέμα για το ο­ποίο το Υπουργείο Παιδείας έχει δώσει σαφή οδη:.. γία προς τους καθηγητές να μην διδάσκεται στους μαθητές. Ο ορισμός του ορίου στο σχ. βι­βλίο περιλαμβάνεται φέροντας το γνωστό αστερί­σκο (*), δηλαδή εκτός διδακτέας ύλης.

3. «Είναι σημαντικό να βρεθεί παράδειγμα» - και σε αυτό συμφωνούμε απόλυτα - το οποίο ό­μως να δίνει απάντηση όχι μόνο στους καθηγητές, αλλά και στους μαθητές. Αυτό ήταν και είναι το ζητούμενο.

Η Σ.Ε. απαντά: Μια απάντηση στο ζήτημα που θέτει ο συνά­

δελφος υπάρχει στην ίδια στήλη και δίνεται από την ομάδα μαθηματικών των Εκπαιδευτηρίων «Γείτονα» που συντονίζει ο συνάδελφος Πολύ­καρπος Λόης.

ΕΥΚΛΕΙΔΗΣ Β' λ.ε. τ.l/29

Page 32: Ευκλειδης Β 41

f-1 σπ1Λn τοv μa Bntι1 Από τον μαθητή Δημήτριο Πιλάλα, 1 ου Ενιαί­

ου Λυκείου Γιαννιτσών λάβαμε την επιστολή: Αγαπητέ Ευκλείδη

Καταρχήν, θα ήθελα να ευχηθώ σε όλους τους συνεργάτες και τους αναγνώστες σου καλή αρχή. Αποφεύγω να περιττολογήσω με επαίνους για την προσφορά σου σε όλα τα επίπεδα η οποία είναι γνωστή. Θα ήθελα να αναφερθώ σε ένα ζήτημα που αφορά την δικτυακή παρουσία της Μαθηματι­κής Εταιρείας.

Στην ιστοσελίδα της Μαθηματικής Εταιρείας, η οποία είναι εξαιρετικά αξιόλογη, εμφανίζονται τρεις βάσεις ασκήσεων (προτεινόμενα θέματα Β' και Γ Λυκείου, θέματα των διαγωνισμών της Ε.Μ.Ε. και οι ασκήσεις του μήνα). Ομοίως, στη δι­εύθυνση www.telemath.gr υπάρχουν μεγάλες βά­σεις ασκήσεων με προτεινόμενα θέματα για δια­γωνισμούς της Ε.Μ.Ε., θέματα ρωσικών διαγωνι­σμών και ασκήσεις γενικού ενδιαφέροντος. Ωστό­σος, απ' όσο γνωρίζω πουθενά στο δίκτυο δεν υ­πάρχει κάποια βάση ασκήσεων (ελληνική ή ξένη), όπου να συλλέγονται ασκήσεις που να αφορούν κάθε μαθητικό πεδίο. Τις ασκήσεις θα μπορούσαν να στέλνουν αναγνώστες με την μορφή e-mail ή με το ταχυδρομείο, στη συνέχεια μαθηματικοί θα α­ναλάμβαναν τον έλεγχο και την κατηγοριοποίηση των ασκήσεων κατά πεδίο και βαθμό δύσκολίας. Με την ολοκλήρωση της διαδικασίας οι ασκήσεις τοποθετούνται στην ιστοσελίδα. Τα οφέλη μιας τέ­τοιες προσπάθειας είναι τεράστια για όλους και κυρίως για μαθητές και φοιτητές.

Κατανοώ ότι οι δυσκολίες ενός τέτοωυ εγχει­ρήματος είναι πολύ μεγάλες, αφού υπάρχει ανάγκη διεύρυνσης των υπαρχουσών εγκαταστάσεων και συνεχούς επίβλεψης. Πιστεύω αυτό το εγχείρημα μπορεί να πραγματοποιηθεί στα πλαίσια της Ε.Μ.Ε., αφού δε λείπει το μεράκι και τα απαιτού­μενα κονδύλια δεν είναι απαγορευτικά. Εξάλλου, η Μαθηματική Εταιρεία φαίνεται να είναι η πλέον αρμόδια για να το επιχειρήσει.

Ευχαριστώ για τον πολύτιμο χρόνο που μου διαθέσατε. Προτείνω την παρακάτω άσκηση

Ν α δείξετε ότι: lχ- (α + β)2 1+ 1χ- 4αβl � (α-β)2 , για κάθε xε ffi.

(α, β δοσμένοι πραγματικοί αριθμοί).

Από τον μαθητή Αθανάσιο Ντόρτο, 1 ου Ε-

νιαίου Λυκείου Άμφισσας λάβαμε τις ασκήσεις:

ΑΣΚΗΣΗ l Αν Sν είναι το άθροισμα των ν πρώτων ό-

ρων γεωμετρικής προόδου αι ' α2 ' α3 ' ... , αν ' με θετικό λόγο διαφορετικό από το 1, Sν το ά­θροισμα των αντιστρφων αυτών των όρων και Γν το γινόμενό τους, τότε: Γν = �s: (sν )-ν •

α1 (λ- 1)λ• 1

(α�λν-Ι ) ή s: · (sνΓ = (α�λ� )Ζν ( 1 ). Ό Γ λ λ2 �ν-1 , μως: ν = αι · αι · αι · · · · · αι /Ι, η Γ _ ν � 1+2+3+· · +(ν-1) , Γ _ νλν21[Ι+(ν-Ι)] , ν - α! . /1, η ν - α! . η

( ""f )Ζν � / ν (- )-ν Η (2) γίνεται: Γν = α1λ -νSν Sν .

ΑΣΚΗΣΗ 2 Δίνεται κύκλος (O,R) και οι ίσες χορδές του

ΑΒ = ΒΓ = Γ Δ . Να δειχθεί ότι: ΑΓ2 - ΑΒ2 = ΑΒ · ΑΔ .

Λύση Έστω Μ το μέσο της ΒΓ. Τότε:

Ar -AW =2BΓ ·EM} , Ar -AW =2AB·EM (1). ΑΒ=ΒΓ

η

ΕΥΚΛΕΙΔΗΣ Β' λ.ε. τ.l/30

Page 33: Ευκλειδης Β 41

Η Στήλη του Μαθητή

Σχ. 1 Σχ.2 Όμως ΑΒ = ΓΔ ή AB = U ή Α1 = fΊ , άρα

ΒΓ Ι Ι ΑΔ . ΟΜ .1 ΒΓ (ως απόστημα) και ΒΓ 1 1 ΑΔ άρα

ΟΙ .l ΑΔ . Επομένως ΟΙ απόστημα της χορδής ΑΔ ή Ι

μέσο της ΑΔ. Άρα ΕΜΙΑορθογώνιο. Οπότε ΕΜ = ΑΙ = .!_ ΑΔ (2).

2 ω �ΑΓ2 -ΑΒ2 = ΑΒ ·ΑΔ . (2)

Άλλη λύση από την Σ.Ε. Στο τρίγωνο ΑΓΔ από το 2° θεώρημα των

διαμέσων ισχύει: ΑΓ2 - Γ Δ2 = 2 · ΑΔ · ΙΚ , όπου Ι το μέσο της

ΑΔ και ΓΚ .l ΑΔ . Άρα ΑΓ2 -ΑΒ2 = ΑΔ · (2 · ΙΚ) . Αρκεί να δείξουμε ότι 2 · ΙΚ = ΒΓ . Φέρω ΒΛ .1 ΑΔ . Τότε ΑΛ = ΚΔ (από τα ίσα

τρίγωνα ΑΒΛ και ΓΚΔ), άρα ΑΙ = ΙΚ (αφού Ι μέ­σο του ΑΔ), άρα 2 · ΙΚ = ΛΚ = ΒΓ = ΑΒ .

Ο Ευκλείδης προτείνει: Για τους πραγματικούς αριθμούς α, β, γ δεχό-

μαστε ότι: lαl < jβ + γj , jβ j < jγ + αj , jγj < Ι α+ βj . Ν α δείξετε ότι:

i) αβγ :;t: Ο , ii) είναι όλοι θετικοί ή όλοι αρνητικοί, iii) (α+β+γ)( --α+β+γ)(α-β+γ)(α+β-γ) >0 , iν) για οποιουσδήποτε πραγματικούς αριθμούς χ,

y, z ισχύει: α2 (χ -y)(x -z)+β2 (y-x)(y-z) +y (z-x)( z-y) ;?:0 .

Blfliaι mM); �ε 1 ) Μαθηματικά Γ Λυκείου «Γενικής Παιδείαρ>

Γ. Τ σικαλουδάκη. 2) Στατιστική Γ Λυκείου Γ. Τσικαλουδάκη. 3) Flatland «Η Επιπεδοχώρα» Edwin Α. Abbott -

ΑΙΩΡΑ. 4) «ΠΟΙΗΜΑΤ Α>> Μανόλης Μπερβανάκης. 5) «ΜΕΝΤΟΡ ΑΣ» Περιοδικό Επιστημονικών και

Εκπαιδευτικών Ερευνών Παιδαγωγικό Ινστι­τούτο.

6) Μαθηματικά - Άλγεβρα Γ Ενιαίου Λυκείου Χ.Κ. Αχτσαλωτίδη.

7) <Πα παιδιά του Ευκλείδη» Ηλίας Κωνσταντό­πουλος.

8) Επιθεώρηση επιστημονικών και εκπαιδευτι­κών θεμάτων Περιοδικά: 3° και 4° Παιδαγωγι­κό Ινστιτούτο.

9) Κωνσταντίνος Καραθεοδωρή - Ο Σοφός Έλ­λην του Μονάχου Δέσποινα Καραθεοδωρή­Ροδοπούλου

Δέσποινα Βλαχοστεργίου-Βασβατέκη. 10) «Η εκατόμβη του Πυθαγόρα» Γιάννης Γ. Ταμ­

βακλής. 1 1 ) ΣΠΟΥ ΔΑΙ I SOUDAI Πανεπιστήμιο Πειραι­

ώς. 12) Συνθετική Δημιουργική Εργασία στα Μαθη­

ματικά και την Πληροφορική. Α.Ε. ΜΠΑΡ­ΜΠΑΣ

13) Μαθηματικά Γενικής Παιδείας, Άλγεβρα­Γεωμετρία (Διαγωνισμ. Νέου Τύπου) Ελένη

Σουλιώτη - Αργύρης Γαμβρέλλης. 14) «Το τελευταίο θεώρημα του Φερμά» Simon

Singh, Εκδόσεις: Π. Τραυλός. 1 5) Μαθηματικά - Διαγωνίσματα Νέου Τύπου Α­

σπιώτη Όλγα 16) «Παίζει ο Θεός ζάρια - Η επιστήμη του Χά­

ους» Ian Stewart Εκδόσεις: Π. Τραυλός. 1 7) Μαθηματικά Β' Ενιαίου Λυκείου Θετικής Κα­

τεύθυνσης (3) θεωρία Αριθμών Κώστα Αντώνη Κυριακόπουλου.

1 8) Μαθηματικά Β' Ενιαίου Λυκείου Θετικής Κα­τεύθυνσης ( 1) Διανύσματα Ευθεία στο επίπε­δο.

1 9) ΓΕΩΜΕΤΡΙΑ Β' ΛΥΚΕΙΟΥ Θ. ΑΝΔΡΙΟ­ΠΟΥΛΟΣ

20) «Προβλήματα Θεωρίας Αριθμών» Δημήτρης I. Μπουνάκης.

2 1 ) ΜΑΘΗΜΑτΙΑ Β' Λυκείου (Α' και Β ' τόμος) Δ. Γουβίτσα.

22) ΆΛΓΕΒΡΑ Β' Λυκείου Αλέξανδρος Π. Τρα­γανίτης.

23) «Στοιχειώδεις Διαφορικές Εξισώσεις και προ­βλήματα συνοριακών τιμών» William Ε. Boyce - Richard C. Dipήma Πανεmστημιακές εκδόσεις Ε.Μ.Π.

24) Ασκήσεις Διαφορικής Γεωμετρίας Θέματα Πτυχιακών Εξετάσεων Θ. Κυριακόπουλος

25) «Μαθηματικά Γ Λυκείου» Δύο Τόμοι, Ηλίας Κωνσταντόπουλος, Εκδόσεις Γκρίτζαλης.

ΕΥΚΛΕΙΔΗΣ Β' λ.ε. τ.l/31

Page 34: Ευκλειδης Β 41

Α ν κάποιος θέλει να πάψει να φοβάται το κε­φάλαιο της Τριγωνομετρίας, πρέπει ν' αποφασίσει να διαβάσει προσεκτικά τους ορισμούς των αρχι­κών εννοιών απ' όποιο βιβλίο κι αν θελήσει.

Ειδικά θα πρέπει να συνειδητοποιήσει ότι οι τριγωνομετρικοί αριθμοί (ημίτονο, συνημίτονο, ε­φαπτομένη και συνεφαπτομένη) μιας γωνίας δεν αφορούν αποκλειστικά οξείες γωνίες αλλά και γω­νίες ω με 0° � ω � 360° , επίσης γωνίες μεγαλύτε­ρες των 360° αλλά και αρνητικές γωνίες.

Επομένως: Οι ορισμοί που έμαθε στην Β' Γυ­μνασίου και αφορούν τους τριγωνομετρικούς αριθ­μούς μιας οξείας γωνίας δεν είναι τελικά οι μόνοι που υπάρχσuν.

Στην Γ Γυμνασίου αλλά ειδικά στην Α' Λυ­κείου τέθηκαν οι ορισμοί των τριγωνομετρικών α­ριθμών μιας γωνίας (γενικά) όχι κατ' ανάγκη οξεί­ας. Σύμφωνα μ' αυτούς, για να οριστούν οι τριγω­νομετρικοί αριθμοί μιας γωνίας ω πρέπει να τοπο­θετηθεί «κατάλληλα» σ' ένα σύστημα καρτεσια­νών συντεταγμένων όπως φαίνεται στο σχήμα 1 . Συγκεκριμένα, η γωνία ω ταυτίζεται με τη γωνία που παράγεται από τον ημιάξονα Οχ (αρχική πλευρά) όταν στραφεί γύρω από το Ο κατά την θε­nκή φορά (δηλαδή την αντίθετη των δεικτών του ρολογίου). Η τελική θέση του ημιάξονα Οδ είναι η τελική πλευρά της γωνίας.

Στη συνέχεια, αν Μ(α,β) τυχαίο σημείο της τελικής πλευράς της γωνίας ω θα ισχύει:

Υ δ Μ

� -------- β (ΟΜ)=ρ=�α2 +β2 >0 .( 1 )

α χ

Σχ. 1 · Ονομάζουμε:

β ( τεταγμενη του σημειου Μ ) • ημω =- = ---'�___:_ __ ..::....._ __

ρ αποσταση του Μ απο το Ο

• συνω = � (= τετμημενη του σημειου Μ ) ρ αποσταση του Μ απο το Ο

εφω =- με α :;e Ο • β ( τεταγμενη του σημεισυ Μ ) α τετμημενη τουσημεισυ Μ

του Κώστα Βακαλόπουλου

• σφω = � ( τετμημενη τουσημεισυ Μ ) με β :;e 0 . β τεταγμενη του σημεισυ Μ

Προφανώς: Αν συνω :;e Ο τότε εφω = ημω συν ω

ο , συνω και αν ημω :;e τοτε σφω = -- . ημω Επειδή το σημείο Μ είναι τυχαίο και οι παρα­

πάνω λόγοι είναι σταθεροί δηλαδή ανεξάρτητοι α­πό την επιλογή του σημείου Μ ( αφού λόγω των ο­μοίων τριγώνων που σχηματίζονται οι λόγοι είναι σταθεροί), λαμβάνουμε ως σημείο Μ το σημείο που η τελική πλευρά της γωνίας τέμνει τον κύκλο κέντρου 0(0,0) και ακτίνας ίση με τη μονάδα μή­κους ( ρ = 1 ) (τΡΙΓΩΝΟΜΕΤΡΙΚΟΣ ΚΥΚΛΟΣ). - Κι αυτό, γιατί; - Απλούστατα γιατί πλέον οι προηγούμενοι ορι-

-Ι . Α

σμοί, απαλλάσσονται από τον παρανομαστή α­φού αυτός ισούται με 1.

Υ

Β' - 1 Σχ. 2

Α χ

Έτσι για το ημίτονο και το συνημίτονο έ­χουμε: (σχ. 2) • ημω = β (= τεταγ­

μένη του Μ) • συνω = α ( = τετ μη­

μένη του Μ)

Εύκολα αποδεικνύεται (βλέπε απόδειξη σχο­λικού βιβλίου) όn η εφω είναι ίση με την τεταγμέ­νη του σημείου Ε, σημείο που η τελική πλευρά της γωνίας ή η προέκτασή της τέμνει τον εφαπτόμενο άξονα του κύκλου στο σημείο Α (σχ. 3).

Αντίστοιχα η σφω είναι ίση με την τετμημένη του σημείου Σ, σημείο που η τελική πλευρά της γωνίας ή η προέκτασή της τέμνει τον εφαπτόμενο άξονα του κύκλου στο σημείο Β (σχ. 3).

Υ Σ Β

σφω

Σχήμα 3

χ

Επομένως: Αν Μ, Ε και Σ τα

σημεία του τριγωνομε­τρικού κύκλου όπως αυτά ορίστηκαν πιο πάνω ι­σχύει:

ΕΥΚΛΕΙΔΗΣ Β' λε τ.l/32

Page 35: Ευκλειδης Β 41

Μαθηματικά yια την Α· Λυκεiου ημω = yΜ · συνω = χΜ εφω = yε σφω = χΣ

ΆΣΚΗΣΗ

(τεταγμένη του Μ)

(τετμημένη του Μ)

(τεταγμένη του Ε) (τετμημένη του Σ)

Ας κάνουμε τώρα μια άσκηση και να εφαρμό­σουμε τους παραπάνω ορισμούς:

Βρείτε τα σημεία Μ, Ε και Σ που αντιστοι-

χούν στις γωνίες 0° (0), 90° (π

), 180° (π), 270° 2

( 3π

), 360° (2π) 2

(Απ.: ι , Ο, -, 0 / Ο, -ι , Ο, - / -ι , Ο, -, 0 / Ο, ι , Ο, -)

ΕΦΑΡΜΟΓΕΣ

Ο Με τη βοήθεια του Τριγωνομετρικού κύκλου εύκολα καταλαβαίνουμε γιατί ισχύουν οι παρακάτω σχέσεις: • Για κάθε γωνία ω ισχύει: 1-ι ::;; ημω ::;; ιj και

1-ι ::;; συνω ::;; ι! , αφού οι τεταγμένες και οι τε­

τμημένες του σημείου Μ όλων των γωνιών στον τριγωνομετρικό κύκλο βρίσκονται μετα­ξύ των αριθμών -ι και ι (σχ. 2).

• Για κάθε γωνία ω και για κάθε κ ε .Ζ ισχύει:

jημ(κ · 360° + ω) = ημωj (ή jημ (κ · 2π + ω) = ημωj ),αφού οι τελικές πλευρές

των γωνιών: κ · 360° + ω , (ή κ · 2π + ω), κ ε .Ζ ταυτίζονται με την τελική πλευρά της γωνίας ω (ο­μοίως -yια τους υπόλοιπους τριγωνομετρικούς α­ριθμούς).

ΠΑΡΆΔΕΙΓΜΑ:

ημ Β:π

= ημ 13�; 2π

ημ[( ι ι + ι� )2π J =

ημ(ι ι - 2π + �; )= ημ(ι ι - 2π +� )= ημ: =�. 8 Με τη βοήθεια του τριγωνομετρικού κύκλου

μπορούμε να αναγάγουμε τους τριγωνομετρι­κούς αριθμούς μιας γωνίας σε τριγωνομετρι­κούς αριθμούς γωνίας που η τελική της πλευρά βρίσκεται στο zo τεταρτημόριο (χωρίς να χρειά­ζεται να aπομνημονεύσουμε τους αντίστοιχους . ,. τυπους./. Αντίθετες γωνίες:

Στο σχήμα 4 οι ευθείες ΟΜ ΟΜ' τέμνουν τον τριγωνομετρικό κύκλο στα σημεία Μ, Μ' αντί­στοιχα, που είναι συμμετρικά ως προς τον άξονα χ'χ: έχουν ίσες τετμημένες και αντίθετες τεταγμέ­νες, δηλ. ίσα συνημίτονα και αντίθετα ημίτονα.

Προφανώς θα έχουν αντίθετες εφαπτομένες και συνεφαπτομένες.

ημ(-χ) = -ημχ

συν(-χ) = συνχ

εφ(-χ) = -εφχ

σφ(-χ) = -σφχ

ΑΣΚIΙΣΗ

Υ

Σχ. 4

Επαληθεύοντας τις μέχρι τώρα γνώσεις σας και με πολύ προσοχή συμπληρώστε τα κενά στους πα­ρακάτω πίνακες εκφράζοντας τους τριγωνομετρι-

π κούς αριθμούς των γωνιών: π - χ , π + χ , 2 - χ

συναρτήσει των τριγωνομετρικών αριθμών της γωνίας χ

Παραπληρωματικές γωνίες Στο σχήμα 5 Τα σημεία Μ και Μ' είναι συμ­

μετρικά ως προς τον άξονα y'y, άρα έχουν αντίθε­τες τετμημένες και ίσες τεταγμένες.

ημ(π - χ) = ;

συν(π - χ) = ;

εφ(π - χ) = ;

σφ(π - χ) = ;

Γωνίες με διαφορά π

Υ

Σχ. 5

Στο Σχήμα 6 τα σημεία Μ και Μ' είναι συμ­μετρικά ως προς την αρχή 0(0,0) των αf,όνων, άρα f:χ.ουν αντίθετες τετμημένες και αντίθετες τεταγμένες δηλ . ....

ημ(π + χ) = ; .

συν(π + χ) = ;

εφ(π + χ) = ;

σφ (π + χ) = ;

Υ

Συμπληρωματικές γωνίες

ημ(; - Χ )= ; Υ

συν(; - χ )= ;

εφ(; - χ )= ;

σ�(; - χ )= ;

χ

Σχ. 6

Σχ. 7

Στο Σχήμα 7 τα σημεία Μ και Μ' είναι συμ­μετρικά ως προς την διχοτόμο της γωνία xOy, άρα

ΕΥΚΛΕΙΔΗΣ Β' λε τ.l/33

Page 36: Ευκλειδης Β 41

Μαθηματικά yια την Α" Λυκείου

το Μ' έχει τετμημένη την τεταγμένη του Μ και τε­ταγμένη την τετμημένη του Μ.

ΠΑΡΑΤΗΡΗΣΕΙΣ (ΣΤΗΝ ΕΦΑΡΜΟΓΗ 2) 1 ) Κατ' αρχήν οι απαντήσεις στη παραπάνω

άσκηση: i) ημχ, -συνχ, -εφχ, -σφχ, ii) -ημχ, -συνχ, εφχ, σφχ, iii) συνχ, ημχ, σφχ, εφχ.

2) Οι παραπάνω τύποι περιέχονται στο σχο­λικό βιβλίο με την μορφή: η μ (1 80° - ω) = . . . για την περίπτωση (i), η μ (1 80° +ω) = . . . για την πε­ρίπτωση (ii) και ημ(90° -ω) = . . . για την περί­πτωση (iii).

3) Οι παραπάνω τύποι: αναγωγής στο 1 ο τε­ταρτημόριο ισχύουν και για οποιαδήποτε γωνία χ.

' π 3π 3π 4) Για τις γωνιες - + χ , - - χ και - + χ , 2 2 2

έργαζόμαστε όπως στο παράδειγμα που ακολου­θεί: • ημ(�+χ )=ημ[�-(-χ)Ι� συν(-χ) = συνχ

• ημ(� -χ )=ημ[ π+(�-χ )]� -ημ(�-χ J�-rnm • ημ(3; + χ )= ημ[2π - (�- χ )]�

=ημ[ -(�- χ )]�- ημ(� - χ )= -συνχ

(*) θυμίζουμε ότι: ημ(κ · 2π + χ) = ημχ , κε Ζ .

5) Γενικά ισχύει: i) Γωνίας, που η τελική της πλευρά βρίσκε­

ται στο 2° τεταρτημόριο, οι τριγωνομετρικοί αριθ­μοί υπολογίζονται συναρτήσει των τριγωνομετρι­κών αριθμών της γωνίας που υπολείπεται των 1 80° (ή π rad).

1 π.χ. συν1200=συν(1800-60")=-συν60"=-- . . 2

ii) Γωνίας, που η τελική της πλευρά βρίσκεται στο 3° τεταρτημόρω, οι τριγωνομετρικοί αριθμοί υ­πολογίζονται συναρτήσει των τριγωνομετρικών α­ριθμών της γωνίας που υπερβαίνει των 180° (ή π rad). π.χ. εφ Ίπ = εφ(π +�)= εφ� = J3 . 6 6 6 3

iii) Γωνίας, που η τελική της πλευρά βρίσκεται στο 4° τεταρτημόρω, οι τριγωνομετρικοί αριθμοί υ­πολογίζονται συναρτήσει των τριγωνομετρικών α­ριθμών της γωνίας που υπολείπεται των 360° (ή 2π rad). π.χ. σφ Sπ = σφ(2π - �)= -σφ π = -J3 .

3 3 . 3 3 Ο Με τη βοήθεια του τριγωνομετρικού κύκλου

μπορούμε να λύσουμε τριγωνομετρικές εξισώ-

σεις και ανισώσεις. Ισχύει:

-� ημχ = α <=> ημχ = ημθ <=>

{ χ= κ: · 2π+θ <::::> ή , Κ: Ε Ζ χ = κ: · 2π+π-θ

(όπου θ , γωνία που έχει ημίτονο α).

-1 Σχ. 8

Εξήγηση: Οι γωνίες χ (σε rad) που έχουν ημί­τονο α, δηλ. όσο το ημθ είναι οι γωνίες που οι τε­λικές τους πλευρές τέμνσύν τον κύκλο

α)

• στο Μ οπότε: χ = κ · 2π+ θ , κ ε Ζ ή • στο Μ' �πότε: χ = κ · 2π + (π-θ) , κε Ζ ΠΑΡΆΔΕΙΓΜΑ α) Να λυθεί η εξίσωση: ημχ =_!_ (1) 2 β) Να λυθεί η ανίσωση: ημχ � _!_ (2) 2 Λύση

Υ ι

-1 Σχ. 9

1 β) ημχ � 2

1 π ημχ = 2 = ημ6 (::>

2 π ' χ = κ · π+- η 6 5π

χ χ = κ · 2π +- , κ ε Ζ . 6

Σύμφωνα με τον τριγωνομετρικό κύκλο του σχήματος, λύσεις της ανίσωσης (2) είναι οι γωνίες των οποίων οι τελικές πλευρές τέμνουν τον κύκλο σε σημεία του ελάσσονος τόξου ΜΜ' δηλ. οι γω-, π < < 5π ' νιες με - _ χ _ - και γενικοτερα: 6 6

π 5π Κ · 2π + - :5; Χ :5; Κ · 2π +- , Κ Ε Ζ . 6 6

ΑΡΑ: Λύσεις της παραπάνω ανίσωσης είναι οι πραγματικοί αριθμοί χ που ανήκουν στην ένωση των διαστημάτων της μορφής: [κ · 2π+� κ · 2π + Sπ] κε Ζ . 6 ' 6 '

ΑΣΚΗΣΗ Μελετώντας προσεκτικά τον τριγωνομετρικό κύκλο να συμπληρώσετε τις λύσεις των παρακάτω

εξισώσεων:

ΕΥΚΛΕΙΔΗΣ Β" λε τ.l/34

χ

Page 37: Ευκλειδης Β 41

Μαθηματικά για την Α' Λυκείου

{χ =; -ι,;;α,;;ι

συνχ =α <=> συνχ = συνθ <=> χ�

; •

(θ: γωνία με συνημίτονο α) αεΙR

• εφχ = α<=> εφχ = εφθ <=> χ = ; (θ: γωνία με εφαπτομένη α)

αεΙR • σφχ = α<=>σφχ = σφθ <=> χ = ;

(θ: γωνία με συνεφαπτομένη α) (Απ.: χ = κ · 2π+ θ ή χ = κ · 2π-θ I χ = κπ+ θ I χ = κπ+ θ, κ ε Ζ )

Ασκήσεις προς λύση 1 ) Να αποδειχθεί η ισοδυναμία:

Ιεφχ . εφyl < 1 <=> Ιημyl < lσυvxl .

2) Αν α,β ε � δείξτε ότι: (ί) l4 · ημα + 1 1 · συναl � 15 , (ii) 14αημχ - 5βσυvχl � 4lαl + 5 lβΙ .

3) Να επιλύσετε (ως προς χ) τις εξισώσεις: (ί) χ2 - (ημα+ συνα)χ + ημα · συνα = Ο . (ii) χ2 - 2χ + ημ2α = Ο .

(ίίί) χ2 - χ + 1 = ο . ημα · συνα

4) Να υπολογιστεί η τιμή των παραστάσεων:

Α ημ(1 170°) + συν1 170° + 2 εφl 080° - 3 · συν1 080°

17π ημ-- εφπ - 2 Β - 1 7 συν� +εφ(-π)+ 1 2 .

5) i) Ν' απλοποιηθεί η παράσταση:

Α = ημ(π + χ) · συν(τ - χ }εφ(τ + χ ) _ ημ(2π - χ) ίί) Ν' αποδείξετε ότι:

ημ2( -� )+συν(7π+χ) · ημ(-�-χ )= 1+συνχ . 2 ( 13π ) συν 9π+ημ -τ-χ

6) Να υπολογιστούν οι τιμές των παραστά-σεων:

Α = ημz36ο + ημz 54 ο + ημz 1 8ο + ημz72ο

Β = (η μ � . η μ 25π } (η μ 3; . η μ �π )

7) Ν α λυθεί η εξίσωση: 3εφ( χ-�)+ εφ(-χ - π)

=_!_ 2εφ(π - χ) + 6 2

3� I ..,. Σ � l8' ιtΙΙΙJΑι ιι Ε TIM'rι;� Το κεφάλαιο των απόλυτων τιμών από τη φύ­

ση του προβληματίζει και δημιουργεί ερωτηματικά και στα οποία θα προσπαθήσουμε με παραδείγμα­τα και ασκήσεις που θα προτείνουμε να ξεπερα­στούν οι όποιες δυσκολίες.

Την απόλυτη τιμή ενός πραγματικού αριθμού α την συμβολίζουμε με lαl . Ως lαl ορίζουμε: «τον ίδιο τον αριθμό α, αν ο α είναι θετικός ή μηδέν και τον αντίθετό του -α, αν ο α είναι αρνητικόρ).

{α για α � Ο Δηλαδή lαl = ορσ -α για α < Ο

Ας πάρουμε μια απέραντη ευθεία και ας είναι Ο η αρχή και Ι ένα αυθαίρετο σημείο διαφορετικό από το Ο. Στο Ο aντιστοιχούμε το Ο και στο Ι τον 1 . Το διάνυσμα ΟΙ είναι μοναδιαίο.

ο Μ � ------��---4�--------�----- +οο

ο χ

Επιμέλεια: Νίκος Α. Κόντζιας Σε κάθε σημείο Μ του άξονα αντιστοιχεί ένας

αριθμός χ τέτοιος ώστε: ΟΜ = χ · ΟΙ . Ο χ λέγεται τετμημένη του Μ.

Αντίστροφα σε κάθε πραγματικό αριθμό χ α­ντιστοιχεί ένα σημείο Μ του άξονα τέτοιο ώστε: - -χ · ΟΙ = ΟΜ .

Συνεπώς μπορούμε τα σημεία του άξονα να τα ταυτίσουμε με τις τετμημένες τους. Έτσι μπορούμε να μιλάμε για άξονα των πραγματικών αριθμών και να έχουμε μια γεωμετρική παράσταση του πραγματικού αριθμού.

Έστω Α και Β δύο σημεία του άξονα με τε­τμημένες α και β αντίστοιχα και το σημείο Ο με τετμημένη Ο.

Απόσταση των σημείων Α και Β ονομάζουμε την απόλυτη τιμή της διαφοράς των τεrμημένων τους δηλαδή (ΑΒ) = Ια - βι = Ιβ - αι .

Απόσταση του σημείου Α από το Ο ονομά-

ΕΥΚΛΕΙΔΗΣ Β' λε τ.l/35

Page 38: Ευκλειδης Β 41

Μαθηματικά 'fUI την Α' Λυκείου

ζουμε την απόλυτη τιμή της τετμημέvης του Α α­φού: (ΑΟ) = I α- Ol = lαl .

Έτσι μπορούμε να πούμε ότι η απόσταση με­ταξύ του Μ με τετμημέvη χ και του Α με τετμημέ-νη -α είναι lx + αl αφού lx + αl = lx - (-α)l .

Θεωρούμε απαραίτητο να αναφέρουμε κά­ποιες βασικές ιδιότητες των απόλυτων τιμών που η απόδειξή τους βρίσκεται στο σχολικό βιβλίο.

α) Για κάθε πραγματικό αριθμό α ισχύουν οι ιδιότητες

1) -lαl � α < lαl 2) 1-αl = lαl 3) lαl2 = α2 4) lαl2v = α2ν , ν Ε Ν•

I 12ν+Ι α , α - 0 5) α = νΕ Ν { 2ν+Ι > Ο -α2ν+Ι ' α < Ο

6) Για κάθε πραγματικό αριθμό χ και θ > Ο ισχύει η ισοδυναμία lxl � θ <=> -θ � χ � θ

7) Α ν θ > Ο και β, χ τυχαίοι πραγματικοί α-ριθμοί ισχύουν jx -βj � θ <=> β -θ � χ � β + θ

8) lx + yj � lxi + IYI για κάθε x, yE IR 9) lx - yl � lxi + IYI 10) ι;ι =:;: . y :;e o 1 1)Αν θ > Ο τότε lχl > θ <=> χ < -θ ή χ > θ Εστιάζουμε την προσοχή μας στις ανισότητες

της μορφής: Ι χ -βl < θ ή ( lx -βl � θ ) και Ι χ -βl > θ ή ( Ι χ -βl � θ > με θ > ο για να δούμε την γεωμετρική της ερμηνεία στον άξονα των πραγμα­τικών αριθμών. • lχ - βl < θ

Η ανισότητα αυτή δηλώνει το διάστημα στο ο­ποίο βρίσκονται οι πραγματικοί αριθμοί χ που η α­πόσταση τους από το β είναι μικρότερη του θ δη­λαδή

χ' θ μονάδες θ μονάδες 14 �14 �I

• • 0')---ιι·--- χ β-θ β χ β+θ

Δηλαδή ο αριθμός χ διατρέχει το ανοιχτό διά­στημα (β -θ,β +θ) . (Πράγματι:

1 χ -β Ι < θ <=> -θ < χ -β < θ <=> β -θ < χ < β + θ > • Ιχ -βl > θ

Η ανισότητα αυτή δηλώνει το διάστημα στο ο­ποίοπ βρίσκονται οι πραγματικοί αριθμοί χ που η απόσταση τους από το β είναι μεγαλύτερη του θ

χ χ χ' --ο,____, • ._ _ __, • ._ ______ ο-- χ

β-θ β β+θ

Δηλαδή ο αριθμός χ διατρέχει την ένωση των

διαστημάτων ( -οο, β -θ) υ (β + θ, + οο) .

(Πράγματι: Ι χ -βl > θ <=> χ -β < -θ ή χ -β > θ <=> χ < β -θ ή χ > β + θ ).

Παραδείγματα 1) l x - 21 < 4 <=> -4 < χ - 2 < 4 <=> -2 < χ < 6

Δηλαδή λύσεις είναι η αριθμοί που απέχουν από το 2 λιγότερο από τέσσερις μονάδες οπότε παίρνουμε το διάστημα (-2,6) . 2) lx + 41 � 2 <=> χ + 4 � -2 ή χ + 4 � 2 <=> χ � -6 ή χ � -2 <=> (-oo, -6]u[-2, +oo) .

Άλλωστε η aνίσωση γράφεται: I χ -( -4)1 � 2 και έχει λύσεις τους πραγματικούς αριθμούς που απέχουν από το -4 περισσότερο ή ίσο από δύο μονάδες.

ΑΣΚΗΣΕΙΣ 1) Αν y = χ - 2lxl - t4 να αποδείξετε ότι 7 + lxl

yια κάθε χ Ε IR ισχύει -3 < y < -1 .

Λύση 7 + lxl > Ο για κάθε χ Ε IR . Συνεπώς η παρά­

σταση ορίζεται για κάθε χ Ε IR . Θα προσπαθήσουμε να βρούμε αν υπάρχουν

πραγματικοί αριθμοί β με κ > Ο ώστε η σχέση -3 < y < -1 να γράφεται ισοδύναμα στην μορφή ly + βl < κ . Έχουμε ότι -3< y<-1 <=>-3+β< y+β<-1+β . Απαιτούμε -3+β+( -1 +β) =0<=>2β=4<=>1β=21 . Συνεπώς η -3 < y < -1 γράφεται -1 < y + 2 < 1 . , x -2 lxl - 14 Εχουμε Υ I I <=> 7 + χ

2 x -2 lxl - 14 2 x - 2 lxl - 14 + 14+ 2 lxl y+ + = 7 + lxl 7+ lxl χ , I I I χ I lx l = -ι-ι αρα y + 2 = -ι-ι = -ι-1 < 1 . 7 + χ 7 + χ 7 + χ Συνεπώς έχουμε ly + 21 < 1 <=> -3 < y < -1 για

κάθε χ Ε IR . 2) Να επιλυθεί η ανίσωση

I ι

I > s (1). 2χ- 1

Λύση π , , , ι , αρατηρουμε οτι η τιμη "2 ειναι απαγορευτι-

κή για το χ καθότι το κλάσμα δεν έχει έννοια. Με την παρατήρηση ότι χ :;e .!.. η aνίσωση (1) γράφε-2

ΕΥΚΛΕΙΔΗΣ Β' U: τ.l/36

Page 39: Ευκλειδης Β 41

Μαθηματικά ΎΙfΙ την Α· Λυκείου

ται l2x - ιl < .!_ και είναι ισοδύναμη με την 5 12(χ -.!..) < .!.. ή ,χ _.!.., < _!_ ή __ ι < χ -.!.. < _ι 2 5 2 ιο ιο 2 10 . 4 6 • 2 3 η - < χ < - η - < χ < - .

10 10 5 5 2ιsr&\ .. Ν 3/5

Άρα η (ι) αληθεύει στο σύνολο S ( 2 ι ) ( ι 3 ) . ξ . . ι = -,- υ - ,- αφου ε αιρειται η nμη - . 5 2 2 5 2

3) Να επιλυθεί η εξίσωση

l5- xl - 3lx + ιΙ = 2χ - 5 . Λύση

χ - 1 5 5-χ + + δ -x+l - ο + +

I α) Αν χ < -ι έχουμε 5 - χ > Ο , χ + ι < Ο άρα

η εξίσωση γίνεται 5 - χ - 3(-χ - ι) = 2χ - 5 Οχ = -ι3 και είναι αδύνατη. β) Αν -ι s; χ :S; 5 , τότε: 5 - χ ;?: Ο , χ + ι s; ο . Η εξίσωση γίνεται 5 - χ -3(χ + 0 = 2χ -5 5- χ- 3χ - 3 = 2χ - 5 -6χ = -7 χ = '!._ δεκτή διόn: -ι < '!._ < 5 .

6 6 γ) Αν χ > 5 , τότε 5 - χ < Ο , χ + ι > Ο Η εξίσωση γίνεται χ - 5 - 3(χ + 0 = 2χ - 5

χ = -� απορρίπτεται. οπότε λύση της εξίσωσης 4

. 7 ειναι το - . 6

4) Να επιλυθεί και να διερευνηθεί η εξί­

σωση χ + lxl = α όπου α πραyματικός αριθμός.

Λύση Αν α < Ο , η εξίσωση δεν έχει λύση. Αφού για κάθε χ ισχύει -Ι χΙ s; χ ή Ο s; χ + lxl .

Έστω α ;?: Ο Διακρίνουμε τις περιπτώσεις: ί) χ ;;?: ο ' ίί) χ < ο . Στην πρώτη περίπτωση η εξίσωση γίνεται:

· 2 · α · δ · χ + χ = α η χ = α η x = l και ειναι εκτη. Στην δεύτερη περίπτωση η εξίσωση γίνεται:

χ - χ = α ή Ο · χ = α οπότε:

Για α = Ο η εξίσωση έχει ως λύση κάθε χ Ε (--οο,Ο) .

Για α > Ο η εξίσωση δεν έχει λύση. Παρατήρηση: Επειδή για κάθε χ Ε JR ισχύει χ + Ι χΙ ;;?: Ο η εξίσωση έχει λύση μόνο όταν α ;;?: Ο .

5) Εάν οι αριθμοί χ, y, ω είναι πραyματικοί

αριθμοί τέτοιοι ώστε lxl < jyj < lωl να δειχθεί

Χ1 + y1 lxl + jyj ω 1 < lωl ·

Λύση Έχουμε lxl < lωl ( 1 ). Πολλαπλασιάζουμε και

τα δύο μέλη της ( 1 ) με το lxl και η (ι) γίνεται lxl2 s; lxl lωl (2). Πολλαπλασιάζω την (2) και τα δύο μέλη της με � αφού lωl > jyj οπότε lωl > Ο . lωl

Σ ώ lxl2 < lxl lωl . � < � (3) υνεπ ς lωl2 - lωl2 η ω2 - lωl · Ομοίως έχουμε �: < ��� (4) αφού jyj > Ο διότι jyj > lxl . Προσθέτοντας κατά μέλη τις (3) και (4) έχου-

Χ2 + y2 lxl jyj . Χ2 + y2 lxl + jyj με ω2 < lωl + lωl η ω2 < lωl 6) Εάν χ = 9 + 3k , y = 4 + 3k και a Ε JR

τότε να αποδείξετε ότι la - χΙ + ja - yj ;;?: 5 • Λύση Παίρνουμε το άθροισμα

ι ι y - x --+-- = . με x ;t: a , y ;t: a . a - x y - a (a - x)(y - a) la�x + y�a l = l<�=:�;=:) l =

= l (a -��;- a) l = la - xfιy - aj la�x l+ ly�a � ;?: �a�x + y�a l = ιa - x�y -aj Οπότε με πολλαπλασιασμό και των δύο μελών

της προηγούμενης ανίσωσης με la - xljy - aj παίρ-νουμε jy - aj + la - xl ;?: 5 .

Για χ = a ή a = 9 + 3k η αποδεικτέα γίνεται: 19 + 3k - 4- 3kl ;;?: 5 και ισχύει ως ισότητα.

Όμοια εργαζόμαστε για y = a . 7) Να δείξετε ότι η απόλυτη τιμή του α-

ΕΥΚΛΕΙΔΗΣ Β' λε τ.l/37

Page 40: Ευκλειδης Β 41

Μαθηματικά για την Α' Λυκείου

θροίσματος δύο aντίστροφων αριθμών είναι πά­ντοτε μεγαλύτερη ή ίση του δύο.

Απόδειξη

Αρκεί να δείξουμε lx + �� � 2 . α) Εάν χ > Ο έχουμε lx2 + 1 1 > 2 lx2 + 11 > 2 2 1 > 2 χ - <=> lxl - <=> χ + - χ

lxl = x άρα χ2 - 2χ + 1 > 0 <::::> (χ - 1)2 � 0 . αφού

β) Ένα χ < Ο lxl = -χ και η σχέση γράφεται χ2 + 1 � 2lxl ή αληθεύει πάντοτε. Η ισότητα ισχύ­ει όταν χ = ±1 .

8) Να δειχθεί ότι η απόσταση Jα lxi + PxJ μπορεί πάντοτε να τεθεί υπό την μορφή Alxi + Bx .

Λύση α) Εάν χ � Ο τότε lxl = χ και η παράσταση

γίνεται Ιαχ + βχl = J(α +β)χJ = lα+βl lxl = Ια + βl χ + κ lxl - κ lxl = (lα + βl - κ)lxl + κχ ( 1). Η σχέση (2) έχει τη ζητούμενη μορφή αν θέσουμε Α = lα + βl - κ και Β = κ .

β) Εάν χ < Ο lxl = -χ και η παράσταση παίρνει τη μορφή Jα(-χ) + βχJ = J(β -α)χJ = lβ - αl lxl = lβ - αl lxl + κ lxl - κlxl = (lβ - αl + κ)lxl + κx πάλι έ­χει τη ζητούμενη μορφή εάν Α = I β - αl + κ και Β = κ .

Επειδή η ( 1) και (2) θέλουμε να συμπίπτουν ανεξάρτητα με τις υποθέσεις για τον πραγματικό αριθμό χ θα πρέπει Ι α + βl - κ = Ι β - αl + κ οπότε

Ια + βi - Ιβ - αl . κ = 2 αρα

Jαlxl + βxJ = Ια + βi + Ιβ - αl lχl + Ια+ βi - Ιβ - αl χ . 2 2 9) Να aντιστοιχίσετε κάθε στοιχείο της

στήλης (Α) με το ίσον του της στήλης (Β). Στήλη (Α) lx - 31 < 2 d (2,x) � 3 d (l,x) � 2

Στήλη (Β) χ � -· ή χ � 3

l < x < 5 χ � -· ή χ � 5

1 Ο) Α ν είναι 1 � χ � 4 τότε η παράσταση �(χ- ι/ +�(χ- 4/ ισούται με

Α. 3 Β. Ο Γ. 2 Δ. 4

11) Να επιλυθεί η εξίσωση lx - lxll = 3 - 2lxl (1).

Λύση Είναι γνωστό ότι -lx l � χ � lxl . Συνεπώς lx - lxl l = lxl - χ . Άρα η (1 ) γράφεται

lxl - χ = 3 - 2 lxl <=> 3lxl - x = 3 (2). Αν χ > Ο η (2) γράφεται

3χ - χ = 3 <=> 2χ = 3 <=> χ = � δεκτή. 2 Αν χ < Ο η (2) γράφεται

-3χ - χ = 3 <=> -4χ = 3 <=> χ = -� δεκτή. 4 12) Να επιλυθεί η εξίσωση

lx - 21 - 1 = 12 - xl (1). 5 Λύση Γνωρίζουμε ότι Ι χ - 21 = 12 - χΙ άρα η (1 ) γίνε-

5 ται lx -2l -5=5lx -21 <=>4lx-21 =-5 <=>lx-21 =--4 αδύνατο.

Οι παρακάτω ασκήσεις προτάθηκαν από τον συνάδελφο ΠαναΎιώτη Κουρσουμπά.

1) Ν α λυθούν οι εξισώσεις i) llxl + 316 1 = 48 ii) IΙχΙ + C23 )2 1 = 231 • 2) Έστω χ, y, z πραγματικοί αριθμοί και ό-μηδέν τέτοιοι ώστε � = lil = �. Αν v 3 7 5 χ+ y + 2z = Ο · να δειχθεί ότι χ, y είναι ομόσημοι.

Λύση Είναι χ + y = -2z άρα lx + Yl = l-2zl = 2lzl . Ακόμα lx i + IYI = 3Izl + Ί lzl = 2lzl άρα 5 5

lx + Yl = lx l + IYI · Συνεπώς χ, y ομόσημοι. 3) Αν χ πραγματικός αριθμός να δειχθεί ότι

Ι χ+ 31 + Ι χ+ ιΙ � lxl + Ι χ+ 41 . Λύση 'Ε χουμε (Ι χ + 31 + lx + 11)2 � Clxl + lx + 41)2 <=>

μετά τις πράξεις έχουμε lx2 + 4χ + 31 � lx2 + 4xl + 3 που ισχύει.

4) Αν -2 � χ � 3 και -l � y � l να βρεθεί η μέγιστη τιμή της παράστασης: Α =12χ + 3yl .

ΕΥΚΛΕΙΔΗΣ Β' λε τ.l/38

Page 41: Ευκλειδης Β 41

Μαθηματικά -yια την Α' Λυκείου

Λύση

-3 < -2 � χ � +3 <=> l x � 31 -1 � Υ � + 1 <=> I YI < 1

Στο άρθρο αυτό αναπτύσσουμε τις βασικές με­θόδους απόδειξης μιας συνεπαγωγής, με στόχο να βοηθηθεί ο μαθητής της πρώτης τάξης στην κατα­νόηση της αποδεικτικής διαδικασίας η οποία κυριαρ­χεί στα μαθηματικά του Λυκείου (και όχι μόνο).

Κάθε άσκηση ή πρόταση ή θεώρημα, συνήθως διαχωρίζεται σε δύο ισχυρισμούς, την υπόθεση (Υ) και το συμπέρασμα (Σ). Για παράδειγμα:

Έχουμε το «Θεώρημω): «Οι διαγώνιες ενός ορθογωνίου είναι ίσεφ στο

οποίο υπόθεση είναι ο ισχυρισμός: <<Έχουμε ένα ορ­θογώνιο παραλληλόγραμμο ΑΒΓ Δ)). Συμπέρασμα είναι ο ισχυρισμός: «Οι διαγώνιες ΑΓ, ΔΒ είναι ί­σε9).

ΥΠΟΘΕΣΗ λοtπόν είναι ένας ισχυρισμός ή μια συνθήκη που ΘΕΩΡΕΙΤΑΙ ΑΛΗΘΗΣ (έστω και αν λογικά ή οπτικά (στο σχήμα) δεν φαίνεται να αλη­θεύει).

Η υπόθεση, δηλαδή, ·είναι όπως λέμε επίσης τα δεδομένα.

ΣΥΜΠΕΡ ΑΣΜΑ είναι μια πρόταση της οποί­ας προσπαθούμε να δικαιολογήσουμε ότι είναι α­ληθής. Στηριζόμαστε στην υπόθεση και (βήμα προς βήμα) με λογική σκέψη (δηλαδή με βάση γνωστούς κανόνες και ιδιότητες) προσπαθούμε να δείξουμε ότι αληθεύει το συμπέρασμα (ζητούμενο).

Γενικά: Κάθε πρόταση που περιέχει υπόθεση και συμπέρασμα λέγεται ΣΥΝΕΠΑΓΩΓΉ.

Η υπόθεση εισάγεται συνήθως με τη λέξη: «'Έ­στω)), «Θεωρούμε)), «Αν)) κτλ. ή ακόμα πολλές φο­ρές η υπόθεση υπονοείταL

Το συμπέρασμα συνήθως εισάγεται με τη λέξη, τότε ή aποδείξτε ότι. Πολλές φορές όμως υπονοεί­ται.

Για παράδειγμα: 1. Αν το άθροισμα των ψηφίων ενός φυσικού

αριθμού α διαφείται με το 3, τότε ο α διαφείται με το 3.

σ ες. 2. Οι διαγώνιες ισοσκελούς τραπεζίου είναι ί-

3. Ισχύει η ταυτότητα: (α + β)(α-β) = α2 - β2 (1)

δηλαδή: αν οι α, β είναι πραγματικοί αριθμοί, τότε ι­σχύει η (1).

Απόδειξη συνεπαγωγής είναι η διαδικασία κατά

Ισχύει l2x + 3yl � l 2xl + I3YI = 2 lxl + 3 IYI � 9 . Το μέγιστο λαμβάνεται για χ = 3 , y = 1 ή y = -1 .

Γ. Τσικαλουδάκης

την οποία, με_ τη βοήθεια γνωστών κανόνων και ιδιο­τήτων που ισχύουν για τα δεδομένα (Υ), προσπα­θούμε να δικαιολογήσουμε την αλήθεια του συμπε­ράσματος (Σ).

Το συμπέρασμα (Σ) είναι συνήθως: μια ισότη­τα, μια ανισότητα, μια ισοδυναμία, (ή μια νέα συ­νεπαγωγή), ή μια έννοια η οποία έχει ορισθεί μέσω άλλων συνθηκών.

Για παράδειγμα: 1 . Αν αε JR* , τότε α2 +� � 2 .

α 2. Αν αε JR, β ε JR και ν ε Ν , τότε

α < β <::::> α2ν+Ι < β2ν+Ι . 3. Α ν ένα τετράπλευρο έχει τις διαγώνιες κάθετες

μεταξύ τους, τότε είναι ρόμβος. Η απόδειξη μιας συνεπαγωγής μπορεί να γίνει

με διάφορες μεθόδους, που θα δούμε παρακάτω. Η επιλογή της μεθόδου, εξαρτάται κάθε φορά

από την υπόθεση (Υ) και το συμπέρασμα (Σ). 1 η ΜΕΘΟΔΟΣ: «ευθεία απόδειξη» Ξεκινάμε από την υπόθεση (Υ) και με τη βοή­

θεια γνωστών ιδιοτήτων και κανόνων προσπα­θούμε να καταλήξουμε στην αλήθεια του συμπε­ράσματος (Σ).

ΠΑΡΆΔΕΙΓΜΑ Ν α αποδείξετε ότι:

6 1 α +-6 = 2 . α

ΑΠΟΔΕΙΞΗ

1 αν α+-= 2 , τότε α

Είναι: α+�= 2 , άρα (α+�)2 = 4 , άρα και

α2 +- = 23 ή α6 + - = 2 . ( 1 )3 1 α2 α6

2η ΜΕΘΟΔΟΣ: «απαγωγή σε άτοπο» Όπως παραπάνω αναφέραμε, σε μια συνεπα­

γωγή η υπόθεση (Υ) θεωρείται αληθής. Τα δεδομένα δηλαδή της άσκησης αποτελούν

την υπόθεση και δεν μπορούμε να θεωρήσουμε ό­τι δεν ισχύουν.

ΕΥΚΛΕΙΔΗΣ Β' λε τ.l/39

Page 42: Ευκλειδης Β 41

Μαθηματικά yια την Α' Λυκείου

Σε πολλές περιπτώσεις δεν είναι εύκολο από τα δεδομένα να καταλήξουμε στην αλήθεια του συμπεράσματος με την «ευθεία μέθοδο». Αυτό φαίνεται καθαρά στα παρακάτω παραδείγματα: ι. Αν α2 > Ο , τότε α :;t Ο . 2. Αν α5 :;t ι , τότε α :;t ι . 3. Αν δύο ευθείες ει , ε2 που τέμνονται από τρίτη

σχηματίζουν τις εντός και επί τα αυτά μέρη γωνίες με άθροισμα μικρότερο από ιsοο' τότε οι ευθείες αυτές ( ει , ε2 ) τέμνονται.

4. Αν α · β :;t Ο , τότε α :;t Ο και β :;t Ο . 5. Αν το γινόμενο α · β δύο φυσικών αριθμών

δεν είναι άρτιος τότε οι α, β δεν είναι άρτιοι. 6. Αν α - β + αβ :;t ι , τότε α :;t ι και β :;t -ι .

ΑΠΟΔΕΙΞΗ Για να δικαιολογήσουμε την αλήθεια του συ­

μπεράσματος, καταφεύ'yουμε στην παρακάτω απλή (λογική) σκέψη. ι. Γιατί αν α = Ο , τότε θα είναι:

α2 = α · α = Ο · Ο = Ο , δηλαδή α2 = Ο . Όμως η υπό­θεση είναι α2 > Ο (το οποίο δεν μπορούμε να α­πορρίψουμε).

Συνεπώς η ισότητα α2 = Ο στην οποία (με υ­πόθεση α = Ο ) καταλήξαμε είναι λάθος ( ανnβαί­νει στην υπόθεση).

Άρα το συμπέρασμα α :;t Ο είναι σωστό. 2. Ομοίως για α = ι προκύπτει α5 = ι το ο­

ποίο αντιβαίνει στην υπόθεση ( α5 :;t ι). Άρα α :;t ι . 3. Αν οι ει , ε2 δεν τέμνονται, τότε (ορισμός)

είναι παράλληλες. Όμως γνωρίζουμε ότι δύο πα­ράλληλες ευθείες τεμνόμενες από τρίτη σχηματί­ζουν τις εντός και επί τα αυτά μέρη γωνίες με ά­θροισμα ιsοο.

Το συμπέρασμα όμως αυτό, ανnβαίνει στην υπόθεση.

Γενικά με τη μέθοδο της «απαγωγής σε άτο­πο» υποθέτουμε όn Γενικά, για να αποδείξουμε μια συνεπαγωγή με τη μέθοδο της παραγωγής σε άτοπο: υποθέτουμε ότι δεν αληθεύει το συμπέ­ρασμα (Σ) (δηλαδή θεωρούμε ότι αληθεύει η άρ­νηση του συμπεράσματος Σ).

Με υπόθεση την πρόταση «{η) Σ» (άρνηση του Σ), καταλήγουμε (με ευθεία μέθοδο) σε μια πρόταση Υ' που είτε γενικά δεν ισχύει είτε έρχεται σε αντίφα­ση με την υπόθεση Υ (της συνεπαγωγής).

ΙΣΟΔΥΝΑΜΙΑ Σε πολλές περιπτώσεις αναφερόμαστε σε δύο

προτάσεις Πι , Π2 (ή σε δύο ποσότητες), λέγοντας όn είναι ισοδύναμες και γράφουμε συμβολικά:

Πι <=> Πz

Ή ακόμα γενικότερα αναφερόμαστε σε δύο μεγέθη, λέγοντας ότι είναι ίσα ως προς κάποω κοινό μέτρο σύγκρισης.

Για παράδειγμα: 1 . Δύο χαρτονομίσματα των 500 δρχ. είναι ι­σοδύναμα (ίσης αξίας) με ένα των ι 000 δρχ.

2. Δύο δοχεία των 50 lt είναι ισοδύναμα (ί­σης χωρητικότητας) με ένα των ι 00 lt. 3. Δύο τρίγωνα με ίσες βάσεις και ίσα ύψη

είναι ισεμβαδικά (ισοδύναμα ως προς το εμβαδόν) αλλά όχι κατ' ανάγκη ίσα.

Γενικά: στα Μαθηματικά, έχοντας ως μέτρο σύγκρισης την αλήθεια ή μη μιας πρότασης, λέμε όn δύο προτάσεις Πι , Π2 είναι ισοδύναμες, όταν είναι ταυτόχρονα και οι δύο αληθείς ή και οι δύο ψευδείς. Τότε γράφουμε: Πι <=> Π2

ΠΑΡΆΔΕΙΓΜΑ ΤΑ: 1. Αν α ε ΊR , τότε οι προτάσεις α2 < 1 και

-ι < α < ι είναι ισοδύναμες. 2. Αν Α , Β , Γ είναι τα μέτρα γωνιών τρι­

γώνου ΑΒΓ, τότε: i) Α = 90° <=> B + f' = 90° ίί) Α = 90ο <=> βz +γz = αz .

ΑΠΟΔΕΙΞΗ ΙΣΟΔ ΥΝΑΜΙΑΣ ( Π1 <=> Π2 )

Για να αποδείξουμε ότι αληθεύει μια ισοδυ­ναμία (ως συμπέρασμα μιας συνεπαγωγής), δηλα­δή ότι και οι δύο προτάσεις είναι ταυτόχρονα αλη­θείς (ή ψευδείς) έχουμε δύο τρόπους.

ι ος τρόπος (Διάσπαση σε δύο συνεπαγωγές) α) Θεωρούμε ως αληθή την Πι (δηλαδή υ­

ποθέτουμε ότι αληθεύει η Πι ) και αποδεικνύουμε (συμπέρασμα) την αλήθεια της Π2 κατόπιν

β) Θεωρούμε ως αληθή την Π2 (υπόθεση η Π2 ) και αποδεικνύουμε (συμπέρασμα) την Πι .

ΠΑΡΆΔΕΙΓΜΑ: 'Εστω υμ , υ1 τα ύψη τριγώνου ΑΒΓ. Ισχύει

η ισοδυναμία: Ρ = γ <=> υμ = υ1 • ΑΠΟΔΕΙΞΗ α) Έστω ότι στο τρίγωνο

ΑΒΓ είναι ΑΒ = ΑΓ (Υ). Τό­τε τα τρίγωνα ΑΒΕ, ΑΔΓ είναι ίσα γιατί έχουν: ΑΒ = ΑΓ Α κοινή Ε=Δ=90° . Άρα ΕΒ = ΓΔ ή υp = υ1 (Σ). Αντίστροφο

β) Έστω ότι υp = υ1 ή ΕΒ = ΓΔ (Υ).

Α

ΕΥΚΛΕΙΔΗΣ Β' λε τ.Ι/40

Page 43: Ευκλειδης Β 41

Μαθηματικά -yια την Α' Λυκείου

Τότε τα τρίγωνα ΑΒΕ, ΑΔΓ είναι ίσα γιατί: ΕΒ = ΓΔ , Α κοινή, Ε = Δ = 90° .

Άρα ΑΒ = ΑΓ (Σ). ΣΗΜΕΙΩΣΗ: ι. Πολλές φορές η ισοδυναμία δύο προτάσε­

ων Πι , Π2 δηλώνεται (φραστικά) ως εξής: ί) Αν (αληθεύει) Πι , τότε (αληθεύει) η Π2 και αντίστροφα. ίί) (αληθεύει) Πι αν και μόνο αν (αληθεύει) 1\ .

2 . Όπως παραπάνω αναφέραμε η απόδειξη μιας ισοδυναμίας « Π 1 � Π 2 » γίνεται με την από­δειξη δύο συνεπαγωγών: (συμβολικά):

α) αν Πι , τότε Π2 β) αν Π2 , τότε Πι . Σε πολλές περιπτώσεις, αφού έχουμε αποδεί­

ξει την μία από τις (α), (β) η άλλη αποδεικνύεται με την μέθοδο της «απαγωγής σε άτοπο».

ΠΑΡΑΔΕΙΓΜΑΤ Α ι. Α ν α, p είναι πραΎματικοί αριθμοί να α-

ποδειχθεί η ισοδυναμία: α1 + Ρ1 = Ο � α = Ρ = Ο .

ΑΠΟΔΕΙΞΗ α) Έστω α = β = Ο , τότε έχουμε: α2 = Ο και

β2 = 0 , άρα α2 + β2 = 0 . Α ντίστροφα: Έστω α2 + β2 = Ο (υπόθεση) Έστω ότι δεν αληθεύει το συμπέρασμα ( α = Ο

και β = Ο ). Τότε θα είναι ή α ;ι!: Ο ή β ;ι!: Ο ή (α * Ο και β ;ι!: 0). Αν είναι α ;ι!: Ο , τότε είναι α2 > Ο . Ο­πότε α2 + β2 > Ο+ β2 και επειδή β2 � Ο συμπεραί­νουμε όn α2 + β2 > 0 , άτοπο (ανnφάσκει στην υ­πόθεση α2 + β2 = Ο ).

Με ανάλογο τρόπο καταλήγουμε σε άτοπο αν υποθέσουμε β ;ι!: Ο .

Άρα το συμπέρασμα α = β = Ο , αληθεύει. 2. Έστω α,βε IR και ν φυσικός αριθμός. Να

αποδείξετε ότι α2ν+ι < β2ν+ι , αν και μόνο αν α < β . ΑΠΟΔΕΙΞΗ Έστω α < β . Διακρίνουμε τις περιπτώσεις: ί) Α ν Ο < α < β , τότε προφανώς είναι

α2ν+ι < β2ν+ι • ίί) Αν α < β < Ο , τότε είναι -α > -β > Ο οπό­

τε ( -α)2v+ι > (-β )2ν-ι ή -α2ν+ι > -β2ν+ι ή α2v+ι < β2ν+ι •

ίίί) Α ν α < Ο < β , τότε πpοφανώς είναι α2ν+ι < 0 < β2v+ι ' άρα α2ν+ι < β2ν+ι •

Α ντίστροφο: Έστω ότι είναι α2ν+ι < β2ν+ι ( 1). Αν α > β , τότε έχουμε β < α και συνεπώς, ό-

. δ ίξ θ . β2ν+ι 2ν+ι πως παραπανω απο ε αμε, α ειναι < α , άτοπο λόγω της (1).

Α ν α = β , τότε ως γνωστό θα είναι α2ν+ι = β2ν+ι , που είναι άτοπο λόγω της ( 1).

Άρα θα ισχύει α < β . 2°ς τρόπος (απόδειξης ισοδυναμίας) «μέθο­

δος ισοδυναμιών» « Π 1 {:::::;> Π 2 » Με υπόθεση την αλήθεια μιας από τις Π:ι ,

Π2 , π.χ. της Πι και στηριζόμενοι σε άλλες γνω­στές προτάσεις και στο ότι είναι ισοδύναμες μετα­ξύ τους και με την Πι και την Π2 , έχουμε την α-λήθεια της Π2 •

Συμβολικά: Πι � Π3 � Π4 � · · · � Π κ και Πκ � Π2 .

ΠΑΡΆΔΕΙΓΜΑ

ι Σε τρίγωνο ΑΒΓ ισχύει: A>B+r � Α>�. ΑΠΟΔΕΙΞΗ

Έχουμε A > B + f' � A + A > A + B + r � 2Α > ι sοο � Α > 90° .

2. Αν αε IR , τότε ισχύει: α1 <ι � -ι<α<ι . ΑΠΟΔΕΙΞΗ

α2 < 1 � α2 - 1 < Ο � (α - Ι) (α + Ι) < Ο � « α - 1, α + 1 : ετερόσημοι» � α- l < Ο < α + l � -1 < -α < l � l > α > -1 .

3η ΜΕΘΟΔΟΣ απόδειξης συνεπαγωγής Μετά από την παραπάνω έννοια των ισοδυνά­μων προτάσεων και τις μεθόδους απόδειξης ισοδυ­ναμίας, μπορούμε τώρα να δούμε και μια πολύ χρήσιμη μέθοδο απόδειξης μιας συνεπαγωγής, τη μέθοδο των ισοδυναμιών.

Έστω ότι θέλουμε να αποδείξουμε την συνε-παγωγή: Αν α < -1 , τότε α + ! < -2 . α

Αν δοκιμάσουμε την ευθεία απόδειξη, θα δια­πιστώσουμε ότι είναι δύσκολο (δεν ξέρουμε πως να αρχίσουμε) και αν ακόμα ο καθηγητής μας συμβουλέψει: «έχω α < -1 , άρα α+ 1 < Ο , άρα ( )2 . (α+ 1)2 Ο λ θη , α+ 1 > Ο , οποτε < κτ . ο κα γητης α για να κάνει αυτά τα βήματα, μάλλον ξέρει τη λύ­ση και από κάποιο άλλο σκεπτικό έχει οδηγηθεί σ' αυτά. Το σκεπτικό αυτό είναι το εξής:

Θεωρούμε ότι το συμπέρασμα είναι αληθές.

ΕΥΚΛΕΙΔΗΣ Β' λε τ.l/41

Page 44: Ευκλειδης Β 41

Μαθηματικά yια την Α' Λυκείου

Δηλαδή θεωρούμε ότι ισχύει η ανισότητα: α+.!. < -2 . Τότε θα αληθεύει και κάθε ισοδύναμη

α

' , , Δ λαδ ' , (α+1)2 Ο (Σ') μ αυτην προταση. η η εχω: <=>-- < α

Να λοιπόν γιατί ο καθηγητής, στην ευθεία α-πόδειξη είπε: α+ 1 < Ο , άρα (α+ 1)2 > Ο κτλ.

Το τελευταίο συμπέρασμα (Σ'), λόγω της υ­πόθεσης (α < -1 ) είναι πλέον προφανές ότι αλη­θεύει, οπότε λόγω των παραπάνω ισοδυναμιών, συμπεραίνουμε ότι πράγματι αληθεύει και το συ­μπέρασμα (Σ) της αρχικής μας συνεπαγωγής.

ΣΗΜΕΙΩΣΗ: Το τελευταίο συμπέρασμα (Σ'), δεν είναι ισο­

δύναμο με την υπόθεση α < -1 . Δηλαδή: δεν ι-

, δ

, (α+ 1)2 Ο 1 σχυει η ισο υναμια: < <=> α < - . α Συνεπώς στην συνεπαγωγή: «αν Υ, τότε Σ» οι

προτάσεις Υ και Σ δεν είναι υποχρεωτικά ισοδύναμες. Γενικεύοντας λοιπόν, για να αποδείξουμε μια

συνεπαγωγή: «αν Υ, τότε Σ» με τη μέθοδο των ι­σοδυναμιών, εργαζόμαστε ως εξής:

Με ισοδύναμες προτάσεις με το (Σ): Σ <=> Σ1 <=> Σ2 <=> . . · <=> {Σκ )

προσπαθούμε να καταλήξουμε σε μια πρόταση (Σκ ) της οποίας η αλήθεια προκύπτει πολύ εύκο­λα από την υπόθεση (Υ) (με την ευθεία απόδειξη).

ΠΑΡΑΔΕΙΓΜΑΤΑ 1. Αν α < β < 1 , να αποδειχθεί ότι

α2 + β > α+αβ .

ΛΥΣΗ: Είναι α2 + β > α + αβ <=> α2 - αβ + β - α > Ο <=>

α(α-β) - (α -β) > Ο <=> (α- β)(α - 1) > Ο ( 1 ) Αλλά έχουμε: α < β <=> α-β < Ο

α < 1 <=> α- 1 < 0 και συνεπώς είναι (α-β )(α - Ο > Ο .

2. Έστω α > 1 και β > 1 .

Ν α αποδειχθεί ότι: α + β < 1 . 1 + αβ

ΛΥΣΗ:

'Ε χουμε: α+ β < 1 <=> α+ β - 1 < Ο ή 1 + αβ 1 + αβ

α + β - 1 - αβ < Ο <=> (α - 0(1 - β) < Ο (1)

1 + αβ 1 + αβ Αλλά είναι: (α - 0(1 -β) < Ο . Ακόμα είναι 1 + αβ > Ο , οπότε η (1) αληθεύει.

ΠΡΟΤΕΙΝΟΜΕΝΕΣ ΑΣΚΗΣΕΙΣ 1 1 1

1 . Αν - + - + - = 0 , να αποδειχθεί ότι α β Υ

(α+ β + γ)2 = α2 + β2 + γ2 . 2. Αν α :;t β και α2 + αβ + β2 = 1 , να αποδεί­

ξετε ότι: α3 - α = β3 -β . 3. Αν α+β + γ = 1 και 3α2 + 3β2 + 3γ2 = 1 ,

να δείξετε ότι α = β = y . 4. Αν α,βε ΊR να αποδείξετε την ισοδυναμία:

α � -β <=> α3 +β3 � αβ(α+β) . 5. Αν ο φυσικός αριθμός ν είναι πολλαπλά­

σιο του 3, να αποδειχθεί ότι και ο ν2 - 2ν είναι πολλαπλάσιο του 3. Με ένα κατάλληλο παράδειγ­μα να δείξετε ότι το αντίστροφο γενικά δεν ισχύει.

6. Α ν α, β ε ΊR , να αποδειχθεί ότι: α :;t β <=> α3 + α :;t β3 + β .

7. Έστω ΑΒΓ ισοσκελές τρίγωνο ( ΑΒ = ΑΓ ) και Δ, Ε τα μέσα των ΑΒ, ΑΓ αντίστοιχα. Έστω Μ σημείο της ΒΓ. Να αποδειχθεί ότι το Μ είναι μέσο της ΒΓ αν και μόνο αν ΜΔ = ΜΕ .

8. Έστω (Ο,ρ) , (O,R) δύο ομόκεντροι κύ-κλοι με ρ < R και ΑΒ, ΑΓ δύο χορδές του (Ο, ρ) . Οι ημιευθείες ΑΒ, A;r τέμνουν τον {Ο, R) στα Δ, Ε αντίστοιχα. Ν α αποδείξετε ότι: ΑΒ = ΑΓ <=> ΒΔ = ΓΕ .

9. Δύο ίσοι κύκλοι (Κ, ρ) , (Λ, ρ) εφάπτο­νται εξωτερικά στο σημείο Ε. Αν η εφαπτομένη του (Κ, ρ) στο σημείο του Α και η εφαπτομένη του (Λ, ρ) στο σημείο του Β, τέμνονται στο ση­μείο Μ, να αποδειχθεί ότι: ΜΑ= ΜΒ <=> ΕΑ = ΕΒ .

�ΝΙΣΙJΤΗΤΕΣ Οι σχέσεις που ισχύουν για την αντιμετώπιση

των Ασκήσεων στις ανισότητες είναι: 1 . α > β και β > y τότε α > y .

Του Σωτήρη Γιαννοσπύρου 2. α > β <=> α± Υ > β± Υ . 3 . Α ν α > β και y � δ τότε α + y > β + δ . 4. Αν α � β και λ > Ο τότε: αλ � βλ και

ΕΥΚΛΕΙΔΗΣ Β' λε τ.l/42

Page 45: Ευκλειδης Β 41

Μαθηματικά -yια την Α' Λυκείου

5. Α ν α > β και λ < Ο τότε: αλ < βλ και α β - <- . λ λ

6. Αν α > β <:::::> -α < -β . 7. Α ν α � β και β � α τότε α = β . 8. Αν α � β και γ � δ τότε α+ γ � β + δ . 9. Αν α > β � Ο και γ > δ � Ο τότε ay > βδ . 10. Αν α > Ο , β > Ο και ν Ε Ν με α � β τότε

αν � βν .

ι ι Α ν α · β > Ο τότε: α < β <:::::> _!_ > _!_ . α β

Πρέπει να τονιστεί όn: • Δεν μπορούμε να αφαφέσουμε ανισότητες κα­

τά μέλη. • Δεν μπορούμε να διαιρέσουμε ανισότητες κα­

τά μέλη ακόμη κι αν τα μέλη είναι όλα θετικά. Για την επίλυση ανισοτήτων οι μέθοδοι που

προτείνονται είναι: lη ΜΕθΟΔΟΣ της ΔΙΑΦΟΡΑΣ Η ανισότητα της μορφής

A(x1 , x2 0 . . . ,xv ) � B(x1 ,x2 0 . . . , xv ) (ι) με μετα­βλητές χ1 , χ2 0 . . . , χν λύνεται με την μέθοδο της διαφοράς ως εξής: Μετασχηματίζουμε την (ι) στην ισοδύναμη: A (x1 , x2 , . . . ,xv )-B (x1 ,x2 , . . . , xv ) � O (2). Η (2) με πράξεις, φτάσει σε προφανή σχέση.

Π. χ. ι ) Ν α δειχτεί ότι για κάθε χ Ε JR ισχύει:

χ2 + 2 � 2 (ι) .Jχ2 + ι Λύση

Έχουμε

(γιατί). Η τελευταία ισχύει για κάθε χ Ε JR . Χ2 1

2) Για κάθε χΕ JR ισχύει: --4 � - (για l + x 2

εξάσκηση).

2η ΜΕθΟΔΟΣ ΣΥΝθΕΤΙΚΉ Παρατηρούμε ποιες απλές ανισότητες παρου­

σιάζονται στην ζητούμενη ανισότητα και συνθέτο­ντας κατάλληλα αυτές καταλήγουμε στην ζητού­μενη σχέση. Π.χ.

ί) Για οποιουσδήποτε θετικούς αριθμούς

χ, y να δείξετε ότι: .j;y - � � Ο . 2 x + y

ίί) Αν α, β, γ Ε JR: , τότε να δειχτεί ότι: ..{σ.β + Jϊi1 + .Γα1 �-_1r_- __![__ � ο .

2 α + β β + γ α+ γ

Λύση i) Έχουμε: χ + y � 2.J;:y

( fx -./Υ)2 � Ο <:::::> χ - 2fx.jY + y � Ο .

αφού

Οπότε: .J;:y(x + y) � 2xy ή Μ � � -2 x + y

ii) Από το ερώτημα i) έχουμε για α, β, γ Ε JR+ :

Fαί3 -� � ο . .JβΎ -Κ� ο . .ΓαΎ -� � ο 2 α+β 2 β+γ 2 α+γ

οπότε με πρόσθεση κατά μέλη προκύπτει: Fαί3+$r+.Γσ.Ύ -� -_k_-� � 0 .

2 α+β β + γ α+γ

3η ΜΕθΟΔΟΣ ΓΡΑΜΜΙΚΟΥ ΜΕΤ Α­ΣΧΗΜΑΤΙΣΜΟΥ

ι) Α ν στην υπόθεσή μας δίνονται μία ή πε­ρισσότερες ανισοτικές σχέσεις, τότε μετασχηματί­ζουμε αυτές τις σχέσεις σε σχέσεις ισότητας δηλα­δή αν: • α, β Ε JR και α > β τότε: α = β + χ με χ > Ο .

• α, β Ε JR: και α > β τότε: � = χ με χ > ι . β

2) Αν έχουμε συμμετρική παράσταση ως προς τις μεταβλητές που περιέχονται στην ανισό­τητα, τότε θεωρούμε μια τυχούσα διάταξη γι' αυ­τές π.χ.

Αν α, β, γ Ε JR: τότε ισχύει: (α+ β)(β +γ)( γ + α) � 8αβγ (ι).

Λύση Η σχέση (ι) είναι συμμετρική ως προς τα α, β,

γ, θεωρούμε λοιπόν την διάταξη α � β � γ τότε: α = γ+ χ , χ � Ο και β = γ + y με y � Ο . Άρα:

Έστω: (α + β)(β + γ) (γ+ α) � 8αβγ <:::::>

(2γ+χ +y)(2γ+y)(2γ+ χ) � 8γ( γ+ χ)( γ+y) <:::::>

γ(χ - y)2 +γχ2 +γy2 + x2y + xy2 � Ο που ισχύει γιατί χ, y,γΕ JR: .

Άλλος τρόπος: x,yE JR+ =::} χ + y � 2.J;:y ( <:::::> ( fx -./Υ γ � Ο ) Άρα:

α+ β � 2Fαβ , β + γ � 2$γ , γ + α � 2�

ΕΥΚΛΕΙΔΙΙΣ Β' λε τ.l/43

Page 46: Ευκλειδης Β 41

Μαθηματικά για την Α· Λυκεiου

Οπότε πολλαπλασιάζουμε κατά μέλη κι έχου- σοδύναμα με: (α+ β)(β +γ)(γ+ α) � 8�α2β2γ2 = Sαβγ . χ2 +2χ�χ2 +β2 +χ2 +β2 -β2 _2χ2 _2χ�χ2 +β2 :5Ο

που ισχύει για κάθε β, χ Ε JR και μάλιστα ως ισό-4η ΜΕΘΟΔΟΣ ΤΗΣ ΑΠΑΓΩΓΗΣ ΣΕ Α­

ΤΟΠΟ Η μέθοδος αυτή εφαρμόζεται σε τύπους α­

σκήσεων της μορφής αν Α τότε Β (αν ισχύει η συνθήκη Α τότε ισχύει και η Β). Γι' αυτό το λόγο δεχόμαστε πως ισχύει αν Α ΚΑΙ ΟΧΙ Β και με πράξεις φθάνουμε σε αποτέλεσμα αντίθετο της υ­πόθεσης.

π Ν . θ . 2 - χ .χ. α συγκρινουμε τους αρt μους: --Υ

y + 2 2 . . . . ο και --+- οταν ειναι γνωστο οτι xy > . χ xy

Λύση Έστω

2 - χ y + 2 2 2 - χ y + 2 2 -- > --+- <=> xy-- > xy--+ xy- <=> Υ χ xy Υ χ xy

χ (2 - χ) > y(y+ 2)+ 2 <=> 2χ - χ2 > y2 + 2y+ 2 <=> y2 + 2y + 1 + 1 + χ2 - 2χ < ο <=> (y+ 1)2 + (χ - 1)2 < ο . Ά 2 - χ y + 2 2 [Τ . . ατοπο. ρα: -- < --+- . ο ισο ισχυει

Υ χ xy μόνον όταν: χ = 1 και y = -1 (γιατί;) αδύνατο α­φού xy > O ] .

ΓΕΝΙΚΕΣ ΑΣΚΗΣΕΙΣ 1) Για κάθε α,p,χΕ R ισχύει:

αz + Pz � 2 (α - χ) (χ + �χz + Pz ) (1).

Λύση Η ( 1) είναι ισοδύναμη με την

α2 + β2 -2(α- χ)(χ +�χ2 + β2 ) � ο ή

α2 _ 2(χ + �χ2 + β2 )α+ β2 + 2χ(χ + �χ2 + β2 ) � ο

(1) ή διότι:

β2 + 2χ (χ + �χ2 +β2 ) = β2 + χ2 + 2χ�χ2 + β2

= (�β2 + χ2 + χ)2 . Θρεωτούμε το τρίγωνο:

f(α) = α2 -2(χ +�χ2 +β2 )α+β2 +2χ(χ+�χ2 +β2 ) . Για να ισχύει η f (α) � Ο , για κάθε α Ε JR πρέ­

πει και αρκεί: «η διακρίνουσα Δ = 4(χ + �χ2 + β2 )2 -4[β2 + 2χ�χ2 + β2 ] να είναι μικρότερη ή ίση με το Ο (θεωρία τριωνύμου διδά­σκεται στην τελευταία περίοδο της Α'Τάξης) ή ι-

τη τα.

Παρατήρηση: Επειδή Δ = Ο συμπεραίνουμε ότι η (1) αναλύ­

εται σε τέλειο τετράγωνο.

2) Α ν α, β θετικοί ριιτοί και ,Jii άρριιτος

, • Ρ -αz ι τοτε. α + -- + ( ) > .JP (1)

2α + 1 4 2α +1

Λύση Έστω ότι δεν αληθεύει η ( 1) τότε: β -α2 1 α+--+

( ) :5 � κι επειδή 2α + 1 > Ο η 2α+ 1 4 2α+ 1

σχέση αυτή είναι ισοδύναμη με την 4α(2α+0+4(β - α2 )+ 1 :5 2 (2α + 1)2� ή

[(2α+ 1)- 2�Τ :5 ο � 2α + 1 - 2� = ο ή Γα 2α + 1 . . . Γαβ . ν μ = -

2- , που ειναι ατοπο γιατι ο ν μ αρρητος

2α + 1 · Ά (1) αλ θ · και ο -2- ρητος. ρα η η ευει.

τες: 3) Αν α,p,γΕ JR: να δειχθούν οι ανισότη-

i) (α; Ρ )z � αp .

ii) α + Ρ + Ύ � _!f!_ + .J!L. + __!!__ • 2 « + Ρ Ρ + Ύ α+ Ύ

Λύση

i) (α; β )2 � αβ <=> (α + β )2 � 4αβ <=>

α2 + 2αβ + β2 � 4αβ <=> (α- β)2 � Ο που ισχύει.

. . ) 1 . (α+ β )2 > β α+ β > αβ . ιι σχυει: -- _ α <=> -- _ -- , ο-2 4 α + β

μοια: γ+ α � _!!:1_ , όμοια: β + γ � Κ και συ-4 α + γ 4 β + γ

νεπώς α + β + γ + α + β + γ � �+_!!:'l_+K 4 Α 4 α + β α+γ β + γ

ή α+β+γ � �+K+_!!:'l_ . 2 α+β β + γ α+γ

ΕΥΚΛΕΙΔΗΣ Β' λε τ.l/44

Page 47: Ευκλειδης Β 41

Μαθηματικά -yια την Β· Λυκείου

Μαθηματικά yιa τn Β' τά�n του Λuκι:ίοu

Η συνάρτηση1 f με f(x) = ημ χ αποδεικνύεται ότι είναι περιοδική με ελάχιστη θεnκή περίοδο: 2π. Δηλαδή για κάθε χε IR ισχύει: f(x) = f(x + 2π) = f(x -2π). Γενικότερα για κάθε χε !R ισχύει: f(x) = f(x + 2kπ) Ι k ε Ζ.

Για το λiYyo αυτό είναι αρκετό η μελέτη της συνάρτησης αυτής να γίνει στο κλειστό διάστημα [0, 2π].

Α'

Β'

Α χ

Καθώς ο χ διατρέχει το πρώτο τεταρτημόριο, δηλαδή για Ο :::;; χ :::;; π οι

2 τιμές, που παίρνει η συ­νάρτηση, αυξάνουν καθώς αυξάνουν οι τιμές του χ.

Με άJλα λόγια η σχέση Ο :::;; χ1 < χ2 :::;; π συνε-2

πάγεται τη σχέση Υ1 = ημχ1 < ημχz = Yz · Με όμοιο τρόπο δείχνουμε ότι: η συνάρτηση

' ' ' φθ' [π 3π] ' αυτη ει ναι γνησια ινουσα στο "2, z και γνη-

σια αύξουσα στο [ �, 2π J. Είναι φανερό ότι: «η συνάρτηση αυτή παίρνει

τη μέγιστη τιμή της 1 για χ = π και την ελάχιστη 2

1 Η ανεξάρτητη μεταβλητή, αν δεν ορίζεται διαφοp>­τικά, εκφράζεται σε ακτίνια.

2 ο θ ' ο π 3π 2 ' μ ' ' ι αρι μοι: , -, π, -, π ειναι κομ ικα σημεια, 2 2

γιατί είναι τα σημεία που αλλάζει η συμπεριφορά (μονοτονία) της συνάρτησης. Αριστερά του Ο και δεξιά του 2π η συνάρτηση είναι ορισμένη. Οι διακεκομμένες γραμμές δηλώνουν ότι η μελέτη περιορίζεται, λ{yyo της περιοδικότητας, στο διάστημα [0, 2π].

Από τη Σ.Ε. Τελικά κατασκευάζουμε την γραφική παρά­

σταση της συνάρτησης f l [0,2π]3• Υ

ο π 7 π

.J ---------------------

Προβλήματα:

3π τ

Pz

2π χ

1°: Να μελετηθεί και να γίνει η γραφική παράσταση της συνάρτησης g με g(x) = ημ2χ.

Απάντηση: Παρατηρούμε ότι «για κάθε χ ε IR ισχύει:

g(x)·= g(x + π) = g(x - π) ». Διότι:

g(x) = ημ2χ = ημ(2χ + 2π) = ημ[2(χ + π)]= g(x + π), για κάθε χ ε IR.

Συνεπώς ο π παίζει για τη g το ρόλο που παί­ζει ο 2π για την f. Είναι αρκετό λοιπόν να μελετή­σουμε τη g στο διάστημα [0, π].

Συγκροτούμε τον πίνακα τιμών4: χ ο π/4 π/2 3π/4 π

2χ ο π/2 π 3π/2 2π

g(x) ο I ο -1 ο Τελικά κατασκευάζουμε την γραφική παρά­

σταση της συνάρτησης g Ι [0, π]5• Υ

χ

3 Η εξήγηση για την «καμπυλότητα» στο [Ο,π] και [π,2π] θα γίνει αργότερα (στην τρίτη τάξη). Το ίδιο γιατί «αλλάζει η καμπυλότητα στο π». 4 Πρακτικά φτιάχνουμε πρώτα τη δεύτερη γραμμή, που αποτελείται απ' τα κομβικά σημεία.

5 Κρατάμε τη «δομή» του σχήματος της Cr . Η όλη διαδικασία μοιάζει να συντελείται μέσω ((συμzίεση.; κατά μήκος του οριζόντιου άξονα» απ' το 2χ στο •-

ΕΥΚΛΕΙΔΗΣ Β' λ.ε. τ.Ι/45

Page 48: Ευκλειδης Β 41

Μαθηματικά για την Β· Λυκείου

2°: Να μελετηθεί και να γίνει η γραφική παράσταση της συνάρτησης h με h (χ) = ημ � . 3

Απάντηση: Παρατηρούμε ότι: «για κάθε χ Ε IR ισχύει:

h (x) = h (x + 6π) = h(x - 6π) ». Δηλαδή ο αριθμός 6π παίζει για την h τον ίδιο ρόλο που έπαιζε ο 2π για την f (πως βρίσκεται ο αριθμός 6π;)

Συγκροτούμε τον πίνακα τιμών: χ ο 3π/2 3 π 9π/2 6π

χ./3 ο π/2 π 3 π/2 2 π

h(x) ο ι ο - 1 ο

Τελικά κατασκευάζουμε την γραφική παράστα ση της συνάρτησης h Ι [0,6π]6•

ι ------ I 9π ' --------------- Jπ τ 6π 'Ε '· .� =---�-------------::ΞΞ::::::: i � :

Pz

3°: Να μελετηθεί και να γίνει η γραφική παράσταση της συνάρτησης φ με

φ (χ) =iημ( χ -:} Απάντηση: Παρατηρούμε ότι: «για κάθε χ Ε IR ισχύει:

φ(χ) = φ (χ + 2π) = φ(χ - 2π)». Δηλαδή η συνάρ­τηση φ έχει την ίδια περίοδο με την f. (Πώς το κα­ταλαβαίνουμε αυτό;)

Συγκροτούμε τον πίνακα τιμών: χ π/4 3π/4 5π/4 7π/4 9π/4

χ-π/4 ο π/2 π 3π/2 2π

φ( χ) ο 1 ο - 1 ο

Τελικά κατασκευάζουμε την γραφική παρά-

σταση της συνάρτησης φ 1 [: , 9: J 7• Υ

χ

4°: Να μελετηθεί και να γίνει η γραφική

6 Στην ουσία το σχήμα της Cr «τεντώνεται» απ' το 2π στο 6π.

7 Το σχήμα της Cr μετατοπίζεται προς τα δεξιά κατά

� και συμπιέζεται από πάνω και από κάτω ώστε η 4 . . Ι λ ' -Ι μεγιστη τιμη ειναι "2 και η ε αχιστη 2 .

παράσταση της συνάρτησης

u (χ) = l +�ημ(; - 2χ} Απάντηση:

υ με

Παρατηρούμε ότι: «για κάθε χ Ε IR ισχύει: u(x) = u (x + π) = u(x- π) )).

Διότι: u(χ)=l+�ημ(�-2χ )=1-�ημ(2χ-� )=

1 -�ημ[( 2χ -� )+ 2π] = 1 -�ημ [ 2 (χ + π) -�] =

1+�ημ[�-2(χ+π)]= u(χ +π) , για κάθε χ Ε IR . Συνεπώς η μελέτη της συνάρτησης αυτής

. . δ . [π 7π] μπορει να γινει στο ιαστημα "6'6" . Συγκροτούμε τον πίνακα τιμών:

χ π/6 5 π1 1 2 2π13 1 Ι π/1 2 7π/6

2χ-π/3 ο π/2 π 3π/2 2π

ημ(2χ-π/3) ο I ο - ι ο

υ( χ) I 1/3 I 513 I rnin Π>aΧ Τελικά κατασκευάζουμε την γραφική

σταση της συνάρτησης u Ι [ �· 76π J 8•

Υ 5/3 ---------------- Ρ,

ο π ο 5π 2π ! !π 7π π τ π τ

Η συνάρτηση F με F(x) = συνχ αποδεικνύεται ότι είναι περιοδική με ελάχι­στη θετική περίοδο 2π. Για τον λόγο αυτό η μελέτη της είναι αρκετό να γίνει στο κλειστό διάστημα [0,2π] .

χ

Β'

παρά-

Με άλλα λόγια η σχέση Ο � χ1 < χ2 � � συνε-2

πάγεται, όπως φαίνεται απ' το σχήμα, τη σχέση Υι = συνχ, > συνχ2 = Υ2 .

8 Στην ουσία γίνεται μια παράλληλη μετατόπιση από

το Ο στο ο'(�, Ι ) της καμπύλης g της προηγουμέ­

νης παραγράφου και μετά παίρνουμε το συμμετρικό ως προς την ευθεία με εξίσωση y = Ι σχήμα.

ΕΥΚΛΕΙΔΗΣ Β' λ.ε. τ.l/46

Page 49: Ευκλειδης Β 41

Μαθηματικά για την Β' Λυκείου

Με όμοιο τρόπο δείχνουμε ότι: «η συνάρτηση αυτή είναι γνήσια φθίνουσα στο διάστημα [�,π}>.

Ενώ είναι γνήσια αύξουσα στο διάστημα [π,2π]. Είναι φανερό ότι: «η συνάρτηση αυτή παίρνει

τη μέγιστη τιμή της 1 , για χ = Ο ή χ = 2π και την ελάχιστη τιμή της -1 για χ = π ».

Συγκροτούμε έτσι τον πίνακα μεταβολώΥ: χ

f(x)

ό πί2 ; ;

. . . � 3 π(2 2'π

('\.ι � '\.ι-:ι/� ./'i ιηηχ : ιηiη : m::ιχ Τελικά κατασκευάζουμε την γραφική παρά­

σταση της συνάρτηση F Ι [0,2π] . Υ

Πρόβλημα Να μελετηθεί και να γίνει η γραφική παρά-

σταση της συνάρτηση G με G (x) = 3 + συν� . 2 Απάντηση: Παρατηρούμε κατ' αρχάς ότι:

« G(x) = G(x + 4π) = G(x - 4π) , για κάθε x ε JR ».

G (χ) = 3 + συν; = 3 + συν(; + 2π )= χ + 4π '

= 3 + συν -- = G (χ + 4π) , για καθε χ ε JR . 2

Συνεπώς ο αριθμός 4π παίζει για την G τον ί­διο ρόλο που έπαιζε ο π για την F. Είναι λοιπόν αρκετό να μελετήσουμε τη G στο διάστημα [0,4π] .

Συγκροτούμε τον πίνακα τιμών: 2π 3π ••

χ/2 π12 )π/2 2π

συv(χ./2) - ι

Ci(x) 4 2 4 rnax ιτι.iη "'""' Τελικά κατασκευάζουμε τη γραφική παρά­

σταση της συνάρτησης G I [0,4π] . Υ

ΡΊ Ρ, 4 ---------------------3

I Ρ 2 _____ .ι, __ ...... __ -=-ι......--

ο 1 I I π 2π 3π

I I I I I I χ I 4π

9 Ισχύουν όσα σημειώσαμε στην υποσημείωση (2) της προηγούμενης παραγράφου.

Η συνάρτηση Φ με Φ(χ) = εφχ = ημχ είναι συνχ

ορισμένη σ' όλο το JR , εκτός απ' τους χ, για τους οποίους ισχύει συνχ = Ο .

Συνεπώς το πεδίο ορισμού της είναι:

Α = JR -{κπ+ � Ι κ ε z} . Κι επειδή εφχ = εφ(χ + π) , για κάθε χ ε Α , η

συνάρτηση αυτή είναι περιοδική με ελάχιστη θετι­κή περίοδο π. Συνεπώς η μελέτη της συνάρτησης αυτής είναι αρκετό να γίνει στο ανοικτό διάστημα ( -�·�}

Καθώς ο χ διατρέ­χει το πρώτο τεταρτη-μόριο δηλαδή για ο π ' ::;; χ < - οι τιμες, που

2 Α' παίρνει η συνάρτηση, αυξάνουν καθώς αυξά-νουν οι τιμές του χ. Με άλλα λόγια η συνθήκη:

Β'

ο< π ' '

' ' _χ1 <� <- συνεπαγεται οπως φαινεται απ το 2

σχήμα τη συνθήκη: Υι =εφχι = {Α'fι )< {ΑΤz )=εφχz =Yz ·

Ενδιαφέρον έχει να μελετηθεί η συμπεριφορά της συνάρτησης, καθώς ο χ προσεγγίζει τον

π ή ι-2

σοδύναμα καθώς το αντίστοιχο σημείο Μ πάνω στον κύκλο προσεγγίζει το Β.

Όπως φαίνεται απ' το σχήμα η ευθεία ΟΜ τείνει να πάρει τη διεύθυνση της ΟΒ καθώς το Μ προσεγγίζει το Β. Έτσι το αντίστοιχο σημείο Τ α­πομακρύνεται απ' το Α επ' άπειρον. Κι αυτό ση­μαίνει ότι η εφχ μεγαλώνει απεριόριστα, καθώς ο χ προσεγγίζει το � . Δηλαδή καθώς ο χ προσεγγίζει

2 το � , ο αντίστοιχος αριθμός εφχ μπορεί να γίνει

2 και να παραμείνει μεγαλύτερος από οποιονδήποτε θετικό αριθμό κ, όσο μεγάλος κι αν είναι. Κι αυτό σημαίνει ότι: «η γραφική παράστασης της συνάρ­τησης ως τροχιά ενός κινητού τείνει να πάρει τη διεύθυνση της κατακόρυφης ευθείας χ = π , καθώς

2 'ζ π ο χ προσεγγι ει τον - ».

2 Αργότερα (τρίτη τάξη - όρια) μαθαίνουμε ότι

η παραπάνω παρατήρηση συμβολικά διατυπώνεται

ΕΥΚΛΕΙΔΗΣ Β' λ.ε. τ.l/47

Page 50: Ευκλειδης Β 41

Μαθηματικά yια την Β' Λυκείου

π ως εξής: «όταν χ --7 2" , τότε εφχ --7 -too >> και

διαβάζεται: «η εφαπτομένη του χ τείνει στο -too , όταν το χ τείνει στο � καθώς διατρέχει το πρώτο

2 τεταρτημόριο>>.

Η συνάρτηση της εφαπτομένης είναι περιττή (αντίθετα τόξα έχουν αντίθετες εφαπτόμενες). Συ­νεπώς η γραφική παράσταση της συνάρτησης αυ-

τής για τους χ του διαστήματ�ς ( -�,0] είναι

συμμετρική ως προς την αρχή Ο των αξόνων του

τμήματος που αντιστοιχεί στο διάστημα [Ο,�) . Συγκροτούμε τον πίνακα τιμών:

-π π π π χ ο - - - Χ --7 -

6 4 3 2

εφ χ ο i1 ::: 0 57 3 ' 1 J3::: 1, 73 εψχ -Η·οο

Τελικά κατασκευάζουμε την γραφική παρά­

σταση της συνάρτησης Φ Ι ( -�, � ) 10•

- Π 7

I

Υ

χ

Η καμπύλη της συνάρτησης προσεγγίζει aσυ­μπτωτικά προς τα πάνω την ευθεία χ = � και προς

2

τα κάτω την ευθεία χ = -π

. 2

Πρόβλημα Να μελετηθεί και να γίνει η γραφική παρά-

σταση της συνάρτηση φ με φ (χ) = -1 + 3εφ πχ • 6

Απάντηση Η συνάρτηση αυτή είναι ορισμένη σ' όλο το

10 Η εξήγηση γιατί ο δεξιός κλάδος της καμπύλης (α­ντίστοιχα ο αριστερός) εφάπτεται του οριζόντιου ά­ξονα θα δοθεί αργότερα (τρίτη τάξη - παράγωγος). Το ίδιο για την «καμπυλότητα».

� , εκτός απ' τους χ, για τους οποίους ισχύει: πχ Ο Σ ' δ' ' ' συν6 = . υνεπως το πε ιο ορισμου της ειναι:

Α = � -{6κ +3 1 κ ε Ζ} .

Ισχύει: « φ(χ) = φ(χ + 6) , γα κάθε χε Α >>.

Πραγματικά:

φ (χ) = -1 + 3εφ 7 = -1 + 3εφ( 7 + π)=

-1 + 3εφ( πχ :6π )= -1 + 3εφ[:(χ + 6)] = φ(χ +6)

, για κάθε χ ε Α . Άρα η συνάρτηση αυτή είναι περιοδική με ε­

λάχιστη θετική περίοδο 6. Επομένως η μελέτη της είναι αρκετό να γίνει στο ανοικτό διάστημα (-3,3) .

Συγκροτούμε τον πίνακα τιμών1 1 : χ ο ι

πχ π - ο -6 6 πχ ../3 = 0,57 εφ- ο 6 3

φ( χ) - 1 0.71

-3

3 - 2 2 π π - -4 3 I .J3 = I, 73

2 4,19

χ � 3-πχ π· ---fi· 2 1τΧ εφ- -++cο fi

Φ(χ) �-ι-

3 χ

Η καμπύλη αν συνεχιστεί προς τα πάνω "τείνει" aσυμπτωτικά να προσεγγίσει την ευθεία χ=3. Όμοια αν συνεχίσει προς τα κάτω «τείνει)) a­συμπτωτικά να προσεγγίσει την ευθεία χ=3.

Ν α διαδίδουμε

1 1 Διαισθητικά καταλαβαίνουμε ότι: «αν χ --7 κ , τότε λχ --7 λκ >> και «αν y --7 +co και θ > Ο , τότε θy --7 οο . Αργότερα (τρίτη τάξη - όρια) θα μάθουμε ότι είναι ιδιότητες του ορίου.

ΕΥΚΛΕΙΔΗΣ Β' λ.ε. τ.l/48

Page 51: Ευκλειδης Β 41

Μαθηματικά Ύια την Β· Λυκείου

Καλίκας Ε. Σταμάτης - Βάρναλης Γ. Νίκος

Σκοπός του άρθρου αυτού είναι να βοηθήσει τον μαθητή της Α · λυκείου α) να κατανοήσει την έννοια της ομοιό­τητας, των ευθύγραμμων σχημάτων, β) να μάθει τα κριτήρια με τα οποία μπορεί να αποδείξει ότι δύο τρίγωνα είναι ό­μοια. Επίσης το άρθρο αυτό απευθύνεται και στους μαθητές της Β · λυκείου οι οποίοι χρειάζονται τις γνώσεις αυτές για να μπορούν να αποδείξουν τις μετρικές σχέσεις στα ορθογώνια τρίγωνα και τις μετρικές σχέσεις στο κύκλο.

Καταρχάς θα αναφέρουμε τον ορισμό των ό­μοιων ευθύγραμμων σχημάτων.

Ορισμός 1 : Δύο ευθύγραμμα σχήματα λέγονται όμοια, αν

έχουν τις πλευρές τους ανάλογε� και τις γωνίες που σχηματίζονται από ομόλογες1 πλευρές τους ί­σες μία προς μία.

Ορισμός 2 : Ο λόγος των ομόλογων πλευρών δύο ευθύ­

γραμμων σχημάτων λέγεται λόγος ομοιότητας αυ­τών και συμβολίζεται με λ.

Συμβολισμός: Η ομοιότητα μεταξύ δύο ευθύγραμμων σχη­

μάτων συμβολίζεται με = .

Θεώρημα 1 :

Ο λiJyoς των περιμέτρων δύο όμοιων ευθύyραμ­μων σχημάτων ισούται με το λiJyo ομοιότητάς τους.

Προσοχή! Αν δύο ευθύγραμμα σχήματα έχουν τις γωνί­

ες τους ίσες μία προς μία δεν είναι απαραίτητα ό­μοια. Για παράδειγμα αναφέρουμε δύο ορθογώνια ΑΒΓΔ, Α'Β'ΓΔ'. Ενώ οι γωνίες τους είναι ίσες (ορθές) οι πλευρές τους μπορεί να μην είναι ανά­λογες. (π.χ αν οι πλευρές του ενός είναι ΑΒ=1 και ΒΓ=2 ενώ του άλλου Α'Β '=3 και Β'Γ=8 και 1 3 ) Ό , , . - -::F- - • μως για τα τριγωνα ισχuει το παρακατω 2 8 θεώρημα:

Θεώρημα 2: Α ν δύο τρίγωνα έχουν δύο γωνίες τους ίσες

μία προς μία, τότε είναι όμοια. Θεώρημα 3: Αν δύο τρίγωνα έχουν δύο πλευρές τους ανά­

λογες μία προς μία και τις περιεχόμενες στις πλευ­ρές αυτές γωνίες ίσες, τότε είναι όμοια.

Θεώρημα 4: Αν δύο τρίγωνα έχουν τις πλευρές τους ανά­

λογες μία Προς μία, τότε είναι όμοια. Από τα παραπάνω θεωρήματα προκύ­

πτουν τα εξής πορίσματα: Πόρισμα 1 :

12 Ομόλογες πλευρές λέ-yονται οι πλευρές που βρίσκονται απέναντι από ίσες γωνίες σε δύο όμοια σχή­ματα.

Ο λόγος ομοιότητας δύο όμοιων τριγώνων εί­ναι ίσος με το λόγο δύο ομόλογων υψών τους.

Πόρισμα 2: Ο λόγος ομοιότητας δύο όμοιων τριγώνων εί­

ναι ίσος με το λόγο δύο ομόλογων διχοτόμων τους. Πόρισμα 3: Ο λόγος ομοιότητας δύο όμοιων τριγώνων εί­

ναι ίσος με το λόγο δύο ομόλογων διαμέσων τους. Γενικά: Ο λόγος ομοιότητας δύο όμοιων τριγώνων εί­

ναι ίσος με το λόγο δύο ομόλογων γραμμικών στοιχείων των τριγώνων (όχι γωνιακών). (π.χ. υ­ψών, διαμέσων, διχοτόμων, ακτίνων εγγεγραμμέ­νων κύκλων, ακτίνων περιγεγραμμένων κύκλων).

Βασικές εφαρμογές: Τα παρακάτω σχήματα τα συναντάμε συχνά

σε θεωρήματα ή ασκήσεις και έτσι καλό είναι να γνωρίζουμε τα όμοια τρίγωνα που εμφανίζονται σε αυτά και τις αναλογίες που προκύπτουν από την ομοιότητα.

Στο σχήμα 1 είναι ΔΕ//ΒΓ και τα τρίγωνα ΑΒΓ, ΑΔΕ είναι όμοια. Ομοίως και στο σχήμα 2. Όμως στα σχήματα 1 και 2 ισχύουν οι αναλογίες: ΑΔ = ΑΕ = ΔΕ (1 ) ΑΒ ΑΓ ΒΓ

Α

Β Γ Β (σχ. 1 ) (σχ. 2)

Γ

Στο σχήμα 3 οι χορδές ΑΒ και Γ Δ τέμνονται στο σημείο Ρ και ΑΡ Δ = ΒΓΡ αλλά και ΑΡΓ = ΒΡ Δ. Εδώ οι αναλογίες που ισχύουν είναι: ΑΡ

= ΡΔ = ΑΔ (2) και ΑΡ

= ΡΓ

= ΑΓ (3) ΡΓ ΡΒ ΒΓ ΡΔ ΡΒ ΒΔ

Στο σχήμα 4 το τρίγωνο ΑΒΓ είναι εγγεγραμ­μένο στον κύκλο (O,R) και το τμήμα ΑΔ είναι διάμετρος του κύκλου ενώ το τμήμα ΑΗ είναι ύ­ψος του τριγώνου ΑΒΓ. Εδώ ΑΗΓ = ΑΒΔ. Οι α-αλ , , , ΑΗ ΑΓ ΗΓ (4) ν ογιες που ισχuουν ειναι: - = - = -

ΑΒ ΑΔ ΒΔ

ΕΥΚΛΕΙΔΗΣ Β' λ.ε. τ.l/49

Page 52: Ευκλειδης Β 41

Μαθηματικά για την Β' Λυκείου

Δ (σχ. 3) (σχ. 4)

Στο σχήμα 5 το τρίγωνο ΑΒΓ είναι ορθογώνιο και ΑΔ είναι το ύψος που αντιστοιχεί στην υπο­τείνουσα ΒΓ.

Τα τρίγωνα ΑΒΓ, ΑΒΔ και ΑΓΔ είναι όμοια. Από την ομοιότητα των τριγώνων ΑΒΓ και ΑΒΔ

, ΑΒ ΑΔ ΒΔ προκυπτει: - = - = - (5). ΒΓ ΑΓ ΑΒ

Β

Δ

Γ (σχ. 5) Από την ομοιότητα των τριγώνων ΑΒΓ και

, ΑΓ ΑΔ ΔΓ ΑΓ Δ προκυπτει: -= - = - (6). ΒΓ ΑΒ ΑΓ

Από την ομοιότητα των τριγώνων ΑΒΔ και , ΑΒ ΒΔ ΑΔ ΑΓ Δ προκυπτει: - = - = - (7).

ΑΓ ΑΔ ΓΔ Στο σχήμα 6 το τετράπλευρο ΑΒΓ Δ είναι

τραπέζιο και οι διαγώνιες ΑΓ και ΒΔ τέμνονται στο σημείο Ο. Τα τρίγωνα ΑΒΟ και ΓΔΟ είναι ό­μοια. Οι αναλογίες που ισχύουν είναι: ΟΑ = ΟΒ

= ΑΒ (8). ΟΓ ΟΔ ΓΔ

Παρακάτω δίνουμε μερικές ερωτήσεις για να ελέγξετε τις γνώσεις σας στην ομοιότητα.

Ερωτήσεις κατανόησης: 1. Αν δύο τετράπλευρα έχουν τις γωνίες τους ίσες μία

προς μία, είναι όμοια; 2. Αν δύο τετράπλευρα έχουν τις πλευρές τους ανά­

λογες, είναι όμοια; 3. Αν δύο τρίγωνα είναι ίσα, είναι όμοια; Αν ναι ποιος

είναι ο λόγος ομοιότητας τους; 4. Αν δύο τρίγωνα είναι όμοια, είναι ίσα; 5. Να εξετάσετε αν είναι αληθής η παρακάτω πρότα­

ση: «Α ν μια ευθεία τέμνει δύο πλευρές τριγώνου και σχηματίζει με αυτές ένα τρίγωνο όμοιο με αυ­τό, τότε η ευθεία είναι παράλληλη με την τρίτη πλευρά του τριγώνου».

6. Α ν δύο τρίγωνα έχουν τις πλευρές τους παράλληλες μια προς μια είναι όμοια;

7. Α ν δύο τρίγωνα έχουν τις πλευρές τους κάθετες μια προς μια είναι όμοια;

8. Δύο ισόπλευρα τρίγωνα είναι πάντα όμοια;

9. Δύο ισοσκελή τρίγωνα που έχουν ίσες τις γωνίες των κορυφών τους είναι όμοια;

10. Δύο ισοσκελή τρίγωνα που έχουν ίσες τις γωνίες της βάσης τους είναι όμοια;

11. Δύο ορθογώνια τρίγωνα που έχουν μια οξεία γωνία τους ίση είναι όμοια;

12. Δύο ορθογώνια τρίγωνα που έχουν τις κάθετες πλευρές τους ανάλογες είναι όμοια;

13. Δύο ορθογώνια τρίγωνα που έχουν τον λόγο των υ­ποτεινουσών τους ίσο με τον λόγο δύο κάθετων πλευρών είναι όμοια;

Βασική άσκηση: Από το σημείο τομής Θ- των διαγωνίων τρα­

πεζίου, φέρουμε την παράλληλη προς τις βάσεις η οποία τέμνει τις μη παράλληλες πλευρές στα Ε και Ζ. Να αποδείξετε ότι ΟΕ=ΟΖ (σχήμα 7)

� Ε� Δ Γ Δ Γ (σχ. 6) (σχ. 7)

Απόδειξη: Τα τρίγωνα ΑΕΟ και ΑΓΔ είναι όμοια άρα

ΕΟ = ΑΕ (1) . ΓΔ ΑΔ

Ομοίως τα τρίγωνα ΒΟΖ και ΒΓ Δ είναι όμοια άρα ΖΟ = ΒΖ (2).

ΓΔ ΒΓ Όμως από το θεώρημα του Θαλή προκύπτει

ότι ΑΕ = ΒΖ (3). ΑΔ ΒΓ Από τις σχέσεις (1), (2) και (3) έχουμε ότι

ΕΟ ΖΟ , -= - και ετσι ΕΟ=ΖΟ. ΓΔ ΓΔ

Λυμένες Ασκήσεις Λ Λ 1 . Σε τρίγωνο ΑΒΓ είναι Β = 2 Γ . Να προε-

κτείνεται την ΑΒ κατά τμήμα ΒΔ=ΒΓ και να δείξετε ότι ΑΒΓ""' Γ ΑΔ.

Στη συνέχεια να δείξετε ότι β2 = γ( α + γ) .

Ν α αποδείξετε ότι ισχύει και το αντίστροφο. Δηλαδή αν σε τρίγωνο ΑΒΓ ισχύει β2 = γ(α + γ) Λ Λ να αποδείξετε ότι Β = 2 Γ .

Γ

ω Δ (σχ. 8)

ΕΥΚΛΕΙΔΗΣ Β' λ.ε. τ.l/50

Page 53: Ευκλειδης Β 41

Μαθηματικά yια την Β· Λυκείου

Απόδειξη: (Σχ. 8) Προεκτείνουμε την πλευρά ΑΒ κατά τμήμα

ΒΔ=ΒΓ. Το τρίγωνο ΒΓΔ είναι ισοσκελές (ΒΓ=ΒΔ) και επειδή η Β = 2ω είναι εξωτερική θα

Λ Λ ισχύει ΒΔΓ = ΒΓΔ = ώ .

Τα τρίγωνα ΑΒΓ και ΑΓ Δ είναι όμοια γιατί έ-Λ χουν δύο γωνίες τους ίσες, την Α κοινή και ΑrΒ = ΑΔΓ = ώ . 'Εiσι ΑΓ = ΒΓ = ΑΒ ή ΑΔ ΓΔ ΑΓ _β_ = � = r οπότε β2 = γ(α + γ) . α + γ ΓΔ β

(το αντίστροφο αφήνεται ως άσκηση στους μαθη-τές)

2. Αν ΑΔ, ΒΕ, ΓΖ τα ύψη ενός τριγώνου ΑΒΓ να αποδείξετε

(α) ότι τα τρίΎωνα ΑΗΕ και ΒΗΔ είναι όμοια (β) τα τρίγωνα ΗΒΖ και ΗΕΓ είναι όμοια (γ) ΗΔ·ΗΑ=ΗΒ·ΗΕ,.;ΗΓ·ΗΖ. (σχήμα 9)

Α

Β Γ (σχ. 9) Απόδειξη: Τα τρίγωνα ΑΗΕ και ΒΗΔ είναι όμοια γιατί

είναι ορθογώνια και έχουν και άλλη μία · οξεία γω-, , ' 'Ε ΗΒ ΗΔ νια τους ιση ως κατακορυφην. τσι

ΗΑ =

ΗΕ και άρα ΗΔ·ΗΑ=ΗΒ·ΗΕ. Ομοίως τα τρίγωνα ΗΒΖ και ΗΕΓ είναι όμοια και προκύπτει ότι ΗΒ·ΗΕ=ΗΓ·ΗΖ.

Τελικά ΗΔ·ΗΑ=ΗΒ·ΗΕ=ΗΓ·ΗΖ. 3. Α ν θεωρήσουμε ένα εσωτερικό σημείο

Μ του τριΎώνου ΑΒΓ και φέρουμε τρεις ευθείες ΔΕ, ΖΗ και ΘΚ που να περνάνε από το Μ και να είναι ΔΕ/ΙΒΓ, ΖΗ//ΑΓ και ΘΚ//ΑΒ να απο-δ ίξ , ΔΗ ΚΕ ΘΖ ( , 10) ε ετε οτι - + - + - = 1 . σχημα

ΑΒ ΑΓ ΒΓ Α

Β Γ (σχ. 1 0) Απόδειξη: Τα τρίγωνα ΔΗΜ, ΚΕΜ, ΜΘΖ και ΑΒΓ είναι

όμοια γιατί έχουν πλευρές παράλληλες και έτσι οι γωνίες τους είναι ίσες. Από την ομοιότητα των πα­ραπάνω τριγώνων με το ΑΒΓ προκύπτει ότι

ΔΗ = ΔΜ ( 1 ) ΚΕ =ΜΕ (2). ΑΒ ΒΓ ' ΑΓ ΒΓ

Έτσι λόγω των (1) και (2) και επειδή τα ΒΔΜΘ, ΜΕΓΖ είναι παραλληλόγραμμα, η σχέση , ΔΗ ΚΕ ΘΖ ΔΜ ΜΕ ΘΖ γινεται - + - + - = - + - + - =

ΑΒ ΑΓ ΒΓ ΒΓ ΒΓ ΒΓ = ΒΘ + ΖΓ + ΘΖ = ΒΓ = 1 .

ΒΓ ΒΓ ΒΓ ΒΓ 4. Δίνεται κύκλος κέντρου Ο και ευθεία (ε)

εξωτερική αυτού. Έστω ΑΒ διάμετρος κάθετη στην ευθεία (ε). Από το σημείο Α φέρνουμε τέ­μνουσα η οποία συναντά το κύκλο στο Γ και την (ε) στο Δ. Να αποδειχθεί ότι το γινόμενο ΑΓ·ΑΔ είναι σταθερό. (Σχολή Ικάρων 1948) (σχήμα 11)

Απόδειξη: Έστω Κ το σημείο τομής του φορέα της δια­

μέτρου ΑΒ με την ευθεία (ε). Τότε τα τρίγωνα ΑΒΓ και ΑΚΔ είναι όμοια γιατί είναι ορθογώνια

ΑΒ ΑΓ και έχουν την γωνία Α κοινή. Έτσι ΑΔ = ΑΚ ή

ΑΓ·ΑΔ.=ΑΒ·ΑΚ=σταθερό. Δ

ε (σχ. 1 1 ) 5. Σε ορθοοyώνιο τρίγωνο ΑΒΓ είναι εγγε­

γραμμένο τετράΎωνο του οποίου μια πλευρά κείται στην υποτείνουσα ΒΓ . Να αποδειχθεί ότι η πλευρά του τετραγώνου είναι μέση ανάλοοyος μεταξύ των δυο τμη�άτων της υποτείνουσας τα οποία μένουν.

Απόδειξη: (Σχ. 12) Αρκεί να δείξουμε ότι ΔΗ2=ΒΗ·ΖΓ ( 1 ) ή

ΔΗ ΖΓ , ΔΗ ΖΓ , - = - η - = - (2). Τα τριγωνα ΒΗΔ και ΒΗ ΔΗ ΒΗ ΖΕ

ΖΕΓ είναι όμοια γιατί είναι ορθογώνια και Λ Λ Λ

Β = ΖΕΓ (ως συμπληρωματικές της Γ ). Έτσι ι­σχύει η (2) και επομένως η (1).

Γ Α

Β

ΕΥΚΛΕΙΔΗΣ Β' λ.ε. τ.l/51

Page 54: Ευκλειδης Β 41

Μαθηματικά για την Β' Λυκείου

6. Ισοσκελές τρίγωνο ΑΒΓ (ΑΒ=ΑΓ) είναι εγγεγραμμένο σε κύκλο (Ο,ρ). Από την κορυφή Α φέρνουμε μια ευθεία η οποία τέμνει την ΒΓ στο Δ και τον κύκλο στο Ε. Να αποδειχθεί ότι ΑΒ2=ΑΔ·ΑΕ.

Απόδειξη: (Σχ. 13)

Αρκεί να δείξουμε ότι ΑΒ = ΑΔ (1). Τα τρί­ΑΕ ΑΒ

γωνα ΑΒΔ και ΑΒΕ είναι όμοια γιατί έχουν την Λ Λ Λ

γωνία ΒΑΔ κοινή και ΑΒΔ = ΑΕΒ (είναι ίσες Λ

με την ΑΓΒ ). Έτσι ισχύει η ( 1). Παρατήρηση: Για να αποδείξουμε αναλογίες καλό είναι να

παρατηρούμε τα τρίγωνα που σχηματίζουν τα γράμματα των aριθμητών και των παρονομαστών των κλασμάτων. Συνήθως τα τρίγωνα αυτά είναι όμοια και έτσι οδηγούμαστε στην απόδειξη των α-

ναλογιών π.χ στην άσκηση 6 από την προς απόδει­ΑΒΔ

,---J'---.,

ξ ' ΑΒ ΑΔ ' ΑΒΔ ΑΒΕ ' η σχεση ΑΕ

= ΑΒ

τα τριγωνα και ει-'---------v----ΑΒΕ

ναι όμοια. Αλλιώς παρατηρούμε τα τρίγωνα που σχηματίζουν τα γράμματα του πρώτου κλάσματος

και του δεύτερου ΑΒΕ{ ΑΒ = ΑΔ }ΑΒΔ . Η ο-ΑΕ ΑΒ

μοιότητα συνήθως των τριγώνων αυτών μας οδη­γεί στην απόδειξη των αναλογιών π. χ στην άσκηση 5 τα τρίγωνα ΒΗΔ και ΖΕΓ είναι όμοια.

Απαντήσεις στις ερωτήσεις κατανόησης. ι όχι 2. όχι 3. ναι, λ=1 4. όχι 5. όχι 6 έ­

ως 13 . ναι .

r rι ,.ΙJ4ΙΤ&ι' Αι Τ' .. ιr� MIJ' ΙΙifι/� ..,.,., Ai/. Α!.Ι!'· Ιι Αι<' � �- ... · 5 . .! rιιn· ] ιw.: 69J /Α .. .. ! Ι#.., #J' rιι _ι#Ι!� Ι!ΙJΙ!#.,

1°: θεωρούμε μη ισοσκελές τρίγωνο ΑΒΓ. Έστω ΑΜ η διάμεσός του και ΜΔ η διχοτόμος

Λ

της γωνίας ΑΜΓ και είναι ΑΔ > ΔΓ . Δείξτε ό-τι: i) Α < 90° . ii) Το τετράπλευρο ΑΔΜΒ είναι εγγράψιμο σε κύκλο αν και μόνο αν

2 α2 β = - + αμα .

2 Απάντηση

Β Μ Γ Σχ. 1 i) Από το θεώρημα της εσωτερικής διχοτό-

, ΑΜΓ , ΑΔ ΑΜ , μου στο τριγωνο εχουμε ΔΓ

= ΜΓ

η ισο-

δύναμα ΑΔ = 2μα > 1 ΔΓ α

2βz + 2γz -αz > αz ή ή α2 < β2 + γ2 , απ' όπου συμπεραίνουμε ότι Α < 90° .

ii) Επειδή η ΜΔ είναι διχοτόμος στο τρίγωνο β - �

ΑΜΓ, έχουμε ΔΓ = --2- . Το τετράπλευρο α - + μ 2 α

του Σπύρου Γιαννακόπουλου ΑΔΜΒ είναι εγγράψιμο σε κύκλο αν και μόνο αν ισχύει Γ Δ · Γ Α = ΓΜ · ΓΒ ή ισοδύναμα

β -�2 α (α ) α2

-α--β =2α ή βz = α

2+ μ

α ή βz =

τ+ αμ

α.

l + μα

2°: Τρίγωνο ΑΒΓ είναι εγγεγραμμένο σε κύκλο (O,R). Με κέντρο το Α και ακτίνα το ύ­ψος ΑΔ του τριγώνου γράφουμε κύκλο που τέ­μνει τον αρχικό κύκλο στα Μ, Ν. Αν η ΜΝ τέ­μνει τις πλευρές ΑΒ, ΑΓ στα σημεία Ε και Ζ α­ντίστοιχα, δείξτε ότι ΔΕ .1 ΑΒ, ΔΖ .1 ΑΓ.

Απάντηση

.......

Σχ.2

Οι ΜΝ, ΑΓ είναι χορ­δές του κύκλου (O,R) και τέμνονται στο Ζ . Άρα ισχύει ΖΑ · ΖΓ = ΖΜ · ΖΝ ( 1). Η ΜΝ είναι χορδή και του κύκλου (Α,ΑΔ),

οπότε θα ισχύει ΖΜ · ΖΝ = ΑΔ2 - ΑΖ2 (2). Απ' την άλλη έχουμε ΑΖ·ΑΓ=ΑΖ(ΑΖ+ΖΓ)=

(1 ),(2) = ΑΖ2 +ΑΖ · ΖΓ = ΑΖ2 + ΑΔ2 - ΑΖ2 = ΑΔ2 •

Άρα ΑΔ2 = ΑΖ · ΑΓ (3). Αν υποθέσουμε ότι η προβολή του Δ πάνω

στην ΑΓ είναι το Ζ' (το z· είναι εσωτερικό σημείο του ΑΓ). Επειδή το τρίγωνο ΑΔΓ είναι ορθογώνιο, θα έχουμε ΑΔ2 = ΑΖ' · ΑΓ (4). Από τις (3), (4)

ΕΥΚΛΕΙΔΗΣ Β' λ.ε. τ.l/52

Page 55: Ευκλειδης Β 41

Μαθηματικά για την Β' Λυκείου

προκύπτει ότι ΑΖ · ΑΓ = ΑΖ' · ΑΓ <::::> ΑΖ = ΑΖ' , που σημαίνει ότι τα σημεία Ζ, z· συμπίπτουν.

Άρα ΔΖ _l_ ΑΓ . Όμοια εργαζόμαστε για ΔΕ · ΑΒ . 3°: Θεωρούμε τρίγωνο ΑΒΓ εγγεγραμμένο

σε κύκλο (O,R). Η διχοτόμος ΑΔ του τριγώνου τέμνει τον κύκλο στο Ε. Δείξτε ότι ΓΕ2 = ΕΔ · ΕΑ .

Απάντηση (Σχ. 3) Επειδή η ΑΔ είναι διχοτόμος, το Ε είναι το

μέσο του τόξου ΒΓ και ΟΕ _l ΒΓ . Φέρνουμε τη διάμετρο ΕΖ και από το ορθογώνιο τρίγωνο ΓΕΖ παίρνουμε ΓΕ2 = ΕΜ · ΕΖ (1).

Το τετράπλευρο ΑΔΜΖ είναι εγγράψιμο γιατί Λ Λ

ΔΑΖ+ ΔΜΖ = 90° + 90° = 1 80° , οπότε έχουμε ΕΜ · ΕΖ = ΕΔ · ΕΑ (2). Από τις (1), (2) προκύπτει ότι ΓΕ2 = ΕΔ · ΕΑ .

y ;

Δ� Γ Α Ο Κ Β Ε

Σχ. 4

4°: Δίνεται ημικύκλιο (O,R) διαμέτρου ΑΒ και σημείο Γ στην προέκταση της ΒΑ έτσι ώστε ΟΓ = d . Στο Γ υψώνουμε κάθετο πάνω στην ΑΒ, την Γy. Να προσδιορίσετε σημείο Μ του η­μικυκλίου έτσι ώστε η εφαπτομένη του ημικυ­κλίου στο σημείο αυτό να τέμνει την Γy και την προέκταση της ΑΒ στα σημεία Δ, Ε αντίστοιχα και να ισχύει ΜΔ = ΜΕ .

Απάντηση (Σχ. 4) Ανάλυση: Υποθέτουμε ότι το σημείο Μ προσδιορίστηκε

και η ΔΜΕ είναι εφαπτομένη του ημικυκλίου με ΜΔ = ΜΕ .

Θέτουμε ΟΕ = χ . Φέρνουμε το ΜΚ _l ΑΒ και επειδή ΜΚ 11 ΓΔ και Μ μέσο του ΕΔ το Κ εί­ναι μέσο του ΓΕ. Από το ορθογώνιο τρίγωνο ΜΟΕ έχουμε ΜΕ2 = ΚΕ · χ <=> ΜΕ2 = d + χ χ . Ακόμα έ-

2 χουμε ΜΕ2 = 0Ε2 - R2 = χ2 -R2 . Πρέπει d + x 2 2 2 2 --χ = χ -R <::::> χ - dx - 2R = Ο . Δεδομέ-

2 νου ότι δεκτή λύση της εξίσωσης είναι η θετική,

, d+ .Jd2 +8R2 παιρνουμε χ = . Άρα το Ε προσδιο-

2

ρίζεται. Σύνθεση: Στην προέκταση του ΑΒ προς το Β

παίρνουμε σημείο Ε έτσι, ώστε να απέχει από το Ο . d + .Jd2 + 8R2

Α . Ε . αποσταση . πο το κατασκευα-2

ζουμε την εφαπτομένη του ημικυκλίου (κατασκευή γνωστή από την Α' Λυκείου) και έστω Μ το ση­μείο επαφής και Δ το σημείο τομής της εφαπτομέ­νης με την Γy. Το Μ είναι το ζητούμενο σημείο.

Απόδειξη: Φέρνουμε την ΜΚ _l ΑΒ . Από το ορθογώνιο τρίγωνο ΜΟΕ έχουμε

2R2 ��+8R2 -d ΟΜ2 =0Ε·ΟΚ<::::>ΟΚ -ι=, == d+νd2 +8R2 4

ΚΕ = ΟΕ - ΟΚ = d+ .Jd2 +8R2 .Jd2 - 8R2 -d =

3d+ .Jd2 + 8R2 ΓΕ = 4 2

2 4

Άρα το Κ είναι μέσο του ΓΕ. Επειδή ΚΜ 1 1 Γ Δ το Μ είναι μέσο του ΔΕ.

Διερεύνηση: Το Μ ορίζεται όταν

d+�d2 +8R2 .. --OE>R<=> >R <::::>d+..fd2 +8R2 >2R <::::>

2 .Jd2 + 8R 2 > 2R -d , που ισχύει αφού • για d � 2R είναι προφανής • για d > 2R είναι ισοδύναμη με την

4R2 +4Rd > O Συνεπώς το πρόβλημα έχει πάντα μία λύση. 5°: α) Έστω ΑΒΓ τρίγωνο με ΑΒ '* ΑΓ ,

ΑΔ το ύψος του και Δ εσωτερικό της ΒΓ. Αν ι-, ΑΒ2 ΒΔ δ ίξ , , , σχυει

ΑΓ2 = ΔΓ

ε τε οτι το τριγωνο ειναι

ορθογώνιο στην κορυφή Α. β) Θεωρούμε τρίγωνο ΑΒΓ με ΑΒ < ΑΓ και

εσωτερική διχοτόμο την ΑΔ. Έστω Ε1 , Ε2 τα εμ­βαδά των τριγώνων ΑΒΔ και ΑΔΓ αντίστοιχα. i) Να δείξτε ότι ΔΓ · Ε1 = ΒΔ · Ε2 • ii) Στο Δ πάνω στην ΒΓ υψώνουμε κάθετο και παίρνουμε πάνω στην κάθετο σημείο Μ τέ-τοιο, ώστε να ισχύει ΜΒ · .JE; = ΜΓ · .JE.. Να δείξτε ότι το τρίγωνο ΜΒΓ είναι ορθογώνω.

Απάντηση

α) (Σχ. 5) Θέτουμε ΑΒ: = ΒΔ = λ ( 1) και εί-ΑΓ ΔΓ

ναι Ο < λ '* 1 . Από την (1) έχουμε ΑΒ2 = λΑΓ2 <=> ΑΒ2 + ΑΓ2 = (λ + 1)ΑΓ2 (2).

ΕΥΚΛΕΙΔΗΣ Β' λ.ε. τ.l/53

Page 56: Ευκλειδης Β 41

Μαθηματικά -yια την Β· Λυκείου

Επίσης ΒΔ = λΔΓ <==> ΒΔ + ΔΓ = (λ+ 1) ΔΓ <==> ΒΓ = (λ + 1)ΔΓ <==> ΒΓ2 = (λ + 1)2 ΔΓ2 (3).

Έχουμε Μ =ΑΒ2 -ΔΒ2 και Μ =ΑΓ2 -Δr , οπότε ΑΒ2 -ΔΒ2 = ΑΓ2 - ΔΓ2 <==> λΑΓ2 -λ2ΔΓ2 = ΑΓ2 -ΔΓ2 <==> λ2 ΔΓ2 -ΔΓ2 = λΑΓ2 -ΑΓ2 <==>

(λ2 - 1)ΔΓ2 = (λ- 1)ΑΓ2 <==> ΔΓ2 = -1-ΑΓ2 . Άρα λ + 1

η (3) γράφεται ΒΓ2 = (λ + 1)ΑΓ2 (4). Από τις (2), ( 4) παίρνουμε ΑΒ2 + ΑΓ2 = ΒΓ2 , που σημαίνει ότι

Λ ΒΑΓ = 90° .

Γ

Σχ. 5 Σχ.6 β) (Σχ. 6)

Γ

i) Τα τρίγωνα ΑΒΔ και ΑΔΓ έχουν το ί-διο ύψος ΑΚ, άρα !Ι_ = ΒΔ

<==> ΔΓ · Ε1 = ΒΔ · Ε2 . Ε2 ΔΓ

ii) ΜΒ · .jE; = ΜΓ · JE: <==> ΜΒ = � ΜΓ νΕ2

ΜΒ2 Ε i) ΜΒ2 ΒΔ ή ΜΓ2 =t<==> ΜΓ2 = ΔΓ

. 2

Σύμφωνα με το α) η τελευταία σχέση μας ε-Λ

ξασφαλίζει ότι ΒΜΓ = 90° . 6°: Δίνεται τρίΎωνο ΑΒΓ με ΑΒ < ΑΓ και

τα ύψη του ΑΔ, ΒΕ. Αν το ορθόκεντρο του τρι­Ύώνου είναι το μέσο Η της ΑΔ και το μέσο της ΑΓ είναι το Ζ, τότε να δείξετε ότι 1.) ΗΓz HAz Ε<ΗΑΕ> ΑΓz .. ) Ε 1 Ε - = -- .U (ΗΒΖ) = - (ΑΔΓ) • Ε(ΗΒ� 4

Απάντηση ( σχ. 7) i) Τα τρίγωνα ΔΒΗ, ΕΑΗ έχουν

Λ Λ ΒΗΔ = ΑΗΕ , οπότε

Ε(ΗΑΕ) = ΗΕ · ΗΑ <==> Ε(ΗΑΕ) = ΗΕ ( 1). Ε(ΗΒΔ) ΗΒ . ΗΔ Ε(ΗΒΔ) ΗΒ

Στο τρίγωνο ΗΑΓ από το 2° θεώρημα των διαμέσων έχουμε ΗΓ2 - ΗΑ 2 = 2ΑΓ · ΕΖ (2). Στο τρίγωνο ΑΔΓ το τμήμα ΗΖ ενώνει τα μέσα δύο πλευρών του, άρα ΗΖ 1 1 ΒΓ , οπότε από το θεώρη­μα του Θαλή έχουμε ΕΖ = ΕΗ <==> 2ΕΖ = ΕΗ <==> 2ΕΖ � Ε<ΗΑΕ) (3). ΖΓ ΗΒ ΑΓ ΗΒ ΑΓ Ε<ΗΒΔ)

2ΕΖ ω Ε (2) <==> ΗΓ2 -ΗΑ 2 = --ΑΓ2 = (ΗΑΕ) ΑΓ2 <==> ΑΓ Ε(ΗΒΔ)

Ε ΗΓ2 - ΗΑ2 = (ΗΑΕ) ΑΓ2 Ε(ΗΒΔ)

ii) Τα τρίγωνα ΗΒΖ και ΓΗΖ έχουν κοινή βάση την ΗΖ και οι κορυφές τους Β, Γ βρίσκονται σε παράλληλο προς την ΗΖ. Άρα Ε(ΒΗΖ) = E<rHz) (4). Όμως η ΗΖ είναι διάμεσος στο τρίγωνο ΗΑΓ, οπότε Ε<ΗΖΓ> = .!_Ε(ΗΑΓ) (5). Επίσης η ΗΓ είναι

2 διάμεσος στο τρίγωνο ΑΔΓ και άρα είναι

1 Ε(ΗΑΓ) = -Ε<ΑΔΓ> . Συνεπώς η (5) γράφεται

2 1 (4) 1

Ε(ΗΖΓ) = 4 Ε(ΑΔΓ) <==> Ε(ΒΗΖ) = 4 Ε(ΑΔΓ) . Σ

Α

Γ Σχ. 7 Σχ.8

7°: Έστω ένα τρίΎωνο ΑΒΓ επε-yραμμένο σε κύκλο (O,R). Προεκτείνουμε την πλευρά ΒΑ και παίρνουμε στην προέκταση τμήμα ΑΣ. Η ΣΓ τέμνει τον κύκλο στο Δ. Ονομάζουμε ΕΑΣr = Ε0 ΕΣΒΓ = Ε2 και ΕΑΒΓ = Ε . Αν

ΣΔ · ΣΓ = .!ΑΒ2 , να δείξτε ότι Ε = !:JE: .JE; . 4 3

Απάντηση (Σχ. 8)

Τα τρίγωνα ΑΣΓ και ΑΒΓ έχουν Λ Λ Ε ΑΣ · ΑΓ Ε ΑΣ Α+ΓΑΣ = 180° οπότε -1 = ή _ι = -

' Ε ΑΒ · ΑΓ Ε ΑΒ ( 1 ). Επίσης τα τρίγωνα ΣΒΓ και ΑΒΓ έχουν κοινή

Β . Ε2 ΒΣ · ΒΓ Ε2 ΒΣ γωνία τη , οποτε Ε= ΒΑ . ΒΓ <==>Ε= ΑΒ

(2). Πολλαπλασιάζοντας τις (1 ), (2) κατά μέλη , Ε Ε ΑΣ · ΒΣ παιρνουμε -1 -2 = (3).

Ε2 ΑΒ2 Οι ΣΑΒ, ΣΔΓ είναι τέμνουσες του κύκλου, ο-

πότε έχουμε ΑΣ · ΒΣ = ΣΔ · ΣΓ ή ΑΣ · ΒΣ = �ΑΒ2 (4).

4 Η (3) σύμφωνα με την (4) γράφεται Ε1�2 = �

Ε 4 ή Ε2 =_±Ε1Ε2 ή Ε = �JE: .jE; . 9 3

ΕΥΚΛΕΙΔΗΣ Β' λ.ε. τ.l/54

Page 57: Ευκλειδης Β 41

Μαθηματικά για την Β' Λυκείου

� fJ w. H••ill•• 114-S r1 ονίιι•ι �ΙΑΙΙΠΑΑ (νεετ•ιιJ Η στενή αλληλεπίδραση των Μαθηματικών και

της Φυσικής γέννησε την έννοια του διανύσματος. Έτσι διακρίνουμε τα φυσικά μεγέθη σε: α) μονόμετρα, όπως είναι το μήκος, το πλάτος, το

εμβαδόν, ο όγκος, η μάζα κ.ά. και β) διανυσματικά, όπως είναι η δύναμη, η ταχύτη­

τα, το βάρος, η μαγνητική επαγωγή κ.ά. Πριν από 120 περίπου χρόνια δημιουργήθηκε η

σύγχρονη θεωρία του Διανυσματικού Λογισμού. Το διάνυσμα είναι στοιχείο του "διανυσματικού

χώρου". Ένα διάνυσμα καθορίζεται αν γνωρίζουμε: i) το φορέα, δηλαδή την ευθεία πάνω στην οποία

βρίσκεται, ίί) το μέτρο ή μήκος του ίίί) τη φορά και iv) σε προβλήματα της Φυσικής και της Γεωμετρί­

ας ορίζουμε το διάνυσμα με αρχή (ή σημείο. εφαρμο­γής) και τέλος.

Σε κάθε βιβλίο μπορούμε να βρούμε τη σχετική θεωρία για τα διανύσματα.

Με τη θεωρία του διανυσματικού λογισμού μπορού­με να απαντήσουμε σε προβλήματα όπως: πότε τρία ση­μεία Α, Β, Γ είναι συνευθειακά, πότε μία ευθεία είναι πα­ράλληλη ή κάθετη σε άλλη ευθεία, πότε τρεις ή περισσό­τερες ευθείες διέρχονται από το ίδιο σημείο, ποιο είναι το εμβαδόν ενός τριγώνου, ποια είναι η απόσταση ενός ση­μείου από μία ευθεία, ποια η εξίσωση της εφαπτομένης σε σημείο Α ενός κύκλου, κ.ά.

Με τις πιο κάτω Ερωτήσεις, Ασκήσεις και Προ­βλήματα θα προσπαθήσουμε να δώσουμε σύντομα, α­πλά και σε βάθος τα βασικά που πρέπει να γνωρίζουν οι μαθητές της Β' Λυκείου.

Προτάσεις που είναι αληθείς - -1) Α ν ΑΒ = Γ Δ , τότε τα ευθύγραμμα τμήματα

ΑΔ, ΒΓ έχουν κοινό μέσο και αντιστρόφως. - -2) Α ν Μ το μέσο του ΑΒ τότε ΑΜ = ΜΒ και α-

ντιστρόφως. 3) Για τυχαίο Ο σημείο το ΑΒ γράφεται - - -

ΑΒ = ΟΒ -ΟΑ . 4) Α ν α παράλληλο στο β :;t: Ο , τότε α= λβ ,

λ ε JR και αντιστρόφως. 5) Αν α /'/' β , τότε αβ = lαl jβj . 6) Αν α /'ι/ β ' τότε αβ = -ΙαΙ iβi . 7) α(β+ Ύ) = αβ+αγ 8) -2 ι -ι2 2 2 • _ <-- _ ) α = α = α1 + α2 οπου α = α1 ,α2 • 9) Αν α = β , τότε αγ = βγ 10) Αν α/'/' β , τότε Ια+ βi = lαl + jβj 11) Αν α l. β , τότε Ια+βj = lα-βj και αντιστρόφως

του Παναγιώτη Π. Χριστόπουλου

12) αβ=βα=α·προβαβ=β ·προ!\α όπου α, β :;t: δ . Προτάσεις όχι αληθείς (προσοχή: δεν ισχύ­

ουν πάντοτε)

1 ) α(βγ) = (αβ)γ 2) jαβj = ΙαΙ jβj (γενικά ισχύει: ΙαΙ jβj � jαβj ) 3) (αβ)2 = α2 . β2 (ισχύει μόνο αν αιι β ) 4) αγ = βγ ( γ :;t: ο), τότε α = β (δηλαδή δεν ισχύ­

ει η διαγραφή). 5) Δεν ορίζονται οι δυνάμεις α3 , α4 , • • • 6) Δεν ορίζεται η διαίρεση α : β

Μεθοδολογία 1) Αν θέλουμε να αποδείξουμε μία ισότητα διανυσμά­

των, τα αναλύουμε σε κοινά γνωστά δtανύσματα ή θεωρούμε τυχαίο σημείο, συνήθως όποιο αναφέρεται τις περισσότερες φορές στο πρόβλημα, και τα ανσJαι­ουμε σε διανύσματα θέσεως ή χρησιμοποιούμε το μέ­σο ευθυγράμμων τμημάτων ή τη διάμεσο τριγώνου.

2) Όταν θέλουμε να υπολογίσουμε διανύσματα ή σyj.­σεις μεταξύ διανυσμάτων, τότε ψάχνουμε για κάθε­τα ή παράλληλα διανύσματα, πολλές φορές πολλα­πλασιάζουμε με κάποιο διάνυσμα και εκμεταλλευ­όμαστε τις ιδιότητες του εσωτερικού γινομένου.

3) Για να δείξουμε ότι τρία σημεία Α, Β, Γ είναι συ­νευθειακά, αποδεικνύουμε ότι δύο από τα διανύ-- - -σματα ΑΒ , ΑΓ, ΒΓ είναι συγγραμμικά (π.χ. - -ΑΒ = λΑΓ , λε JR ).

4) Όταν ζητείται να αναλύσουμε ένα διάνυσμα ii σε συνιστώσες με διεύθυνσεις γνωστών διανυσμάτων υ , ν . ί) Με κ, λ ε JR , γράφουμε α = κίi + λv . ίί) Εκμεταλευόμαστε τη σχέση κατάλληλα και

καταλήγουμε σε σύστημα με αγνώστους τους κ, λ.

5) Σε πολλά προβλήματα πρέπει να εφαρμόσουμε ι­διότητες της Ευκλείδειας Γεωμετρίας . .

6) Για προβλήματα Γεωμετρικών Τόπων ακολουθού­με δύο τρόπους ί) το διανυσματικό τρόπο και ίί) με συντεταγμένες. Στην πρώτη περίπτωση μετασχημα­τίζουμε την ιδιότητα της εφωνηθέντος προβλήμα­τος έτσι ώστε να πάρει τη μορφή «το σημείο του γ.τ. να απέχει σταθερή απόσταση από ένα σταθερό σημείο» (κύκλος).

Δύο διανύσματα είναι συγyραμμικά δηλ. α 1 1 β Όταν: i) Έχουν την ίδια διεύθυνση, δηλ. βρίσκονται

πάνω στην ίδια ευθεία ή σε παράλληλες ευθείες.

ii) Ι α+βi = ΙαΙ + iβl ή la+ βi = I ΙαΙ - iβi l ΕΥΚΛΕΙΔΗΣ Β' λ.ε. τ.l/SS

Page 58: Ευκλειδης Β 41

Μαθηματικά yια την Β' Λυκείου

iii) α = λβ το μηδενικό διάνυσμα θεωρείται συγγραμμικό όλων.

iν) det (α,β) = O ν) jα . βj = ΙαΙ jβj νi) Α ν λa = λβ όταν τα α , β δεν είναι πα-

ράλληλα στον άξονα y'y . Δύο διανύσματα είναι ομόρροπα δηλ. α/'/β Όταν: i) Έχουν ίδια διεύθυνση και ίδια φορά ii) la+ βj = ιaι + lβl iii) α = λβ με λ > ο iν) α - β = ΙαΙ jβj Ένα ίσον που μας δίνει "πολλά".

{ΙΔΙΑ ΔΙΕΥΘΥΝΣΗ i) α = β <=> ΙΔΙΑ ΦΟΡ Α

ΙΣΑ ΜΕΤΡΑ ii) Α ν α = ( α1 , α2 } , β = (β1 , β2 } , τότε: α = β <=> {αι = β ι αz = βz Ερώτηση Α: Ισχύουν οι γνωστές ταυτότητες

με διανύσματα, όπως (α+ β )2 = α2 + 2αβ + β2 ; Ερώτηση Β: Ισχύει η ισότητα

I προ β β αι = I αι συν (α)) ; Ερώτηση Γ: Αν ε ευθεία και α , β μη μηδε-

νικά διανύσματα ισχύει η ισότητα προβε (α+β) = προβ.α+ προβεβ ;

Ερώτηση Δ. Α ν αβ = αγ τότε α l_ (β -γ) ; Άσκηση lη

Να δείξετε ότι (λii)β =α(λβ) =λ(αβ) , λε IR . Απόδειξη

i) Αν λ = Ο , ισχύει (Οα)β = ο (αβ) ii) Αν λ * Ο , έχουμε

(λii)β = lλiil jβj συν (λa, β) =

= ΙλΙ ΙαΙ jβj συν(�,β} = αν λ > ο , λlαΙ jβj συν (α)}= λ( αβ) αν λ < Ο , -λlαΙ jβj [ -συν (a)}] = λ(α- β) διότι (λα Λ β) = (α Λ β) αν λ > Ο και (λii Λ β)= π- (αΛ β) αν λ < Ο δηλαδή

(λα) β = λ( αβ) ομοίως α(λβ) = λ( αβ)

Γενικεύοντας τώρα τη σχέση έχουμε:

(λii)(μβ) = λμ( αβ) Άσκηση 2η

Αν α = (3,-ι) , β = (6,-2) τότε να δείξετε ότι

α/"/" β Απόδειξη Πράγματι έχουμε β = 2 · ( 3, -ι) = 2α επειδή

2 > Ο=> β/'/'α ή α - β = (3,-ι)(6,-2) = 3 · 6+ (-0(-2) = ι s+2 = 20 ΙαΙ - Iβl = .../9 + ι · .../36 + 4 = .JϊΟ_ . .J40 = .../400 = 20 άρα α · β = ΙαΙ - Iβl <=> α/"/" β

Άσκηση 3η Αν τα σημεία Α, Β, Γ είναι συνευθειακά, τότε

για κάθε σημείο Ο ισχύει:

i) (ΟΒ-ΟΓ)= κ(ΟΑ-ΟΓ) , κε IR ii) λΟΑ+μΟΒ = (λ+μ)ΟΓ , λ,με R

Άσκηση 4η Αν αβ * -ι και χ + (χα)β = γ , να βρεθεί το χ . Απόδειξη Πολλαπλασιάζω την ισότητα με α και έχω

[χ + <χα)β]α = γα δηλαδή χα+ (χα)βα = γα χα (ι + αβ) = γα <=> χα = γ�_

ι + αβ με αντικατάσταση στην αρχική

_ γαβ- - · _ _ γα -

β χ +-----==- · = γ εχουμε χ = γ------==- · ι + αβ ι+αβ

Άσκηση 5η Αν β * δ και α = αι + α2 με αι 1 1 β , α2 l_ β

δ ίξ . _ αββ- _ _ αβ

β-ε τε οτι α1 = jβj2 και α2 = α- jβj2 • Άσκηση 6η

Δείξτε ότι jα + βj � lαl + jβj . Πότε ισχύει η ωότητα ; Άσκηση 7η Α ν α , β μη μηδενικά διανύσματα και ισχύει

Ι α+ βj + Ι α-βj = ι , τότε να δείξετε ότι: αβ � .!.. . 4 Και ακόμη ότι a l_ β με την επιπλέον υπόθε-

ση ότι α2 +β2 =.!.. . 4 Απόδειξη (εργαζόμαστε για το πρώτο το

δεύτερο αφήνεται ως άσκηση)

Ια+βl = ι-la-βj <=> la+�2 =(ι-la-�)2 <=> ii2 +β2 + 2iiβ = ι+ la-βl2 -2 Ιa-βl <=>

ΕΥΚΛΕΙΔΗΣ Β' λ.ε. τ.Ι/56

Page 59: Ευκλειδης Β 41

Μαθηματικά για την Β' Λυκείου

α2 + β2 + 2αβ = 1 + α2 +β2 -2αβ -2 Ια-βl <=> 2Ια-βl 7= ι - 4αβ � ο άρα αβ ::;� Άσκηση 8η Στο τρίγωνο ΑΒΓ με Α(Ο,2), Β(2,2),

Γ ( 3 + J3, 3 + J3) να υπολογιστούν οι γωνίες του. Άσκηση 9η Να επαληθεύσετε όλες τις μετρικές σχέσεις

της Γεωμετρίας που ισχύουν σε ορθογώνιο τρίγω­νο με διανυσματικό λογισμό.

Άσκηση ιοη Για τα μοναδιαία διανύσματα α ' β ' γ να α­

ποδείξετε ότι: i) Αν αβ + βγ + γα+ 1 = Ο τότε δύο από αυτά

είναι αντίθετα ii) Αν αβ +βγ = 2 τότε α = β = γ iii) Ισχύει Ια+ βl = 2 lσυν�l · θ = (α:β) iν) Ισχύει la-βl = 2 1ημ�� · θ = (α:β) Άσκηση llη Δίνονται δύο μη μηδενικά διανύσματα α , β .

Α ν υπάρχει λ Ε JR ' τέτοιος ώστε α+ λβ = ι ' όπου

ϊ μοναδιαίο διάνυσμα, να δείξετε ότι i) ιαι ::; -1- ' ημφ όπου φ γωνία των α , β , ii) το τρίγωνο με πλευρές

τα α ' β με κοινή αρχή το ο έχει εμβαδόν ::; ιβι _ 2 Άσκηση 12η Αν για τα α , β ισχύουν lβl = 2 lαl και

lα +βl = lαl , δείξτε ότι a και β είναι αντίρροπα . Λύση: Αρκεί α - β = -ΙαΙ iβl . Από Ιa+� =I� � α+β)2 =fi <=> α2 + β2 + 2αβ = αz <=> 2αβ = -ιβι -

2iiβ1��-ι�2}�α-β=-ισJ ιβι άρα a /ι/ β .

αλλά � =2lσJ Άσκηση 13η Αν Α ( 1 , 1 ) Β (3,2) Γ (2,3), προσδιορίστε Δ έ­

τσι ώστε το ΑΒΔΓ να είναι παραλληλόγραμμο.

Απάντηση: Αρκεί ΓΔ=ΑΒ (οπότε ΓΔ = ΑΒ και ΓΔ//ΑΒ) Έστω Δ (χ, y) . Έχω ΑΒ = ( 3 - 1, 2 - 1) δηλ.

ΑΒ = ( 2, 1) , Γ Δ = (χ -2, y- 3) Άρα �=� :�}<=> � : :}<=> Δ (4, 4) Άσκηση 14η Αν έχουμε το τρίγωνο ΑΒΓ και ισχύει

(2+κ)ΑΒ+ ΑΓ +(κ-λ)ΒΓ =0 , υπολογίστε τα κ, λ. Απόδειξη: (2 + κ)ΑΒ+ΑΓ + (κ-λ) (ΑΓ -ΑΒ) = 0 (2 + κ + λ-κ)ΑΒ+ (1 +κ -λ)ΑΓ =0 2 + λ = Ο } {λ = -2 l + κ-λ= Ο <=> κ = -3 Άσκηση 15η Αν Α (0,3), Β (-2, 1 ), Γ (2.J3, ι) οι κορυφές

τριγώνου ΑΒΓ, υπολογίστε: - - Λ

i) ΑΒ · ΑΓ ii) τη γωνία ΑΓΒ iii) ΑΜ , IAMI , ΑΚ όπου Μ μέσον του ΒΓ

και Κ το βαρύκεντρο του ΑΒΓ. Άσκηση 16η Αν IPI = 1 , li:il = .J2 και η γωνία των p , q εί­

ναι 45°, να βρείτε τη γωνία των διανυσμάτων p - q , q .

Άσκηση 17η Έστω lαl = 2 , lβl = 5 και η γωνία των α , β

, 2π Α -δ 5- 4β- λ , , ει ναι - . ν = α- να υπο ογιστει το μετρο 3 του δ .

Άσκηση 18η Αν ιαι =.J2 ' lβl =.J3 ' ΙγΙ = 2 και

(α: β) = (β: γ) = � και α, β, γ συνοπτικά να βρεθεί 4 το μέτρο του 2ii-3β+γ .

Άσκηση 19η Αν ΙαΙ = Iβi = ΙγΙ = 2 και α+β+γ = Ο , να βρε­

θεί το μέτρο του 2ii + 3β +γ . Άσκηση 20η Έστω a , β διανύσματα του επιπέδου με

Ιαl = 2 , lβl = 3 και ·(a:β) = 60° . Προσδιορίστε το χ Ε JR στις παρακάτω περι­

πτώσεις: i) (2a + 3β)( α- χβ) = -2 ii) 2α+ 3β .L a - χβ .

Άσκηση 21η Δίνονται τα διανύσματα a = (10, 2) ,

β = (-1, 4) . Υπολογίστε:

ΕΥΚΛΕΙΔΗΣ Β' λ.ε. τ.l/57

Page 60: Ευκλειδης Β 41

Μαθηματικά για την Β' Λυκείου

i) προββ (a) ' προβ(ί (β) ' προβ(ί ( a- β) ii) τις γωνίες των διανυσμάτων a +β και a-β με τον άξονα χ'χ .

Άσκηση 22η Για τα διανύσματα a ' β ' γ του επιπέδου ι-

σχύουν: ίi+β+γ = 0 και 3 lal = 4lβl = 12 1γ1 , να α­ποδειχτεί ότι:

i) a 11 β και β 11 γ . Άσκηση 23η Έστω ένα ορθογώνιο και ισοσκελές τρίγωνο

ΑΒΓ με IABI = IAΓI = 5 . Βρείτε τα σημεία Μ του επιπέδου που ικανοποιούν τις σχέσεις:

Άσκηση 24η Α ν στο τρίγωνο ΑΒΓ ισχύει -2 -2 -2 ΒΓ + ΑΓ = 5ΑΒ , να αποδειχτεί ότι οι διάμεσοι ΑΔ και ΒΕ του τριγώνου τέμνονται κάθετα.

Β

Απόδειξη: (Σχ. 1) Α Έστω ΑΔ, ΒΕ διάμεσοι

του ΑΒΓ. Έστω Κ το κέντρο βάρους του τρι­γώνου ΑΒΓ.

r Θέτουμε ΚΔ = χ και Σχ. 1 ΚΕ=y .

Τότε είναι ΑΚ = 2χ και ΒΚ = 2y . Από τη δοθείσα σχέση έχουμε: --2 --2 -2 -2 -2 -2 ΒΓ +ΑΓ =5ΑΒ ή 4ΒΔ +4ΑΕ =5ΑΒ ή 4(x + 2y)2 +4 (y+2x)2 = 5 (2x -2y)2 ή 4(χ2 + 4xy+ 4y2 ) +4 (y2 +4xy+ 4x2 ) = = 20(x2 -2xy+y2 ) ή xy = O ή ΒΕ _l_ ΑΔ .

Άλλη Λύση - - 1 - - 1 - ­Αρκεί Μ·ΒΕ=Ο=>--(ΑΒ+ΑΙ)-(ΒΑ+ΒΙ)=Ο� 2 2

(ΑΔ ·ΑΓ)(-ΑΒ +ΑΓ -ΑΒ) =Ο � � (ΑΔ · ΑΓ)(ΑΓ- 2ΑΒ) = Ο� - - -2 -2 - -(ΑΒ ·ΑΓ-2ΑΒ +ΑΓ -2ΑΒ ·ΑΓ = Ο � -2 -2 - - · � ΑΓ -2ΑΒ -ΑΒ · ΑΓ = Ο αληθές διότι: Bf +ff =5ΑΒ2 �(ΑΓ-ΑΒ)2 +ΑΓ2 =5ΑΒ2 � -2 -2 -2 - - -2 ΑΓ + ΑΒ +ΑΓ -2ΑΒ ·ΑΓ = 5ΑΒ � -2 -2 - -2ΑΓ -4ΑΒ = 2ΑΒ ·ΑΓ � -2 -2 - -ΑΓ - 2ΑΒ -ΑΒ ·ΑΓ = Ο

Άσκηση 25η α) Αν a , β διανύσματα του καρτεσιανού εm-

πέδου, να αποδειχτεί ότι: la · βl ::;; lϊil · lβl . Πότε ι­σχύει η ισότης;

β) Δίνονται τα διανύσματα a = (χ, y) και β = (3, 4) .

i) Υπολογίστε το εσωτερικό γινόμενο a · β ii) Α ν χ 2 + y2 = 49 να βρείτε το μέγιστο και

το ελάχιστο της παράστασης A= 3x + 4y καθώς και τις τιμές των χ, yε IR. για τις ο­ποίες παρουσιάζονται το μέγιστο και το ε­λάχιστο.

Απόδειξη: α) Έχουμε διαδοχικά:

la · βl ::;; ΙaΙ · Iβl � la · βl2 ::;; (laΙ · Iβl )2 � (a . β)2 ::; Ιal2 · lβl2 � (ιaΙ · Iβl ·συν (a:β)) ::; ΙaΙ2 · lβl 2 �

Ιal2 · lβl 2 συν2 (a:β) ::; Ιa12 · lβl 2 ( 1 ) � Αν ίi = δ ή β = δ η (1) ισχύει ως ισότης. � Αν a :;t: δ και β :;t: δ τότε η (1 ) γίνεται

συν2 (a:β)::;; 1 � lσυν (a:β)j ::;; 1 που ισχύει για

όλες τις γωνίες (a:β) . Η ισότης ιa . βι = lίil · lβl ισχύει όταν και μόνο όταν

lσυν (a:β)j = 1 � συν (a:β) = ±1 � (a:β) = ο ή π � a ι ι β . β) i)Έχουμε: a . β = x3 + y4 = 3χ +4y = Α .

ii) Επειδή Ιa . βι ::;; ιaι . ιβι έχουμε IAI ::; 7 · 5 � IAI ::; 35 � -35 ::; Α ::; 35 .

Επομένως max (Α) = 35 και min (Α) = -35 . Το μέγιστο . και το ελάχιστο παρουσιάζονται όταν ιa . βl = lίil · lβl � a ι ι β � ίi= λ · β, λε IR. � (x, y) = λ(3, 4) .

Επομένως είναι χ = 3λ και y = 4λ . Αλλά χ2 + y2 = 49 ή

9λ2 + 16λ2 = 49 � λ2 = 49 � λ = ±2 . 25 5 Άρα max (A) = 35 , όταν λ =2. οπότε 5

(x,y) = (�1 , 258 ) και min (A) = -35 , όταν

7 ' ( 21 28 ) λ =-s οποτε (x, y) = -5,-5 .

ΕΥΚΛΕΙΔΗΣ Β' λ.ε. τ.l/58

Page 61: Ευκλειδης Β 41

Μαθηματικά για την Γ Λυκείου

Μαθηματικά yιa τn r' τά�n 'IOU Λυκι:ίου

των Γ. Μπαραλή - Γ. Τσικαλουδάκη

Στο άρθρο αυτό αναπτύσσουμε μερικές έννοιες του σχολικού βιβλίου σχετικά με το όριο, την πα­ράγωγο, τα ακρότατα και ακολουθούν διάφορες ασκήσεις επί του κεφαλαίου αυτού.

1 . ΟΡΙΑ Έστω ότι δύο συναρτήσεις f(x), g(x) είναι ί­σες στο διάστημα (α, χ0 ) και στο ( χ0 , β) και η g(x) ορίζεται στο χ0 ενώ η f(x) δεν ορίζεται στο χ0 , όπως για παράδειγμα οι συναρτήσεις:

χ3 - 1 f(x) =-- και g (x) = x2 + χ + 1 , οι9 χ - 1 οποίες είναι ίσες στο (Ο, 1) υ ( 1, 2) , αφού για

κάθε χ 'Φ 1 είναι: χ3 - 1 --=χ2 + χ + 1 . χ - 1 Επομένως οι γραφικές παραστάσεις τους ταυ­τίζονται στο διάστημα (0, 1 ) και στο ( 1 ,2). Ακόμα η

cg διέρχεται από το σημείο (1 ,3), δηλαδή η g παίρνει την τιμή 3 για χ = 1 , ( g (1) = 3 ).

Η Cr δεν περιλαμβάνει το σημείο (1 ,3) δηλα­δή η f δεν παίρνει την τιμή 3 όμως με βάση τη Cr μπορούμε να δεχθούμε ότι για χ :::::: 1 είναι f(x) ::::: 3 . Αυτό δηλώνουμε με την γραφή

limf(x) = 3 = g(1) . χ .... ι Γενικεύοντας έχουμε ότι όταν οι f, g είναι ί­

σες στο (α, χ0 ) υ ( χ0 ,_β) και ορίζεται μόνο η g στο χ0 , τότε θα είναι:

Με βάση την ισότητα ( 1) υπολογίζονται πολ­λά αντίστοιχα όρια (βλέπε άσκηση 6) αρκεί να βρούμε με ποια συνάρτηση g(x) συμπίπτει η f(x), για τιμές του χ κοντά στο χ0 , ( χ :::::: χ0 ).

Είναι ακόμα χρήσιμο να γνωρίζουμε ότι το ό­ριο lim f(x) υπολογίζεται και από τον τύπο

Χ---+Ιο

lim f ( χ0 + h) δηλαδή ισχύει η ισότητα: h .... o

lim f(x) = Iimf(x0 + h) (2) χ .... χ. h .... o

αφού για h � Ο ισχύει χ � χ0 όταν στο πρώτο μέρος της (2) αντί για χ θέσουμε το χ0 + h ( χ = χ0 + h ). Δηλαδή θεωρούμε ότι αφού το χ παίρνει τιμές κοντά στο χ0 θα είναι χ = χ0 + h .

h -- χ χσ +- χσ+h

(Να θυμάστε ότι στο δεξιό μέρος του τύπου (2), η μεταβλητή είναι το h (και όχι το χ0 ), το χ0 είναι σταθερός αριθμός). 2. ΜΟΝΟΤΟΝΙΑ 1. Α ν η συνάρτηση f είναι γνησίως αύξουσα

στο διάστημα (α, χ0 ] και στο [χ0 ,β) , τότε είναι γνησίως αύξουσα και στο (α, β) . Πραγματικά: για χ1 , χ2 ε (α,β) με χ1 � χ0 και χ0 < χ2 είναι: f (χ1 ) � f ( χ0 ) < f ( χ2 ) , άρα

2. Αν για μια συνάρτηση f(x) ισχύει f'(x) � Ο στο (α, β) , τότε δεν μπορούμε να πούμε ότι η f είναι γνησίως αύξουσα στο (α, β) .

Για παράδειγμα: Για τη f(x) = {.J1- x2 , - 1 � χ � Ο 1 , 0 < χ <� 1

συνάρτηση

.εύκολα μπορούμε να αποδείξουμε ότι ισχύει f'(x) � O στο (-1, 1 ) , όμως δεν είναι γνησίως αύ­ξουσα σ' αυτό (γιατί;). Αν όμως η συνάρτηση f είναι συνεχής στο ( α, χ0 ] υ [ χ0 , β) και είναι: f'( χ0 ) = Ο και f'(x) > O για κάθε χ 'Φ χ0 με χ ε (α,β) , τότε βά-

ΕΥΚΛΕΙΔΗΣ Β' λ.ε. τ.l/59

Page 62: Ευκλειδης Β 41

Μαθηματικά για την Γ Λυκείου

σει της παραπάνω παρατήρησης (1 .) η f είναι γνη­σίως αύξουσα στο (α, β) .

Για παράδειγμα η συνάρτηση f (χ) = (χ 3 - 1 )5 είναι γνησίως αύξουσα αφού:

f'(x) = 5 (x3 - l)4 · 3x2 > 0 για κάθε x :;t: O και χ :;t 1 .

3. ΑΚΡΟΤΑΤΑ Με τη βοήθεια των ριζών της παραγώγου

f' (χ) μιας συνάρτησης και της μονοτονίας της (προσήμου της f' ) εντοπίζουμε τοπικά ακρότατα και όχι ακρότατα (ολικά).

Για παράδειγμα η συνάρτηση f(x) = x3 -3χ παρουσιάζει τοπικό ελάχιστα στο 1 , με f (Ι ) = -2 , όμως το -2 δεν είναι ολικό ελάχιστο αφού (π.χ. για χ = -3 είναι f (-3) = -18 .

Όταν όμως το πεδίο ορισμού της συνάρτησης f(x) είναι ένα διάστημα (της μορφής (α,β) ή [α,β) ή (α,β] ή (-σο,β) κτλ.) και η f' μηδενίζε­ται μόνο σε μια θέση χ0 εσωτερική του διαστή­ματος αυτού αλλάζοντας πρόσημο στο χ0 , τότε η f παρουσιάζει (ολικό) ακρότατο στο χ0 • (βλέπε άσκηση 1 0)

Πραγματικά: Έστω π. χ. ότι για μια συνάρτηση f με πεδίο ο­

ρισμού το � ισχύει f' ( χ0 ) = Ο , f' (χ) > Ο για κά-θε χ < χ0 και f'(x) < Ο για κάθε χ > χ0 •

Τότε η f είναι γνησίως αύξουσα στο (-σο, χ0 ] και γνησίως φθίνουσα στο [ χ0, +σο) .

Επομένως για κάθε χ Ε (-σο, χ0 ] είναι χ < χ0 και άρα f (x) < f (x0 ) , ενώ για κάθε Χ Ε [χ0 ,+οο) είναι χ > χ0 και άρα f (x) < f(x0 ) .

Δηλαδή για κάθε χΕ � ισχύει f (x) � f (x0 ) και συνεπώς η f παρουσιάζει μέγιστο στο χ0 .

Στις περιπτώσεις που η f'(x) έχει δύο ή πε­ρισσότερες ρίζες και γενικά δύο ή περισσότερα τοπικά ακρότατα, τότε δεν είναι υποχρεωτικά κά­ποιο από αυτά (ολικό) ακρότατο.

Για παράδειγμα η συνάρτηση f (x) = χ4 - 2χ2 έχει παράγωγο f'(x) = 4x3 -4χ και συνεπώς πα­ρουσιάζει τοπικά ακρότατα στις θέσεις -1 , Ο και 1 .

Εύκολα μπορούμε να δούμε ότι στις θέσεις -1 και 1 έχουμε (ολικό) ελάχιστο, ενώ στο Ο έχουμε τοπικό μέγιστο.

4. ΑΣΚΗΣΕΙΣ 1) Να βρεθεί το πεδίο ορισμού των συναρ-

τήσεων: i) f(x) = J2 - .J5 - x

ii) f(x) =Jln 9χ - xz

14

Λύση i) Για να ορίζεται η f πρέπει:

και <::::::> και <=> 15 - χ � ο 15 � χ

2 - .J5 - x � ο 2 � .J5 - x

{:: 5

<=> {:: 5 <=> 1 � χ � 5 .

4 � 5 - χ x � l

Άρα το πεδίο ορισμού της f είναι το Α = [ 1, 5] . ii) Για να ορίζεται η f πρέπει:

9χ - χ2 9χ - χ2 ]η � ο <=> ]η � ]η 1 <=> 14 14

9χ -χ2 --- � 1 <=> 9χ - χ2 � 14 <=> 14

χ2 -9x + l 4 � 0 <=> 2 � x � 7 . Άρα το πεδίο ορισμού της f είναι το

Α = [2, 7] .

2) Δίνεται η συνάρτηση f (χ) = x.J2s- xz ί) Να βρεθεί το πεδίο ορισμού της. ii) Να δείξετε ότι: f (5συνχ) � 25 .

2

Λύση i) Για να ορίζεται η f πρέπει:

25 - χ2 � Ο <::::::> 25 � χ2 <=> lxl � 5 <=> -5 � χ � 5 . Ά­ρα το πεδίο ορισμού της f είναι το Α = [ -5, 5] .

ii) Για κάθε χ Ε � είναι: Ι � συνχ � Ι ή -5 � 5συνχ � 5 δηλαδή 5συνχ Ε Α = [ -5, 5] .

Επομένως: f (5συνχ) = 5συνχ.J25 -25συν2χ = 5συνχJ25 (Ι -συν2χ) = 5συνχJ25ημ2χ = 5συνχ5 lημχl = 25συνχ lημχι .

Άρα: Ιr (5συνχ )j = l25συνχ Ιημχl l = 25 lημχσυνχl =

25 25 25 τl2ημχσυνχl = τlημ2χl � 2 · 1 <=>

jf(5συνχ)j � 25 δηλ. f (5συνχ) � 25 . 2 2

3) Δίνεται η συνάρτηση f με τύπο ΕΥΚΛΕΙΔΗΣ Β' λ.ε. τ.l/60

Page 63: Ευκλειδης Β 41

Μαθηματικά για την Γ Λυκείου

f (x) = x11 + 8χ- 8 . i) Να αποδείξετε ότι η f είναι γνησίως αύ­

ξουσα. ii) Να λύσετε την ανισότητα f (f (χ)) < 1 .

Λύση i) Το πεδίο ορισμού της f είναι Α = JR . Για

' θ ]]]) ' 1 1 1 1 κα ε χ1 , χ2 ε JL'I>. με χ, < χ2 ειναι: χ1 < χ2 και 8χ1 < 8χ2 . Με πρόσθεση κατά μέλη των παραπά­νω ανισοτήτων έχουμε: χ: 1 + 8χ1 < χ�' + 8χ2 ή χ: Ι + 8χ1 - 8 < Χ�1 + 8Χ2 -8 ή f ( Χ1 ) < f ( Χ2 ) ·

Επειδή για κάθε χ, , χ2 ε Α με χ1 < χ2 ισχύει f (χ, ) < f ( χ2 ) η f είναι γνησίως αύξουσα.

ii) Είναι: f (l) = I1 1 + 8 · Ι - 8 = 9 - 8 = Ι και η f είναι γνησίως αύξουσα. Επομένως: f ( f (χ)) < Ι ή f(f(x)) < f(I) ή f (x) < I ή f (x) < f (I) ή χ < Ι .

4) Δίνεται η συνάρτηση f : JR � JR με χ2 - 2χ + 3 � f (x) � 3χ2 - 6χ + 5 για κάθε χε JR .

Ν α δείξετε ότι η f έχει ελάχιστο, και να το βρείτε.

Λύση Είναι: χ2 - 2x + 3 � f (x) � 3x2 -6χ + 5 ή χ2 - 2x + I + 2 � f(x) � 3x2 -6χ + 3 + 2 ή (χ - Ι)2 + 2 � f (x ) � 3 (x - I)2 + 2 ( Ι) Από την ( Ι ) έχουμε: Για κάθε x ε JR f (x) � (x -02 + 2 � 0 + 2 = 2 Για χ = Ι 0 + 2 � f (I) � 3 · 0 + 2 ή 2 � f (I) � 2

δηλ. f (I) = 2 . Επομένως για κάθε χ ε JR είναι: f (χ) � f (Ι)

δηλαδή η f έχει ελάχιστο το f (Ι) = 2 .

5) Δίνονται οι συναρτήσεις:

f (x) = !.(5• + 5-• ) , g (x) = !.(5• - 5-• ) 2 2 και

h (x) = ;i:�. Να αποδείξετε ότι για κάθε χε JR

ισχύουν: i) f (-x) = f (x) ii) g(-x) + g (x) = Ο iii) f (x) > O ίν) [f (x)]2 - [g (x)]2 = 1 ν) 1 - [b (x)Y = [f (χ)Γ2

Απόδειξη Είναι i) f(-x) =.!.(5-χ + 5-(-χ) ) =.!.(5" +5-χ ) = f(x)

2 2

ii) g (-x) +g (x) = ..!.. (5-x - 5" ) +..!..(5" - 5-χ ) = 2 2 ..!.. (5-" - 5" + 5" - 5-" ) = ..!.. · 0 = 0 2 2

iii) f (x) = ..!_

. (5" + 5-χ ) = ..!..(5" +_Ι )= .!.(52" + Ι )> Ο 2 2 5" 2 5"

iv) [f (x)]2 - [g (x)]2 =..!..(5" + 5-χ )2 _..!_(5" - 5-" )2 =

4 4 ..!..[ (5" + 5-χ / - (5" - 5-χ )2 ] = 4 ..!.. (5" + 5-χ + 5χ - 5-χ ) (5χ + 5-χ - 5χ + 5-χ ) = 4 ..!_ · 2 · 5" · 2 · 5-χ =i . 5x-x = Ι · 5° = Ι 4 4

( ) !(5" -5-χ ) 5" -5-χ ν) Επειδή h(x)=�= 2 =--f(x) !(5" +5-χ ) 5" +5-χ 2

2 (5" -5-χ )2 4 θα είναι: I-[h(x)] = Ι- = 2 = 5" + 5-χ (5" + 5-χ ) Ι Ι

2 = Ι 2 = [f Cx)Γ2 � (5" + 5-χ )2 [� (5χ + 5-χ )] [f (x)]

6)

i)

iii)

Να υπολογιστούν τα όρια:

lim χ2 - 2χ - 8

ii) lim Γx+ll - 3 χ->4 -1 + .Jx- 3 H-2 .Jx+ 27 - 5

I. ..Γχ + � - 2 ιm----χ->1 χ - 1

Λύση Εργαζόμαστε για το (iii) τα άλλα τα αφήνουμε

τ δ' ' f ( ) Γχ + if;. -2 ' ο πε ιο ορισμου της χ = ει-χ - Ι ναι Α = [0, 1) υ (1,-tοο) (γιατί;) και για κάθε χ ε Α έχουμε:

f (x) = Γχ +if;. -2 = Γχ - 1 + if;. - Ι = χ - 1 χ - 1 χ - 1 1 Ι --+-==----Γχ + 1 � + if;. + 1

.

πομενως: χ = --+ = Ε ' limf( ) � 1 1 ] """"* """"* ..rx +1 <.R +rx +1

lim-1- + lim 1 =..!_+_!_ = � . χ->ι j;_ + Ι χ->ι � + if;. + 1 2 3 6

ΕΥΚΛΕΙΔΗΣ Β' λ.ε. τ.l/61

Page 64: Ευκλειδης Β 41

Μαθηματικά για την Γ' Λυκείου

συν2χ 7) Αν f (x) = 2 , χε IR. να αποδείξε-

ι + ημ χ

, ι (π ) '(π ) τε οτι: 3 f 4 = ι + f 4 .

Απόδειξη

Ε, . f'( ) = ( συν2χ J' = -2ημ2χ ιναι. χ 2 2 · 1 +η μ χ ( 1 +η μ 2χ )

Επομένως: 2 li �r( �Η 1:�

=� Yz =Η=� ( Ι ) 4 π (π)

-2ημ2 -2 2 8 1 1+f' 4 =1+( 2J

=1+(3 J

=1-2 =1-9 =9 (2) 1+4 2 4

Από (1 ) και (2) έχουμε: j r( � )= 1 + f'( �} 8) Δίνεται η συνάρτηση

f (χ) = χ3 - 9χ2 + 24χ + λ , λε ΙR. . Αν χ1 , Χ2 εί­ναι θέσεις τοπικών ακροτάτων της f να αποδεί­ξετε ότι η απόσταση των σημείων A (x1 ,f (x1 )) και Β ( χ2 , f ( χ2 ) ) είναι σταθερή.

Απόδειξη Αφού Χι , χ2 θέσεις τοπικών ακροτάτων θα ισχύει f'(χι ) = f'(x2 ) = Ο . Όμως f' (x) = 3χ2 - 1 8χ + 24 οπότε: f'(χι ) = 0 => 3χ� - 1 8χι + 24 = Ο� 3 (χ� - 6Χι + 8) = 0 � 3 (χι - 4)(χι - 2) = 0 f'(x2 ) = 0 � 3 (χ2 -4)(χ2 - 2) = 0

χ -- 2 4 +oc

f' + - δ +

f / � / Τ. Μ. Τ. Ε.

Έστω ότι Χι = 2 τότε f (Χι ) = 20 + λ χ2 = 4 τότε f ( χ2 ) = 1 6 + λ

και

Άρα Α(2, 20 + λ) και Β(4, 16 + λ) . Επομένως d (A,B)=J(x2 - χι )2 + [f (x2 ) - f (xι )]2 =

�(4- 2)2 + (16 + λ - 20-λ)2 =�22 + (-4)2 =

.J4+ 16 =J20 = 2../5 . 9) Δίνεται η συνάρτηση με τύπο

f (x) = χ2 + (5- α)χ- (α + 8) . Για ποια τιμή του α το άθροισμα των τε­

τραyώνων των ριζών της f είναι ελάχιστο;

Λύση Έστω Χι , χ2 οι ρίζες της f(x) . Τότε θα ισχύει: β 5 - α Χι + χ2 = -- � Χι + χ2 =---=α- 5 α 1 γ α+8 ΧιΧ2 = - � ΧιΧ2 = ---=-α-8 α 1 Επομένως:

χ� + χ; = (χι + χ2 )2 - 2χιχ2 = (α- 5)2 -2 (-α -8)=

α2 - 10α+ 25 + 2α + 16 = α2 -8α + 41 Θέτουμε g(α) = α2 -8α+41 οπότε: g'(α) = 2α- 8 = 2(α-4) g'(α) = Ο � 2(α -4) = 0 � α = 4

α --οο 4 +α:

g' - δ +

g � ./ Τ.Ε.

όπως φαίνεται από τον πίνακα μεταβολών το α­κρότατο είναι ολικό. Άρα για α = 4 η g(α) δηλαδή το άθροισμα

των τετραγώνων των ριζών της f είναι ελάχιστο. ιο) Να χωρίσετε τον αριθμό 8 σε δύο θε­

τικούς προσθετέους, ώστε το αποτέλεσμα του πολλαπλασιασμού του yινομένου τους επί τη διαφορά τους να είναι μέyιστο.

Λύση Έστω χ ο μικρότερος από τους δύο αριθμούς στους οποίους χωρίζεται το 8. Τότε Ο ::; χ ::; ! � Ο ::; χ ::; 4 και ο μεγαλύτερος αριθμός 2 είναι ο 8 -χ . Η διαφορά τους είναι (8 - χ)- χ = 8 - 2χ .

Επομένως σύμφωνα με το πρόβλημα θα έχου­με: f (x) = χ (8 - χ)(8 - 2χ) , όπου Ο ::; χ ::; 4 ή f (x) = 2χ3 - 24χ2 + 64χ .

Είναι: f'(x) = 6χ2 - 48χ + 64 f'(x) =Ο� 6χ2 -48χ +64 =0 � 3χ2 -24χ + 32 =0 . Επειδή Δ = 576- 384 = 1 92 θα είναι

ΕΥΚΛΕΙΔΗΣ Β ' λ.ε. τ.l/62

Page 65: Ευκλειδης Β 41

Μαθηματικά για την Γ' Λυκείου

= 24 ± .Ji92 = 4 + {192 = 4 + {16 = 4 + _i_ Χι , χz 6 - ν36 - ν3 - .J3 Από τον πίνακα προσήμου της f' προκύπτει

' f ' ζ ' 4 4 ' οτι η παρουσια ει στο σημειο Χι = - .J3 μεγι-στο, στο διάστημα [0, 4] .

4 4 -χ ο -13 4 f' + 6 -

f / � ΤΜ.

Επομένως ο μεγαλύτερος από τους δύο αριθ-, ' δ ' 8 ' 4 4 μους στους οποιους ιαιρειται το ειναι ο + .J3

' 4 4 και ο μικροτερος ο -.J3 . Σχόλιο: Το παραπάνω πρόβλημα αναφέρεται στην Ι­στορία των Μαθηματικών ως «πρόβλημα του

Tartalia)) (1500- 1 577). Χωρίς να περιγράψει τη λύση ο Tartalia διατύπωσε την απάντηση ως εξής: Πάρτε το μισό του αριθμού 8. Το τετράγωνο αυτού προσαυξημένο κατά το ένα τρίτο του θα ι­σούται με το τετράγωνο της διαφοράς των δύο με­ρών. Δηλαδή αν συμβολίσουμε τους δύο αριθμούς με α και β (α > β ) τότε σύμφωνα με τον Tartalia θα έχουμε: ( )2 ( )2 ( )2 2 42 16 64 α-β = 8 : 2 + 8 : 2 : 3 =4 +-=16+-=-3 3 3 οπότε α -β = .1 ( 1 )

Όμως α+ β = 8 (2) Από (1 ) και (2) με πρόσθεση κατά μέλη προ-8 4 κύπτει 2α = 8+- οπότε α = 4+- και .J3 .J3

β = 4-� . Η συλλογιστική του Tartalia που τον οδήγησε στη σωστή απάντηση έχει άμεση σχέση με τη λύση τριτοβάθμιας εξίσωσης, παρουσιάζει μεγάλο ενδιαφέρον αλλά ξεφεύγει από τον σκοπό αυτού του άρθρου.

11) Ν α αποδείξετε ότι από όλους τους κυ­λίνδρους JU την ίδια διάμετρο, αυτός που έχει τη μεγαλύτερη χωρητικότητα είναι εκείνος στον οποίο ο λόγος της διαμέτρου της βάσης προς το ύψος ισούται με το J2 . Με άλλα λόγια σε δε­δομένη σφαίρα να εγγράψετε έναν κύλινδρο μέ-

γιστου όγκου.

Λύση Έστω χ το μισό του ύψους του κυλίνδρου, και R την ακτίνα της σφαίρας. Η ακτίνα της βάσης του κυλίνδρου είναι r2 = R2 - χ2 ή r = ../R 2 - χ2 με O � x � R .

Ο όγκος του κυλίνδρου δίνεται από τη σχέση ν(χ )= πr2 · h = π(R2 - χ2 ) . 2χ = 2π(R2 - χ2 )χ � ν(χ) = 2π(R2 - χ2 )χ , Ο � χ � R .

I Είναι V'(x) = (2π(R2x-x3 )) = 2π(R2 -3χ2 ) ν'(χ) = Ο � 2π(R2 - 3χ2 ) = 0 � R2 -3χ2 = 0 �

-R Χι = ,-;; απορρ. 3x2 = Rz � x =±_!_= ( ν3

.J3 R δ , χ2 = .J3 εκτη Από τον πίνακα προσήμου της ν' προκύπτει

, ν ' ζ , R , οτι η παρουσια ει στο σημειο χ2 = .J3 τοπικο μέγιστο, με που είναι προφανώς ολικό. v(� }1R'-�}2π� '�= :"%

9

R χ ο J3 R V' + 6 -

ν / � Τ. Μ.

Επομένως η ακτίνα του κυλίνδρου που έχει μέγιστο όγκο θα είναι r =../R2 - x2 =JR2 -�z =J2�z � r = Rff .

Το ύψος του κυλίνδρου είναι h = 2χ = � . 2RJ2

Άρα _2r = ----π- = 2R J2 . .J3 = J2 h 2R 2.J3R .

.J3 Σχόλιο: Το πρόβλημα αυτό είναι το θεώρημα ν (μέρος δεύτερο) που αναφέρει ο Kep1er (1 571 - 1630) στο

ΕΥΚΛΕΙΔΗΣ Β' λ. ε. τ.l/63

Page 66: Ευκλειδης Β 41

Μαθηματικά για την Γ Λυκείου

έργο του "Stereometria doliorum νinorum", όπου υπολογίζει διάφορες αναλογίες μεταξύ του όγκου των βαρελιών και των διαστάσεών τους και λύνει διάφορα προβλήματα πάνω στα μέγιστα και τα ε­λάχιστα τα οποία όπως ισχυρίζονται πολλοί ερευ­νητές βοήθησαν τον Newton στη δημιουργία του διαφορικού λογισμοί>. 12) Η ΔΕΗ καθορίζει την τιμή του ηλεκτρι­

κού ρεύματος ανά κιλοβατώρα (σε τριμηνιαία κατανάλωση) ως εξής:

Από Ο έως και 10 KWh 15 δρχ. Από 1 Ο έως και 50 KWh 17 δρχ. Από 50 έως και 100 KWh 19 δρχ. Από 100 και άνω KWh 20 δρχ. i) Να βρεθεί η συνάρτηση κόστους Χ

KWh. ii) Να υπολογιστεί η αξία ρεύματος για ένα

νοικοκυριό που ο μετρητής έγραψε 80 KWh. iii) Αν ένας καταναλωτής πλήρωσε για κα­τανάλωση ρεύματος 1723 δρχ., να βρείτε πόσες ΚWh έκαψε.

Λύση i) Η συνάρτηση κόστους Κ(χ) έχει τύπο:

15χ, ο :::; χ :::; 10 Κ(χ )= 17χ - 2Ο, 10 < χ :::; 5ο 19χ - 120, 50 < χ :::; 100

20χ - 220, χ > 100 ii) Είναι Κ(80) = 19 · 80 -120 = 1400 iii) Είναι: Κ (15) = 150 , Κ (50) = 830 ,

Κ (100) = 1980 και επειδή είναι 830 < 1723 < 1980 , η κατανάλωση ρεύματος θα εί­ναι στο διάστημα 50 έως 100. Συγκεκριμένα είναι: 1723 = 19χ -120 ή 19χ = 1 843 ή χ = 97Kwh . Άρα ο συγκεκριμένος καταναλωτής έκαψε 97 KWh.

13)Το κόστος παραγωγής της μιας μονάδας ενός προϊόντος, όταν παράγονται χ μονάδες, δί-

νεται από τον τύπο: Κ (χ) = 100- 5χ + 40 . Η τι­χ

μή πώλησης της μιας μονάδας πρέπει να είναι 40% μεγαλύτερη από την τιμή κόστους. i) Να βρεθεί η συνάρτηση των εσόδων από την πώληση χ μονάδων.

ii) Να βρεθεί σε πόσες μονάδες έχουμε με­γιστοποίηση των κερδών.

Λύση i) Έστω Ε (χ) τα έσοδα από την πώληση χ

μονάδων προϊόντος. Είναι: Ε (χ )= χ · Κ (χ) + 40 χ · Κ(χ) ή 100 Ε (χ )=2.χ · Κ (χ) = -7χ2 +56χ + 140 . 5 ii) Έστω Ρ (χ) τα κέρδη από την πώληση χ

μονάδων. Προφανώς είναι Ρ (χ) = χ 40 Κ (χ) . 100 Άρα Ρ(χ) =3_(-5χ2 +40χ + 100) ή 5

Ρ(χ) = -2χ2 + 16χ +40 και ως γνωστό το τριώνυ-μο Ρ (χ) παίρνει μέγιστη τιμή για (χ = -:α }

16 χ =-=4 . 4 Άρα σε παραγωγή και πώληση 4 μονάδων έ­χουμε μεγιστοποίηση των κερδών. 14) Δύο βιομηχανίες συνεργάζονται για την

παραγωγή ενός προϊόντος. Η πρώτη βιομηχανία έχει κόστος παραγωγής v μονάδων Κ1 ( v) = 2v + 3 σε χιλιάδες δρχ.. Η δεύτερη βιομηχανία έχει κόστος αποπεράτωσης προϊό­ντος, κόστους χ χιλιάδων δρχ. της πρώτης: Κ2 (χ) = χ

2 - 3χ + 4 χιλιάδες δρχ ..

i) Να βρεθεί το συνολικό κόστος παραγω­γής v μονάδων προϊόντος.

ίί) Ν α βρείτε σε πόσες το πολύ μονάδες το συνολικό κόστος δεν υπερβαίνει τις 67 χιλιάδες δρχ. και ποιο είναι το κόστος ανά μονάδα για την κάθε βιομηχανία, στην περίπτωση αυτή.

Λύση i) Αν Κ (ν) είναι το συνολικό κόστος παρα-

γωγής ν μονάδων, έχουμε: Κ(ν) = Κ1 (ν) +Κ2 (Κ1 (ν)) δηλαδή: Κ( ν) = 2ν + 3 + (2ν +3)2 -3 (2ν +3)+ 4 ή Κ(ν) = 4ν2 + 8ν + 7 ii) Έχουμε: Κ(ν) :::; 67 {::::) 4ν2 +8ν-6ο:::; ο {::::)

ν2 + 2ν - 1 5 ::; ο{::::) (ν -3)(ν +5) :::; ο {::::) ν :::; 3 Συνεπώς σε 3 το πολύ μονάδες το συνολικό κόστος δεν υπερβαίνει τις 67 χιλιάδες δρχ. Το κόστος παραγωγής ανά μονάδα, όταν πα­ράγονται 3 μονάδες, είναι:

, β , Κ1 (3) 9 3 ιλ , δ για την πρωτη ιομηχανια: -3-= "3 = χ ια ες δρχ. και για την δεύτερη:

ΕΥΚΛΕΙΔΗΣ Β' λ.ε. τ.l/64

Page 67: Ευκλειδης Β 41

Μαθηματικά για την Γ' Λυκείου

Κ2 (9) 92 - 3 · 9 + 4 58 19 33 ιλ ' δ δ -- = =-:::: , χ ια ες ρχ. 3 3 3 15) Από τις πωλήσεις ενός προϊόντος διαπι­

στώθηκε ότι σε ελάττωση χ εκατοντάδων δραχ­μών, ανά μονάδα προϊόντος, έχουμε διαφορά η-μερησίων κερδών Δ (χ) = 6χ2 - 5χ - χ3 εκατο­ντάδες χιλιάδες δρχ.

ί) Να βρεθεί με ποια ελάττωση της τιμής του προϊόντος δεν έχουμε ελάττωση ημερησίων κερδών.

ίί) Αν τα ημερήσια κέρδη ανέρχονται σε 1,2 εκατομμύρια δρχ., να βρείτε πόση πρέπει να εί­ναι η ελάττωση του προϊόντος ώστε να έχουμε διπλασιασμό των ημερησίων κερδών.

Λύση i) Για να μην έχουμε ελάττωση κερδών πρέ­

πει να είναι Δ (χ) � Ο , δηλαδή -χ3 + 6χ2 � 5χ � Ο ( 1) και επειδή είναι χ � Ο η (1) είναι ισοδύναμη με την:

χ2 - 6χ + 5 � ο<=> (χ - 1)(χ - 5) � 0 . Συνεπώς είναι 1 � χ � 5 . ii) Διπλασιασμό ημερησίων κερδών έχουμε

όταν Δ (χ) = 12 δηλαδή όταν -χ3 + 6χ2 - 5χ = 12 ή χ3 - 6χ2 + 5χ + 12 = 0 (2) Με τη βοήθεια του σχήματος Homer η (2) γί-νεται: (χ - 3)(χ2 - 3χ - 4) = 0 ή (χ - 3)(χ - 4)(χ + 1) = ο Άρα σε μείωση της τιμής κατά 300 ή 400 δρχ. έχουμε διπλασιασμό των κερδών.

16)Σώμα Σ ξεκινά από το σημείο Α (σχήμα) με σταθερή ταχύτητα 2 cm/s και κινείται στη γραμμή ΑΒΓ ΔΑ, μέχρι να ξαναφθάσει στο Α.

ί) Να εκφράσετε συναρτήσει του χρόνου t την απόσταση S(t) του Σ από το Α.

ίί) Ν α βρεθεί η χρονική στιγμή που είναι S(t) = 10 cm.

ίίί) Να βρεθεί το εμβαδόν E(t) που έχει δια­γράψει τη χρονική στιγμή t η διανυσματική α­κτίνα ΛΣ.

i)

Λύση υ

l 2 cm

2t, O � t � 4 �4(t -4)2 + 64 , 4 < t � 10 Έχουμε: S(t) = �(28 -2t)2 + 122 ' 10 < t � 14 40 -2t, 14 < t � 20

ii) Είναι: S(t) = 10<=> �4(t -4)2 + 64 = 10 ή 40-2t = 10 (γιατί;) <::::> t = 7sec ή t = 15 sec .

iii)

Ο, O � t � 4 8t - 32, 4< t � 10 Έχουμε: E (t) = 12t - 72, 10 < t � 14 96, 14 < t � 20

��)ΓUΠΙΚΝ ΙΕΣΗ rP�f/JΙK/211 ΠΑΡΑΣτ�U/211 418 ΣIΝΑΡΤΝΣDΙΙ

των Δάφνη Δημητρίου - Δάφνη Σπύρου

Θεωρούμε ότι τα πεδία ορισμού των συναρτήσεων του ζεύγους που μελετάμε, είναι ίδια. (Αν οι συναρτήσεις έχουν διαφορετικό πεδίο ορισμού, τονίζουμε τη διαφορά που υπάρχει στη γραφική παράσταση). Για την μελέτη της σχετικής θέσης δύο συ­ναρτήσεων βρίσκουμε, (όπου είναι δυνατόν· οι δύο συναρτήσεις ορίζονται στην τομή D των πεδίων ο­ρισμού τους), τη διαφορά Α = f (χ) - g (χ) και έ-χουμε τις παρακάτω περιπτώσεις:

1 η Α = Ο . Τότε οι δύο γραφικές παραστάσεις

συμπίπτουν κατά μήκος του D. (Δηλαδή οι συναρ­τήσεις είναι ίσες, στο κοινό πεδίο ορισμού τους). 2η Α = c (σταθερός αριθμός). Τότε:

f(x) = g (x)+ c και η Cr είναι η Cg μετατοπι­σμένη κατακόρυφη κατά c μονάδες.

i. Προς τα πάνω, αν c > Ο ΕΥΚΛΕΙΔΗΣ Β' λ.ε. τ.l/65

Page 68: Ευκλειδης Β 41

Μαθηματικά 'f\O την Γ' Λυκεiου ίί. Προς τα κάτω, αν c < Ο . 3η Το Α να είναι παράσταση της μεταβλητής

χ. Α ν είναι δυνατόν να μελετηθεί ως προς το

πρόσημο έχουμε: ί. Για τα διαστήματα που η παράσταση Α εί­

ναι θεnκή, η Cr είναι πάνω από την C8 • ίί. Για τα διαστήματα που η παράσταση Α εί­

ναι αρνητική, η Cr είναι κάτω από την C8 •

iii. Οι τιμές του χ, όπου Α = Ο , είναι αυτές που δίνουν τα κοινά σημεία των γραφικών παρα­στάσεων.

Δηλαδή, αν η εξίσωση Α = Ο δίνει λύσεις χ = χ1 και χ = χ2 , τα κοινά σημεία των γραφικών

παραστάσεων των συναρτήσεων f και g είναι τα

K(x1 ,f (x1 )) και Λ(χ2 ,f (χ2 )) . Παρατήρηση: Περισσότερα για την μελέτη του προσήμου

της διαφοράς Α = f (χ)-g (χ) και γενικά για τη

γραφική παράσταση συνάρτησης, δίνει η παράγω­γος συνάρτησης.

Άλλες περιπτώσεις: α) Αν έχουμε g(x) = λ · f(x) , τότε για τη

γραφική παράσταση της συνάρτησης g ισχύει: Η τεταγμένη g(x) , για κάθε χ, είναι ίση με το γινό-

μενο της τεταγμένης f(x) με το λ. Για τον λόγο αυτό δεν υπάρχει σταθερή μετατόmση των γραφι­κών παραστάσεων των συναρτήσεων f, g (Παρά­δειγμα σχήμα 1 °').

Παράδειγμα: Οι γραφικές παραστάσεις των συναρτήσεων f,

g, k με τύπους f(x) = ημχ , g(x) = 2ημχ ,

k(χ) = 3ημχ είναι: (Σχ. ι)

Σχ. ι Παράδειγμα: Οι γραφικές παραστάσεις των συναρτήσεων

f, g με τύπους f(x)= x3 και g(x)= -x3 είναι:

(Σχ. 2) Σημείωση: Α ν λ = -ι έχουμε την ειδική μορφή

g(x) = -f(x) και τότε: Οι γραφικές παραστάσεις

των f, g είναι συμμετρικές ως προς τον άξονα χχ' . Γιατί για κάθε σημείο M(x,f(x)) της γραφικής

παράστασης της συνάρτησης f, το

M'(x',-f (x) = g(x)) ανήκει στην γραφική παρά-

σταση της συνάρτησης g. (Παράδειγμα σχήμα 2'").

β) Αν έχουμε g(x) = f(x + c) , η γραφική

παράσταση της συνάρτησης g προκύπτει αν μετα­τοπίσουμε τη γραφική παράσταση της συνάρτησης f παράλληλα προς τον άξονα χ'χ κατά c μονάδες. Και μάλιστα:

ί . Προς τα δεξιά, αν c < Ο ίί. Προς τα αριστερά, αν c > Ο . Παράδειγμα: Οι γραφικές παραστάσεις των συναρτήσεων f,

g, k με τύπους: f (x) = ..!.. , g(χ) = -ι- , χ χ + ι ι ι k (x) =- . (Σχ. 3) Έχουμε: f(x) =- , χ - ι χ ι ι g(x) = - = f(x + ι) , k (x ) = - = f(x - ι) . χ + ι χ - ι

y"

Σχ. 2 Σχ. 3 Άσκηση 1 11 Να βρεθεί η σχετική θέση των γραφικών

παραστάσεων των συναρτήσεων f, g: ι ι 2χ - ι ·

i. f (x) = - + -- και g(x) = --x χ - ι ΧΖ - Χ

xz - ι χ + ι ίί. f (x) = z και g (x) = -- . χ - 3χ + 2 χ - 2 Λύση: i. Πεδίο ορισμού της συνάρτησης f = Πεδίο

ορισμού της συνάρτησης g = Α = JR -{Ο, ι} . ι ι 2χ - ι f(x)- g(x) =-+-- -2- = 0 . χ χ - ι χ - χ

Οι γραφικές παραστάσεις των f και g ταυτίζο­νται.

ΕΥΚΛΕΙΔΗΣ Β' λ.ε. τ.l/66

Page 69: Ευκλειδης Β 41

Μαθηματικά yι.α την Γ Λυκείου

ii. Πεδίο ορισμού της συνάρτησης f, Α = IR -{1,2} υποσύνολο πεδίου ορισμού της συ-νάρτησης g, Β = IR - {2} .

χ2 - 1 x+ l f (x) -g (x) 2 ---=0 . χ -3χ + 2 χ - 2 Οι γραφικές παραστάσεις των f και g ταυτίζο­νται κατά μήκος του συνόλου Α. Άσκηση 2η: Δίνονται οι συναρτήσεις f, g με τύπους:

4χ2 + 2χ - 12 χ - 3 f (x) = 2 και g (x) =--. Να βρε-χ + χ - 2 χ - 1 θεί η σχετική θέση των Ύραφικών τους παρα­στάσεων.

Λύση: Το πεδίο ορισμού της συνάρτησης f, Α = IR -{ -2, 1} είναι υποσύνολο του πεδίου ορι-σμού της συνάρτησης g, Β = IR -{Ι} .

f ( ) ( ) 4χ2 + 2χ - 12 χ - 3 _ 3 ο χ - g χ = ---- > χ2 + χ - 2 χ - 1 οπότε f (x )= g (x )+ 3 .

Άρα η Cr είναι η Cg , μετατοπισμένη κατα­κόρυφα προς τα πάνω κατά 3 μονάδες, με τη δια-φορά που στην Cr δεν ανήκει το σημείο Ρ( -2,%) της cg .

Άσκηση 3η: Για τις συναρτήσεις f, g ισχύει για κάθε

χε ΙR η συνθήκη: Ιr (x) + 2 - g (x)I = O . (1) Να βρεθεί η σχετική θέση των Ύραφικών

παραστάσεών τους.

Λύση : Η ( 1) είναι ισοδύναμη με την

f (x )+ 2-g(x) = O ή f (x)-g (x) = -2 < 0 . Η Cr είναι η Cg , μετατοπισμένη κατακόρυφα προς τα κάτω κατά 2 μονάδες.

Άσκηση 4η: Για τις συναρτήσεις f, g ισχύει Ύια κάθε χε IR η συνθήκη:

f2 (χ) + 1 + g2 (χ) = 2f (x) · g (x) + 2f (x) - 2g(x) . Να βρεθεί η σχετική θέση των Ύραφικών

παραστάσεών τους.

Λύση: Η δοσμένη συνθήκη είναι ισοδύναμη με την

f2 (χ)+ 1 +g2 (x)-2f(x) ·g(x)-2f(x)+2g(x) =0 ή

(Είτε απ' ευθείας, είτε σαν τριώνυμο με μεταβλητή το g(x) ) [f(x)- 1 - g(x)Y = 0 1 ή ισοδύναμα f (x)- 1 - g(x) = O ή f (x)-g (x) = l > O .

Η Cr είναι η Cg , μετατοπισμένη κατακόρυφα προς τα πάνω κατά 1 μονάδα.

Άσκηση Sη: Για τις συναρτήσεις f, g ισχύει Ύtα κάθε χ ε IR

η συνθήκη: f (x) = g (x) + a2 - 4 , aε IR .

i. Να μελετηθεί η θέση των Ύρ«φικών τους παραστάσεων Ύtα τις διάφορες τιμtς της παραμέ­τρου.

ίί. Να βρεθεί η τψή του a, αν οyνωρίζουμε ότι η Ύρ«φική παράσταση της f f:ι.ει προκύψει από την κατακόρυφη μετατόπιση προς τα πάνω της Ύρ«φι­κής παράστασης της g κατά 5 μονάδες.

iii. Να βρεθεί η τψή του a, αν ΎVωρίζουμε ότι η Ύρ«φική παράσταση της f t:ι.,ει προκύψει από την κατακόρυφη μετατόπιση προς τα κάτω της Ύρ«φι­κής παράστασης της g κατά 6a + 3 μονάδες.

Λύση: Έχουμε f (x)-g (x) = a2 - 4 . Τότε: i. α) Αν a2 -4 = 0 ή ισοδύναμα a =±2 οι γραφικές παραστάσεις των f, g συμπίπτουν. β) Αν a2 - 4 > Ο ή ισοδύναμα a ε ( -οο, -2) υ ( 2, +co) η γραφική παράσταση της f

είναι η γραφική παράστασης της g, μετατοπισμένη κατακόρυφα προς τα πάνω κατά a 2 - 4 μονάδες.

γ) Αν a2 - 4 < 0 ή ισοδύναμα aε (-2, 2) η γραφική παράσταση της f είναι η γραφική παρά­σταση της g, μετατοπισμένη κατακόρυφα προς τα κάτω κατά a 2 - 4 μονάδες.

ii. Η γραφική παράσταση της f έχει προκύψει από την κατακόρυφη μετατόπιση προς τα πάνω της γραφικής παράστασης της g κατά 5 μονάδες. Έτσι έχουμε: a2 - 4 = 5 � a2 = 9 � a = ±3 (δεκτές και οι δύο;)

iii. Η γραφική παράσταση της f έχει προκύψει από την κατακόρυφη μετατόπιση προς τα κάτω της γραφικής παράστασης της g κατά 6a + 3 μονάδες. Έτσι έχουμε: a2 -4 < 0 και a2 - 4 = 6a + 3 .

a2 - 4 = 6a +3� a2 - 6a - 7 = 0� a = 7 ή a = -1 . Α ν a = 7 � a 2 -4 > Ο απορρίπτεται (μετατό­πιση προς τα πάνω). Αν a = -l � a2 - 4 = 6a + 3 < 0 δεκτή.

1 Σ'αυτό καταλήγουμε (Είτε απ' ευθείας, είτε αν θa:ο­ρήσουμε το πρώτο μέλος ως τριώνυμο με μεταβλητή το g (x) )

ΕΥΚΛΕΙΔΗΣ Β' λ.ε. τ.l/67

Page 70: Ευκλειδης Β 41

Μαθηματικά yια την Γ Λυκείου

Άσκηση 6η: Αν για τις συναρτήσεις f, g με πεδίο ορι­

σμού το IR ισχύουν οι συνθήκες: ί. Η Cr είναι η Cg , μετατοπισμένη κατα-

κόρυφα προς τα πάνω κατά c μονάδες. ίί. Α (1, 3)ε C1 •

ίίί. f1 (2) + g1 (2) = 2f (2)g (2) + 4 . Να pρεθεί το k ε IR , ώστε το σημείο

B(l,k ) να ανήκει στην Cr . Λύση: Η Cr είναι η Cg , μετατοmσμένη κατακόρυφα

προς τα πάνω κατά c μονάδες :::::> f (x) = g(x)+ c για κάθε χε IR με c > O άρα και f (2) = g(2)+ c (.1 ). f2 (2)+ g2 (2) = 2f (2)g (2)+ 4 ή ισοδύναμα [g (2)+ c]2 +g2 (2) = 2[g (2) + c]g (2)+ 4 ή c2 = 4 {c = 2 δεκτό

ή ή (2) c = -2 απορρίπτεται Από (ι) και (2) συνάγεται ότι: f(x) = g(x) + 2 . Α (ι,3)ε cg ή g(ι) = 3 . Όμως f (l ) = g0)+ 2 οπότε f (ι) = 3 + 2 άρα

f (ι) = 5 = k . Άσκηση 7η: Δίνονται οι συναρτήσεις f, g με τύπους

f (χ) = 2Χ+1 και g (χ) = 2• + 4 . Να pρεθεί η σχε­τική θέση των γραφικών παραστάσεών τους.

Λύση: Προφανώς και οι δύο συναρτήσεις ορίζονται σ' ό'λλJ το � -Έχουμε: h (χ ) = f (χ ) - g ( χ )= 2χ+ι - 2χ - 4 ή

ισοδύναμα h (x) = 2 · 2x -2χ -4 ή h (x) = 2x -4 ή h (x) = 2x - 22 •

Τότε: α) Αν χ > 2 τότε h(x) > O . Η γραφική παράσταση της f είναι πάνω από την γραφική παράσταση της g. β) Αν χ < 2 τότε h (x) < O . Η γραφική παράσταση της f είναι κάτω από την γραφική παράσταση της g. γ) h(x) = O ή ισοδύναμα 2χ = 22 ή χ = 2 . Οι γραφικές παραστάσεις των f, g έχουν ένα

μόνο κοινό σημείο το Α(2,8) . Άσκηση 8η: Για τις συναρτήσεις f, g ισχύει η συνθήκη:

f (χ) = g (χ) + χ1 - ι ' για κάθε χ Ε IR . Να μελε­τηθεί η θέση των γραφικών τους παραστάσεων.

Λύση: Έχουμε f(x )- g(x) = x2 - ι . Τότε: i. Η συνθήκη f (χ) = g (χ) είναι ισοδύναμη

με την χ2 - ι = Ο ή χ = ±ι . Οι γραφικές παραστάσεις των f, g τέμνονται σε δύο σημεία A(ι, f (l )) , Β( -ι, f (-1)) .

ii. f (x) > g(x) ή ισοδύναμα χ2 - ι > Ο ή χε (-οο,-ι) υ (ι,+οο) .

Η γραφική παράσταση της f είναι πάνω από την γραφική παράσταση της g. iii. f(x)< g(x) ή ισοδύναμα χ2 - ι < Ο ή

χ ε (-ι, ι) . Η γραφική παράσταση της f είναι κάτω από την γραφική παράσταση της g. Άσκηση 9η: Να μελετηθεί η σχετική θέση των γραφικών

παραστάσεων των συναρτήσεων f, g όταν: ί. f (x) = 2x3 και g (x) = 2x3 -6χ1 + 6χ - 2 . ίί. f (x) = ln x και g (x) = ln (x + 2) . Λύση: i. f(x) = 2x3 και g(x) = 2x3 -6χ2 + 6χ - 2 . Έχουμε g(x) = 2(x - 1)3 = f (x -0 . Η γραφική παράσταση της συνάρτησης g είναι η γραφική παράσταση της συνάρτησης f με τύπο

f (x) = 2x3 ' μετατοπισμένη δεξιά κατά ι μονάδα. ii. f (x) = ln x και g (x) = ln (x + 2) . Έχουμε g (x) = ιη (χ + 2) = f (x + 2) . Η γραφική παράσταση της συνάρτησης g είναι η γραφική παράσταση της συνάρτησης f με τύπο f (χ) = ln χ , μετατοπισμένη αριστερά κατά 2 μονάδες. Παρατήρηση: Πολλές φορές κάνουμε γραφική παράσταση συνάρτησης, χρησιμοποιώντας τα παραπάνω. Δη­λαδή γράφουμε αυτήν: α) Σαν αλγεβρικό άθροισμα πραγματικού α­ριθμού και γνωστής συνάρτησης. β) Στην μορφή g (x) = f (x + c) . Άσκηση 1 0η : Να γίνει η γραφική παράσταση των παρα-

κάτω συναρτήσεων.

ί. f : f (x) = Ιn (e1x) . ii. 1 - 2χ g : g (x) = -- . χ

ίίί. k (χ) = χ3 + 3χ1 + 3χ + 3 . ΕΥΚΛΕΙΔΗΣ Β ' λ.ε. τ.l/68

Page 71: Ευκλειδης Β 41

Μαθηματικά για την Γ Λυκείου

Λύση: i. f (x) = In(e2x) = lne2 + lnx = 2 + ln x . Η γραφική παράσταση της συνάρτησης f είναι

η γνωστή h (χ) = ln χ , μετατοπισμένη κατακόρυ­φα προς τα πάνω κατά 2 μονάδες. (Σχ. 4) . . ( ) 1 - 2χ 1 ( 2) 11 . g χ = -- =-+ - . χ χ Η γραφική παράστα(τη της συνάρτησης g είναι ' ( ) 1 ' ' η γνωστη η χ =- , μετατοπισμενη κατακορυφα χ προς τα κάτω κατά 2 μονάδες. (Σχ. 5)

Σχ. 4 Σχ. 5 111. k (χ) = χ 3 + 3χ 2 + 3χ + 3 = (χ + 1 )3 + 2 . Άρα η γραφιιcή παράσταση της συνάρτησης k

είναι η γραφική παράσταση της γνωστής p (χ) = χ 3 , (Σχ. 6) μετατοπισμένη αριστερά κατά 1 μονάδα και κατακόρυφα προς τα πάνω κατά 2 μονάδες.

Σχ. 6 Ασκήσεις προς λύση: 1 η) Για τις συναρτήσεις f, g ισχύει για κάθε

χ ε � η συνθήκη: f(x) = g (x)+ a2 - 9 , a ε � . i. Να μελετηθεί η θέση των γραφικών παρα­στάσεων για τις διάφορες τιμές του a ε � . ii. Να βρεθεί η τιμή του a αν γνωρίζουμε ότι

η γραφική παράσταση της f έχει προκύψει από την κατακόρυφη μετατόπιση προς τα πάνω της γραφι­κής παράστασης της g κατά 7 μονάδες. iii. Να βρεθεί η τιμή του a, αν γνωρίζουμε ότι η γραφική παράσταση της f έχει προκύψει από την κατακόρυφη μετατόπιση προς τα κάτω της γραφι­κής παράστασης της g κατά 2a - 1 μονάδες. 2η) Για τις συναρτήσεις f, g ισχύει για κάθε

χε � η συνθήκη: f (x) = g(x)+ x3 + χ2 - 2χ . Να μελετηθεί η θέση των γραφικών τους παραστάσεων.

3η) Αν για τις συναρτήσεις f, g με πεδίο ορι­σμού το � ισχύουν οι συνθήκες: i. Η Cr είναι η Cg μετατοπισμένη κατακό-

ρυφα κάτω κατά c μονάδες. ii. Α(2,5)ε cg .

iii. f2 (5) + g2 (5) = 2f (5)g(5) + 9 . Να βρεθεί το kε � , ώστε το σημείο B(2,k)

να ανήκει στη Cr .

4η) Για τις συναρτήσεις f, g ισχύει για κάθε χ ε � η συνθήκη: lf (x) - g (x) + 21 + lf2 (χ) -2+ g2 (x) -2f(x) · g(x) +f (x) -g(x)l =0 .

Να βρεθεί η σχετική θέση των γραφικών πα­ραστάσεών τους. 5η) Α ν για τις συναρτήσεις f, g με τύπους:

f(x)= (2a+3)x+b-2 και g(x) = (3b-4)x+a+3 οι γραφικές τους παραστάσεις είναι συμμετρικές ως προς τον άξονα χχ' , να βρεθούν τα a, b ε � .

6η) Να γίνει η γραφική παράσταση των παρα­κάτω συναρτήσεων: i. f (x) = log[lOO(x + 1)] και ii. g(x) = x3 -6x2 + 12 - 7 .

r ιιε,ικέs Ε1ΙΙ6ΙΙΙ'.Υtf&ΙS •• γ(j}

6ΤΙΟS ΙΙιylλlικΙόs ιfΙιlpΙόs

ΕΝΟΤΗΤΑ lη : ΘΕΩΡΗΤΙΚΕΣ ΕΠΙΣΗΜΑΝΣΕΙΣ Εγώ θα σας πω Πόσο αγάπησα τη θάλασσα Τα καράβια θα τα ζωγραφίσετε εσείς. Ποίηση: Μένης Καλαντζόπουλος

του Σπ. Γιαννακόπουλου

Θεωρώ δεδομένο ότι το θεωρητικό μέρος εί­ναι γνωστό, οπότε παρακάτω γίνονται μερικές μό­νο χρήσιμες επισημάνσεις. 1 η: Για το μιγαδικό z, η μορφή z = α+ βί με α, β ε � , ονομάζεται αλγεβρική ή κανονική μορφή

Ποιητική συλλογή: «Σωματείο Αχθοφόρων»

ΕΥΚΛΕΙΔΗΣ Β' λ.ε. τ.l/69

Page 72: Ευκλειδης Β 41

Μαθηματικά οyια την Γ Λυκεiου

του z. ( ί2 = -ι ). •:• Το α ονομάζεται πραγματικό μέρος του z και συμβολίζεται Re(z). •:• Το β ονομάζεται φανταστικό μέρος του z και συμβολίζεται lm(z). •:• Το βί ονομάζεται φανταστικός αριθμός. ./ Ένας πραγματικός αριθμός είναι μιγαδικός με φανταστικό μέρος μηδέν. ./ Ένας φανταστικός αριθμός είναι μιγαδικός με πραγματικό μέρος μηδέν. ./ Συζυγής μιγαδικός: Αν z = α + βί τότε ο

z = α -βί λέγεται συζυγής του z και συμβο­λίζεται z z = α-βί 'Αμεσα προκύπτει ότι: (z) = z .

Παράδειγμα:

Ν , δ , 2 - i 'I . α γραψετε το μι γα ικο z = -- σε αι.. γε-l + ί

pρική μορφή. Λύση Έχουμε ι + i = ι - i . Πολλαπλασιάζουμε τον αριθμητή και τον παρονομαστή του z με τον ι :- i

, (2 - ί)(ι -n ι - 3ί ι 3 . και παιρνουμε z = -- = ---ι . (ι + ί)(ι - ί ) 2 2 2

2η: Στο μιγαδικό επίπεδο Oxy, σε κάθε μιγα­δικό z με εικόνα το σημείο Α αντιστοιχεί το διά-wσμα ΟΑ που ονομάζεται διαwσματική ακτίνα του z ή διάνυσμα θέσης του z και αντίστροφα, σε κάθε διάνυσμα ΟΑ αντιστοιχεί ένας μιγαδικός α­ριθμός με εικόνα το Α.

Το loAI ονομάζεται. μέτρο του z και συμβο-λίζεται lzl , δηλαδή lzl = �(Re(z))2 + (Im(z))2 •

Α ν Α ( z1 ) , Β ( z2 ) οι εικόνες των μιγαδικών z1 , z2 , τότε στο διάνυσμα ΑΒ αντιστοιχεί ο μι­γαδικός z = z2 - z1 και είναι lz2 - z1 1 = IABI . Δη­λαδfι το lz2 - z1 1 εκφράζει την απόσταση των ει­κόνων των μιγαδικών.

Υ

Έτσι μια παράσταση με αθροίσματα διαw­σμάτων που αντιστοιχούν σε μιγαδικούς, μπορεί να γίνει παράσταση των μιγαδικών στους οποίους αντιστοιχούν τα διανύσματα και αντίστροφα.

Παράδειγμα: Αν z1 , z1 , z3 μιγαδικοί διαφορετικοί ανά

δύο και Α, Β, Γ οι εικόνες τους αντίστοιχα στο μιγαδικό επίπεδο, να δείξετε ότι: «Α, Β, Γ συ-

z - z • νευθειακά)) <=> 1 1 ε JR • z3 - z1

Λύση

«Α, Β, Γ συνευθειακά>> <=> ΒΑ 1 1 ΒΓ <=> «υ­πάρχει μοναδικός λ ε JR" : ΒΑ= λΒΓ <=>

Ζι -Zz =λ(Ζ:J -Zz) <=> Ζι -Zz =λ . Z:J-Zz 3η: Για τους μιγαδικούς εκείνους που δεν είναι πραγματικοί δεν ισχύουν ορισμένες ιδιότητες που γνωρίζουμε στο JR . Έστω z, w ε C -JR .

Α ν z "Φ w , δεν έχουν νόημα στο C οι σχέσεις z > w ή z < w .

Δεν έχει νόημα να γράφουμε .J;. . (τετραγω­νική ρίζα ενός μιγαδικού z είναι ο μιγαδικός w για τον οποίο ισχύει: w2 = z ). Αν z2 + w2 = 0 , ηε Ν· , δεν έπεται ότι z = w = O . (Δικαιολογήστε το με δικό σας aντιπαράδειγμα).

Παράδειγμα: Αν z = -3 + 4ί , να βρείτε μιγαδικό w έτσι,

, 1 ωστε w = z .

Λύση Έστω w = x + yi , χ, yε JR .

w2 =z <=> x2 -i +2xyi =-3+4i <=> και ! x2 -i =-3

χ..Ο 2 xy=2<=>y=­x

Άρα χ2 - y2 = -3 <=> χ4 + 3χ2 - 4 = 0 <=> χ2 = 1 <=> χ = ±ι . Για χ = -ι είναι y = -2 , οπότε w = -ι - 2i . Για χ = 1 είναι y = 2 , οπότε w = ι + 2i . Παρατήρηση: Ο w είναι τετραγωνική ρίζα του z. (Κάθε μη μηδενικός μιγαδικός έχει δύο αντίθετες τετραγωνικές ρίζες). 4η: Αξιοσημείωτες σχέσεις: α) z + z = 2Re(z) , β) z -z = 2i lm(z) , γ) lzl2 = zz , δ) zε JR <=> z = z , z φανταστικός <:::> z = -z , ε) z1 + z2 + · · · + zv = z1 + z2 + · · · + zv , - - - - -ζ) z1 z2 • • • zv = z1 • z2 • • • zv , η) mz = mz , m ε JR ,

θ) zv = (z)v, νε Ν" , ι) ( Ζι )= Ζι , z2 "Φ Ο ,

Zz Zz

ΕΥΚΛΕΙΔΗΣ Β' λ.ε. τ.l/70

Page 73: Ευκλειδης Β 41

Μαθηματικά pα την Γ' Λυκεiου κ) lzl = l-zl = lzl , λ) lz1Z2 · · · zv l = lzι l lz2 1 . . · 1zv l • μ) Ιzν Ι = lzlv . ν Ε Ν" . ν) I� = ι�ι · z * ο .

ξ) I :: I = :�: I · z2 * ο . ο) 11�1-Ι�Ιι sΙ� ±�I �Ψ,Ι+Ι�Ι · Γενικότερα lzι + Ζ2 + · · · + zv l S lzι l + lz2 1 + · · · + 1zv l • π) zΕ 1R <=>IιJ2 =r, ρ) z φανταστικός <=> lzl2 = -z2 .

Εφαρμογή :

,.... z3 + 2 3 2 Δ ίξ , .ι:.στm w = -3-- , z * . ε π on: w φα-

Ζ - 2 νταστικός <=> lzl = ifi .

Λύση

. _ r+2 «w φανταστικος» <=>w=-w(::;l _3 z -2

(zz)3 = 4 <=> lzl = Ψι = ifi . Sη: Έστω z ένας μιΎαδικός. α) Αν z0 μιΎαδικός με σταθερή εικόνα Κ, τό-τε ο γεωμετρικός τόχος των εικόνων του z που ι­κανοποιεί τη σχέση lz - Ζο I = ρ , ρ > Ο , είναι ο ιcύ­κλος με κέντρο Κ και αιcτίνα ρ. β) Αν z1 , z2 δύο μιΎαδικοί με σταθερές ει­κόνες Α, Β, τότε ο Ύεωμετρικός τόπος των εικόνων του z που ικαναιrοιεί τη σχέση lz - z1 1 = lz - z2 1 , εί­ναι η μεσοκάθετη ευθεία του ΑΒ.

Εφαρμογή : Να βρείτε το σύνολο των εικόνων του μιγα­

δικού z yια τον οποίο qουμε lzl > 2 και

lz - 2il s ι .

.\ύση lzl > 2 <=> lz- (Ο+ Οί� > 2 . Άρα οι εικόνες του

z είναι σημεία εξωτερικά του κυκλικού δίσκου που έχει κέντρο την αρχή των αξόνων και ακτίνα ρ1 = 2 . lz - 2ii :S ι <=> lz- (0+ 2i )l s ι . Άρα οι εικό­νες του z είναι σημεία του κυκλικού δίσκου με κέ­ντρο Κ (Ο, 2) και ακτίνα ρ2 = ι .

Υ. λ

χ

Συνεπώς το ζητούμενο σύνολο των εικόνων του z είναι ο μηνίσκος ΑΛΒ, εκτός των σημείων του τόξου ΑΚΒ. 6η: Έστω η εξίσωση αz2 + βz + γ = Ο , α Ε JR" , β, Ύ Ε 1R • z Ε c . Αν Δ = β2 -4αγ :S Ο , τότε η εξίσωση έχει ρίζες τις

-β ± iν'ίΔί Ειδ ' Δ Ο ίζ ' Ζ1 2 = . ικα αν < , οι ρ ες ειναι · 2α συζυγείς μιγαδικοί αριθμοί. Ζι + z2 = -./!. , ΖιΖ2 =1.. . α α 7η: Όρισμα μιγαδικού: Έστω ο μηδενικός μι­γαδικός z με διάνυσμα θέσης το ΟΜ . Όρισμα του z ονομάζουμε την γωνία που έχει αρχική πλευρά τον θετικό ημιάξονα Οχ και τελική πλευρά την η­μιευθεία ΟΜ.

M(z) Υ

------�._----�χ ο

Όταν z = Ο , δεν ορίζεται όρισμα για τον z. Το όρισμα για τον z * Ο δεν είναι μοναδικό. Αν επιλέξουμε το όρισμα που ανήκει στο [0,2π) το ονομάζουμε προτεύον όρισμα του z και το συμ­βολίζουμε Arg(z) . Έτσι για το τυχαίο όρισμα φ ενός μιγαδικού αριθμού z ισχύει:

φ = 2κπ+Αrg(z), κΕ Ζ .

Παράδειγμα:

Αν φ = 27π ένα όρισμα του z, να βρείτε το 2 Argz .

Λύση Παρατηρούμε ότι το φ δεν ανήκει στο [Ο, 2π) ,

' 27π 27π οποτε - = 2Jσt + Arg(z) <=> Arg(z) =---2ιcπ ( ι), 2 2 με Κ Ε Ζ .

'Ε χουμε Ο :S Argz < 2 <=> Ο :S 27π -2Jσt < 2π <=> 2 27π 23π 23 27 ' -- :S -2κπ < -- <=>- < κ :S - (2). Επειδη 2 2 4 4 κ Ε Ζ από τη (2) προιcύπτει ότι κ = 6 . Άρα από την (ι) παίρνουμε: Arg(z)= 3π . 2

Άλλο ' 27π ι3 π ι2 π ς τροπος: τ= +"2= π+π+"2=

ΕΥΚΛΕΙΔΗΣ Β' λ.ε. τ.ΙΠΙ

Page 74: Ευκλειδης Β 41

Μαθηματικά για την Γ Λυκείου

3π ' 3π 2 · 6π+- . Άρα Arg (z) =- . 2 2 8η: Τριγωνομετρική μορφή μιγαδικού: Έ­στω ο μιγαδικός z = α+ βi , α, β ε � με μέτρο

lzl = ρ και ένα όρισμα το φ. Η μορφή z = ρ ( σuνφ + iημφ) , ονομάζεται τριγωνομετρική ή πολική μορφή του z.

Η τριγωνομετρική μορφή ενός μιγαδικού δεν είναι μοναδική. Το όρισμα φ είναι τέτοιο ώστε: σuνφ = � και ημφ = � . ρ ρ

Παράδειγμα: Να γράψετε τον z = 1 - i με τριγωνομετρι­

κή μορφή.

Λύση Βρίσκουμε το μέτρο του z, ρ = .J2 .

Ο z γράφεται z =.fi( �-� i} Βρίσκουμε γωνία φ τέτοια, ώστε

1 σuνφ =-.J2

π και => φ = --. 4 1 ημφ = -

.J2

Άρα z = .fi[ σuν( -� )+ iημ(-: )] . Βασικές προτάσεις: 1 η) Η μορφή z = λ ( συνφ + ίημφ) είναι τρι-

γωνομετρική μορφή του z αν και μόνο αν λ > Ο • Παράδειγμα: Ν α βρείτε το μέτρο και το όρισμα του z = 1 + συνφ - ίημφ . Λύση Έχουμε 2 2 φ 2" φ φ 2 φ( φ . φ ) z = σuν -- ιημ-σuν-= σuν- σuν--ιημ-2 2 2 2 2 2

z = 2σuν � [ σuν(-� )+ iημ(-� )] ( 1 ). Διακρί-νουμε τις περιπτώσεις:

1η)Αν σuν φ > Ο , τότε λόγω της (1 ) το μέτρο 2

του z είναι ρ = 2σuν φ και ένα όρισμα το - φ . 2 2 2η) Αν σuν φ < Ο , τότε 2

z = -2συν � [-συν(-� )- iημ(-� )] �

z = -2σuν �[ σuν( π-� )+iημ( π-�)] . Άρα το μέτρο του z είναι ρ = -2σuν φ και ένα ό-2 ρισμα το π - φ . 2

3η) Αν σuν φ = Ο , τότε z = Ο , οπότε έχει μέ-2 τρο το μηδέν και δεν έχει όρισμα. 2η)Αν z, w δύο μιγαδικοί με ορίσματα φ, θ

f lzJ = I� αντίστοιχα τότε ισχύει: z = w � και

Αξιοσημείωτες σχέσεις: φ-θ=2ισr, ΚΕ Ζ

Έστω z = ρ1 ( συνφ+iημφ) , w=ρ2 ( συνθ+iημθ) οι τριγωνομετρικές μορφές των z, w. α) zw = ρ1ρ2 [ σuν(φ +θ )+ iημ(φ+ θ)] , β) _!_ = -1 [σuν (-θ )+ iημ (-θ)] , w :#= Ο ,

W ρz γ) ..:.. =Ει[σuν(φ -θ)+ iημ (φ - θ)] , w :#= O ,

W ρz δ) zκ = ρ� [ σuν(κφ) + iημ (κφ)] , κε Ζ (τύποςDe Moiνre). 9η: Μιγάδας στροφής: Έστω δύο διαφορετικοί μιγαδικοί z1 , z2 με εικόνες Α, Β αντίστοιχα και ο μιγαδικός w = σuνθ + iημθ .

Α ν z2 - z1 = ρ ( σuνφ + iημφ) η τριγωνομετρι­κή μορφή του z2 - z1 , παρατηρούμε ότι (z2 - z1 )w = ρ[ σuν(φ +θ)+ iημ(φ +θ)] (1 ).

Στο μιγαδικό z2 - z1 αντιστοιχεί το διάνυσμα ΑΒ . Η σχέση (1) μας λέει ότι ο μιγαδικός ( z2 - z1 ) w αντιστοιχεί σ' ένα διάνυσμα ΑΓ με IAΓI = ρ , το ο­

ποίο προκύπτει από τη στροφή του ΑΒ περί το Α (στο επίπεδο) κατά γωνία θ. Άρα ( z2 - z1 ) w = z3 - Ζ1 , ό-που z3 ο μιγαδικός με εικόνα το Γ.

ΕΥΚΛΕΙΔΗΣ Β' λ.ε. τ.l/72

Page 75: Ευκλειδης Β 41

Μαθηματικά Ύια την Γ Λυκείου

Υ

--r-----------� x ο

Η ιδιότητα αυτή του w τον χαρακτηρίζει ως μιγάδα στροφής. Γενικότερα κάθε μιγαδικός με μέτρο τη μονάδα είναι μιγάδας στροφής.

Παράδειγμα: Δίνονται οι μιΎαδικοί z1 = 1 + 2i και

z2 = 2 + i με εικόνες Α, Β αντίστοιχα. Ν α βρείτε

στο μιΎαδικό EJrUtεδo σημείο Γ τέτοιο, ώστε το τρίΎωνο ΑΒΓ να είναι ισοσκελές με ΑΒ = ΑΓ

Λ π και ΒΑΓ = - .

4 Λύση Στο ΑΒ αντιστοιχεί ο μιγαδικός z2 - z1 = 1 - ί .

Έστω z3 ο μ.ryαδοcός που έχει εικόνα το Γ. Στο ΑΓ αντιστοιχεί ο μryαδικός z3 - z1 • Το σημείο Γ, αφού IABI = IAfi , χροιcύπτει από τη στροφή του διαν6-σματος ΑΒ περί το Α κατά γωνία ±� . 4

Άρα έχουμε z3 - z1 = (z2 - z, >[συν(±: )+ ίημ( ±: )] .

• z3 - z, = (ι - ί)( συν�+ ίημ� }=> z3 - z, = ..fi <=> z3 = z, + ..fi <=> z3 = 1 + ..fi + 2ί .

Άρα Γ(I + ..fi,2) .

• z3 - z1 = (Ι - ί)[ συν(-:)+ ίημ(-: )] <=>

z = zi . Να βρείτε το σημείο Α όταν το Μ είναι μέσον του ΒΓ. γ) Αν το Α διαγράφει τον κύκλο με κέντρο την εικόνα του ί και ακτίνα 2, να βρείτε το σύνολο των σημείων Ν για τα οποία το τετράπλευρο ΑΒΝΓ εί-ναι ρόμβος. (Baccalaureales 1988).

2η: Έστω οι μιγαδικοί z1 , z2 , z3 με εικόνες Α, Β, Γ αντίστοιχα και lzι l = lz2 1 = l z3 1 = ρ > Ο . α) Δείξτε ότι: lzιz2 + z2z3 + zJzι l = ρ lzι + z2 + zJ I · β) Α ν το ΑΒΓ είναι τρίγωνο, δείξτε ότι: ΑΒΓ ι­σόπλευρο τρίγωνο � Ζ1 + z2 + z3 = Ο .

3η: Έστω z, w δύο μιγαδικοί με lzl = 1 και

_!_ = z2 - z + 1 . Α ν Ρ, Q οι εικόνες αντίστοιχα w

των μιγαδικών z, w και Ο < Arg(z) < π δείξτε ό-3 τι ο θετικός ημιάξονας Οχ διχοτομεί την γωνία των

- -διανυσμάτων ΟΡ , OQ .

4η: Θεωρούμε το τριώνυμο φ(z) = z2 + αz + β , α Ε JR , β Ε JR * , z Ε C και τις παραστάσεις ( πο­λυώνυμα) f (z) = z2v + ανzν + βν , P (z) = zv + 1 + (1 + z)v , όπου ν άρτιος θετικός α­κέραιος. Αν 4β > α2 και z1 , z2 είναι κοινές ρίζες των φ(z), f (z) , τότε: 1.) 'ξ , Ζι z2 δ ζ Ρ ( ) Δει τε οτι οι -- , μη ενί συν το z

z2 Ζι (ρίζες του Ρ (z) ).

Ζ3 - z, = -..fii <=> z3 = z, -..fii <=> Ζ3 = 1 + (2 -.J2)i ii) Α ν θ ένα όρισμα του � δείξτε ότι: . Άρα Γ(1, 2-..fi) . Ζ2

Τελικά έχουμε δύο σημεία Γ(l + .J2,2) ή Γ(l, 2 -.J2) .

ΕΝΟΤΗΤ Α 2η: ΑΣΚΗΣΕΙΣ

lη: Έστω Α η εικόνα του μιγαδικού Ζι Ε c* και Β, Γ τα σημεία που βρίσκονται στον κύκλο με κέ­ντρο την αρχή των αξόνων και ακτίνα ΟΑ έτσι, ώ­στε το τρίγωνο ΑΒΓ να είναι ισόπλευρο. α) Να γράψετε τους μιγαδικούς z2 , z3 που έ­χουν εικόyες τα σημεία Β, Γ αντίστοιχα, σε σχέση με τον z1 • β) Θεωρούμε Μ την εικόνα του μιγαδικού z με

συν νθ + 2ν-1συνν � = Ο . 2 2

5η: Θεωρούμε την εξίσωση (συν2θ)z2 - 2(συνθ)z + 2 - συν2θ = Ο ( 1 ),

θ ε (Ο, � ) μz άγνωστο τον z. i) Να επιλύσετε την

( 1). ii) Να δείξετε ότι οι εικόνες των ριζών της (1 ) διαγράφουν τον ένα κλάδο υπερβολής της οποίας να βρείτε την εξίσωση. iii) Αν z1 είναι η ρίζα της (ι) με θετικό φανταστικό μέρος, να βρείτε την ε­λάχιστη τιμή θετικού ακεραίου ν για τον οποίο ο

ΕΥΚΛΕΙΔΗΣ Β' λ.ε. τ.l/73

Page 76: Ευκλειδης Β 41

Μαθηματικά για την Γ' Λυκείου (z1 - 2εφθ - 1 )ν είναι θετικός αριθμός (πραγμα­συνθ τικός).

1 6η: Για τον μιγαδικό z έχουμε z + - = -1 ( 1 ). z

' ' 2001 1 Θεωρουμε τους μιγαδικους w = z + 2001 και

2004 1 ' ' u = z + 2004 • α) Δείξτε οτι: z w = u . β) Να βρείτε το Re(z).

z 1 i) z = - ii) z

7η: α) Για κάθε μιγαδικό αριθμό z δείξτε ότι: -lzl ::;; Re(z) ::;; lzl , - lzl ::;; Im(z) ::;; l zl .

β) Θεωρούμε τους μιγαδικούς z = 2χ + yi με χ ακέραιο και y > Ο για τους οποίους ισχύει

I z + 91 = 3 1 z + 11 . Ν α βρείτε τους μιγαδικούς z. ( 1 )2ν , , z + 8η: Θεωρουμε τους μιγαδικους w = --z + 2

με z ε C - {-2} , ν ε Ν* . Αν για τον z ισχι)ει lz + �� = � ( 1 ), τότε: α) Να βρείτε τον γεωμετρικό

τόπο των εικόνων του z. β) Δείξτε ότι: i) (z + 1) (z+ 2) + (z + 2) (z + l) = O , ii) w ε IR. .

' ( )2004 1 + i ' ' 9η: Δινεται z + 1 = --. ( 1 ). Να δείξετε οτι: 1 - ι i) Ο z δεν είναι πραγματικός. ii) Im (z) < Ο .

10η: α) Δείξτε την ισοδυναμία:

lz1 + z2 1 = Jlz1 1 2 + lz2 1 2 <:Ξ:> z1z2 φανταστικός. β) Για τους μιγαδικούς z1 , z2 , w, u έχουμε ότι ο w είναι φανταστικός, ο u είναι πραγματικός και ισχύουν

I 12 _ 2 2 _ 1 12 w z1 + wz1 = wz1 + w z1 I 12 _1 12 _ 2 _1 12 2 u z2 + u z2 + uz2 = -u z2 - uz2

(1 ),

(2),

Δ ΙιfΕΤΙΣτ� Κ�/ ΕΑ�ΧΙΣτιf /tiErPD/1' ΚΑΙ IJPΙΣJIR'/211 ΙιΙΙΤΑ.ΙΙ'ΚD'ΙΙ

Εισαγωγή • Έστω z, z1 , z2 μιγαδικοί αριθμοί με εικόνες

στο μιγαδικό επίπεδο τα σημεία Μ, Μ1 και Μ2 αντίστοιχα. Είναι γνωστό ότι: (βλ. Σ χ. 1 )

Υ

ο χ

Σχ. 1

Υ

Σχ. 2 Ως μέτρο του z ορίζουμε την απόσταση του Μ από την αρχή Ο, δηλαδή lzi = (OM)=Io� .

χ

Το μέτρο της διαφοράς των μιγαδικών z1 και z2 είναι ίσο με την απόσταση των εικόνων τους, δηλαδή lz1 - z2 1 = (Μ1Μ2 ) .

- Για τα μέτρα των z1 και z2 ισχι)ει 1 :

1 Γνωστή ως τριγωνική ανισότητα

του Γιώργου Κατσούλη

J lzi l - lz2 11 ::;; lzt + z2 l ::;; lzt l + lz2 l · - Το πρωτεύον όρισμα Arg(z) του μιγαδικού z

είναι το μέτρο της γωνίας που σχηματίζει η διανυσματική ακτίνα ΟΜ με τον άξονα χ'χ . Το πρωτεύον όρισμα ενός μιγαδικού αριθμού z είναι πάντα ένας αριθμός του διαστήματος [0,

2π). Δηλαδή: Αν Arg(z) = θ , τότε Ο ::;; θ < 2π . • Επίσης, από τη Γεωμετρία γνωρίζουμε ότι αν

το σημείο Μ διατρέχει: Μια ευθεία ε. (Σχ. 2) Στην περίπτωση αυτή το Ιzl δεν παίρνει μέγι­στη τιμή. Η ελάχιστη τιμή του Ιzl είναι

d (Ο, ε) = (ΟΜ) , όπου ΟΜ j_ ε . Οι συντεταγμέ­νες του σημείου Μ προκύπτουν από τη λύση του συστήματος των ευθειών ε και ΟΜ.

Ένα κύκλο (Κ, ρ) (Σ χ. 3) Τότε

α) Η ελάχιστη τιμή του Ιzl είναι: (ΟΑ) = Ι<οκ) - (ΚΑ)I = Ι(οκ) - ρl

Η μέγιστη τιμή του Ιzl είναι: (ΟΒ) = (ΟΚ) + (ΚΒ) = (ΟΚ) + ρ

ΕΥΚΛΕΙΔΗΣ Β' λ.ε. τ.l/74

Page 77: Ευκλειδης Β 41

Μαθηματικά Ύια την Γ' Λυκείου

'�

Υ

®;Μι Β Μ • οι χ ο χ

Σχ. 3 Σχ. 4

Παρατήρηση: Σύμφωνα με το σχήμα αυτό έχουμε:

• Η ελάχιστη Arg( z) είναι το μέτρο της γωνίας χόΓ

• Η μέγιστη τιμή του Arg( z) είναι το μέτρο της γωνίας χΟΔ

β) Η ελάχιστη τιμή του lz - z, l είναι: (Σύμφωνς με το Σχ. 4) (ΜιΑ)= I(Μ,Κ}- ρl

Η μέγιστη τιμή του lz - z, l είναι: (ΜιΒ) = (Μ,Κ}+ ρ

Οι συντεταγμένες των σημείων Α και Β προ­κύπτουν από τη λύση του συστήματος του κύκλου και της ευθείας ΟΚ ή Μ1Κ αντίστοιχα. • Τέλος, αν τα σημεία Μι και Μ2 διατρέχουν: - Δύο κύκλους (Κ,R) και (Λ,ρ) αντίστοιχα με

ΚΛ > R + ρ (Σχ. 5)

Σχ. 5 Σχ. 6 Τότε η ελάχιστη τιμή του lzι - z2 1 είναι

(ΑΒ) = (ΚΑ) - R - ρ ενώ η μέγιστη τιμή lzι - Ζ2 1 είναι (ΓΔ) = (ΚΛ)+ R + ρ .

Οι συντεταγμένες των σημείων προκύπτουν όπως παραπάνω. - Δύο παράλληλες ευθείες ε1 και ε2 αντίστοιχα

(Σχ. 6) Τότε η ελάχιστη τιμή lzι - z2 1 είναι d (Ει , Ε2 ) Μέγιστη τιμή του lzι - z2 1 δεν υπάρχει

Ενότητu 1 11� A«Jt(ή«:rι\� μι μέτρα 1) Α ν yια τον μιγαδικό z ισχύει

lz + il = lz - 1 + 2il να βρείτε: i) Τη γραμμή που διαγράφει η εικόνα Μ

του z. ii) Την ελάχιστη τιμή του lzl . iii) Τον μιγαδικό z που έχει το ελάχιστο μέ-

τ ρο.

Λύση (Σχ. 7) i) Έστω z = χ + yi . Έχουμε

lz+il = lz-1 +2il <=>lx +(y+ ΨI = l(x -1)+(y+2)il <=>

�x2 + (y + 1)2 =�(x - 1)2 + (y + 2)2 <=> . . · <=>

χ - y '-- 2 = Ο . Άρα το Μ ανήκει στην ευθεία ε : χ - y- 2 = 0 .

Υ (ε)

χ

Υ 3

4 ····· .. : κ Β

Σχ. 7 Σχ. 8 ii) Η ελάχιστη τιμή του Ιzl είναι:

d (Ο, ε) = Ιο - ο - 21 =_}:__ = J2 . .jί2;ϊ2 J2

χ

iii) Επειδή ΟΜ 1. ε είναι λ0Μ = -1 , οπότε η εξίσωση της ΟΜ είναι y = -χ . Οι συντεταγμένες

του Μ είναι η λύση του συστήματος {Υ = -χ x - y - 2 = 0

που είναι το ζεύγος ( 1 ,-1) . Άρα ο μιγαδικός που έχει το ελάχιστο μέτρο είναι z = 1 - i .

2) Αν yια τον μιγαδικό z ισχύει lz - 3 + 4il = 2 , να βρεθεί:

i) Η γραμμή που διαγράφει η εικόνα Μ του z.

ii) Η μέγιστη και ελάχιστη τιμή του lz l .

Λύση (Σχ. 8) i) Έχουμε lz - 3 + 4il = 2 <=> lz - (3 -4i)l = 2 .

Άρα το Μ κινείται σε κύκλο με κέντρο Κ ( 3,-4) και ακτίνα ρ = 2 .

ii) Η ελαχίστη τιμή του Ιzl είναι (ΟΑ) = (ΟΚ) - ρ = 5 - 2 = 3

Η μέγιστη τιμή του Ιzl είναι (ΟΒ) = (ΟΚ) +ρ = 5 + 2 = 7 .

Πρόβλημα Να βρείτε τους μιγαδικούς Ζι , z2 που έχουν

το μικρότερο και μεγαλύτερο μέτρο.

3) Αν για τον μιγαδ!κό z ισχύει z (1 - λi) = 4 - 2i, λε JR να βρείτε:

i) Τη γραμμή που διαγράφει η εικόνα Μ του z.

ii) Τους αριθμούς λ και z ώστε ο z να έχει το μεγαλύτερο κατά το δυνατό μέτρο.

ΕΥΚΛΕΙΔΗΣ Β' λ.ε. τ.l/75

Page 78: Ευκλειδης Β 41

Μαθηματικά Ύια την Γ Λυκείου

Λύση (Σχ. 9) ί) Έστω z = x + yi χ, yε JR . Τότε (x + yi )Cι -λi) = 4-2i �

x+yi -λχi +λy=4-2ί � (x-4+λy)+(y+2-Aχ)i=O. , {χ -4+λy = Ο Πρεπει y + 2-λχ = Ο

Με απαλοιφή του λ παίρνουμε την εξίσωση χ2 - 4x + y2 + 2y = O ή (χ - 2)2 + (y + 1)2 = 5 . Άρα το Μ κινείται σε κύκλο με κέντρο Κ ( 2,-1) και ακτίνα ρ = .J5 . Ο γεωμετρικός τόπος που γρά­φει η εικόνα Μ του z είναι ο παραπάνω κύκλος ε­κτός του 0(0, 0).

ίί) Επειδή ο κύκλος διέρχεται από το Ο (Ο, Ο) το μεγαλύτερο μέτρο αντιστοιχεί στο μιγαδικό z με εικόνα το aντιδιαμετρικό σημείο Α του Ο ως προς Κ. Αφού το σημείο Κ ( 2,-1) είναι το μέσο του ΟΑ θα είναι Α(4,-2) . Άρα z = 4- 2i και προφα­νώς λ = Ο .

Υ Υ

χ

χ

Σχ. 9 Σχ. 10

4) Αν για τον μιγαδικό z ισχύει lz - i l = 1 να αποδείξετε ότι 4 :5 lz + 4 + 2il :5 6 .

Λύση (Σχ. 10)

Έχουμε lz + 4 + 2il = l(z - ί) + (4 + 3i)l :5 lz - il + 14 + 3il = 1 + 5 = 6 και lz + 4 + 2il = l(z - ί) + (4 + 3i)l ;?: llz - il - 14 + 3ill = 11 - 51 = 4 .

Επομένως 4 :5 lz + 4 + 2il :5 6 . 2°ς τρόπος: Επειδή lz - il = 1 η εικόνα Μ του z κινείται σε

κύκλο με κέντρο Κ (Ο, 1) κ�ι ακτίνα ρ = 1 . Επίσης lz + 4 + 2il = lz - (-4- 2ί )i = (MA) , ό­

που Α(-4,-2) . Επομένως η ελάχιστη τιμή του

lz + 4 + 2il είναι (ΑΒ) = (ΑΚ) - ρ = 5 - 1 = 4 και η

μέγιστη τιμή του lz + 4 + 2il είναι ΑΓ = (ΑΚ) + ρ = 5 + 1 = 6 . Άρα 4 :5 lz + 4+ 2il :5 6 .

Πρόβλημα: Για ποιους μιγαδικούς z ισχύουν οι ισότητες;

5) Α ν για τους μιγαδικούς z1 , z2 ισχύουν

οι σχέσεις lz1 + 3il = 1 και lz2 -41 = 2 να βρείτε

την μέγιστη και ελάχιστη τιμή του lzι - z2 1 ·

Λύση (Σχ 11)

Αφού lzι + 3il = 1 η εικόνα Μι του Ζι κινείται σε κύκλο με κέντρο Κ (Ο,-3) και ακτίνα ρ = 1 . Ό­μοια η εικόνα Μ2 του z2 κινείται σε κύκλο με κέντρο Λ ( 4, Ο) και ακτίνα R = 2 .

Άρα η ελάχιστη τιμή του lzι - z2 1 είναι (ΑΒ) = (ΚΛ) - R - ρ = 5 - 2 - 1 = 2 και η μέγιστη τιμή του lzι - z2 1 είναι

(ΓΔ) = (ΚΛ) + R + ρ = 5 + 2 + 1 = 8 .

Υ

χ

(C) Σχ. 1 1 Σχ. 12

Πρόβλημα: Για ποιους μιγαδικούς Ζι , z2 ισχύουν οι ισό­

τητες;

6) α) Να βρείτε το σύνολο των σημείων του μιγαδικού επιπέδου που είναι εικόνες των μιγαδικών z οι οποίοι έχουν την ιδιότητα: «Ο λόγος των aποστάσεών τους από τα σταθερά σημεία z1 = 3 και z2 = -3 να είναι σταθερός και ίσος με 2».

β) Αν για τους μιγαδικούς w1 και w2 lw. - 3 ι ιw2 - 3 ι 2 β • • • -- = = , να ρειτε τη μεyιστη τιμη w1 + 3 W2 + 3

του lw1 - w2 1 . Λύση (Σχ. 12) α) Έστω Μ ένα σημείο του επιπέδου που εί­

ναι εικόνα του z = χ + yi .

Τότε έχουμε 1lz - 3

11 = 2 � lz- 3l = 2lz + 31 �

z+ 3 l(x-3)+yil = 2l(x +3)+yil � · · ·� (χ+5)2 +y2 = 16 . Επομένως τα σημεία Μ κινούνται σε κύκλο με κέντρο Κ( -5,0) και ακτίνα ρ = 4 .

β) Από το (α) οι μιγαδικοί w ι , w 2 έχουν ει­κόνες, έστω, τα σημεία Α, Β του κύκλου C. Επειδή lwι - w2 I = (AB) και η χορδή ΑΒ είναι μικρότερη

ΕΥΚΛΕΙΔΗΣ Β' λ.ε. τ.l/76

Page 79: Ευκλειδης Β 41

Μαθηματικά για την Γ Λυκεiου

ή ίση της διαμέτρου 2ρ = 8 θα έχουμε /w1 - w2 / � 8 . Άρα η μέγιστη τιμή του /w1 - w2 / , είναι και πραγματοποιείται αν τα Α και Β είναι a­ντιδιαμετρικά σημεία του κύκλου.

Ενότητα 2η: Ασκήσεις με πρωτεύον όρισμα

1) Αν για τον μιγαδικό z ισχύει lz - il =..!. 2

να βρείτε: i) Τη γραμμή που διαγράφει η εικόνα Μ του z.

ii) Τη μέyιστη και ελάχιστη τιμή του

Arg(z) . iii) Τους μιγαδικούς z που έχουν το μιιι::ρό­

τερο και μεγαλύτερο πρωτεύον όρισμα.

Λύση (Σχ. 13) ·> Α · ι · ι 1 · Μ · ι φου z - ι = 2 οι εucονες του z κινου-

νται σε κύκλο με κέντρο Κ (Ο, 1) και ακτίνα ρ = _!_ . 2

ίί) Το Arg(z) είναι η γωνία που σχηματίζει η διανυσματική ακτίνα ΟΜ με τον άξονα χ'χ . Άρα η ελάχιστη τιμή του Arg (z) είναι το μέτρο της

Λ Λ

γωνίας χ Ο Α = θ και η μέγιστη τιμή του Arg (z)

Λ Λ είναι το μέτρο χΟΒ=ω .

Αρκεί λοιπόν να βρούμε τις εφαπτόμενες του κύκλου που διέρχονται από την αρχή των αξό­νων. Η εφαπτόμενη ε θα είναι της μορφής ε : y = λχ <=> λχ - y = Ο . Πρέπει d (Κ,ε) = ρ ή

l-11 1 2 r;; � = "2 <=> λ + 1 = 4 <=> λ = ±ν3 .

Άρα εφθ = J3 , οπότε η ελάχιστη τιμή του

Arg (z) είναι � και εφω = -J3 , οπότε η μέγιστη 3

' Ar ( ) ' 2π τιμη του g z ειναι 3 . Υ

χ

Σχ. 13 χ

iii) Λύνοντας το σύστημα των εξισώσεων του

κύκλου και της ευθείας βρίσκουμε Α(�,�). ο­πότε z = J3 + �i και Β(- J3 �) οπότε 1 4 4 4 ' 4 '

J3 3 . z = --+-ι . 2 4 4

2) Αν lz - 2- 2il = 2 να βρείτε τους μιγαδι­κούς z που έχουν το μικρότερο και μεγαλύτερο πρωτεύον όρισμα.

Λύση (Σχ. 14) Είναι z1 = 2 + Oi και Argz1 , που είναι το μι­

κρότερο πρωτεύον όρισμα. Επίσης z2 = Ο+ 2i και

Argz2 = � που είναι το μεγαλύτερο πρωτεύον ό­ρισμα.

3) Αν για τον μιγαδικό z ισχύει

2001 Im(z) 1 1 r::;:. l , ------=-2- +- 2001 + 2001 · ν3ι = 0 να βρειτε: lz - 11 2 i) Τη γραμμή που διαγράφει η εικόνα Μ

του z. ii) Τον μιγαδικό z που έχει το μεγαλύτερο

πρωτεύον όρισμα. Μπορούμε να βρούμε τον μι­γαδικό που έχει το μικρότερο πρωτεύον όρισμα;

Λύση (Σχ. 15) i) Έστω z = χ + yi χ, y ε 1R . Με αντικατά-

σταση στη δεδομένη σχέση προκύπτει 2001 Υ + _!_ 200 1.Ji+3 = ο <=>

(χ - 1)2 + y2 2 y2 + 1 = 0 <=:> (x - 1)2 + y2 + y = 0 <=>

(χ - 1) + y2

(χ - 1)2 +(y+� )2 =� . Άρα το Μ κινείται σε κύ-

κλο με κέντρο κ(1,-�) και ακτίνα ρ =� . Όμως,

επειδή z :;t: 1 πρέπει (x ,y) :;t: (l,O) . Έτσι εξαιρούμε από τον κύκλο το σημείο Α ( 1, Ο) .

ii) Η ΟΑ είναι εφαπτόμενη του κύκλου C. Ε­πειδή το Α εξαιρείται, δεν υπάρχει μιγαδικός z που να έχει ελάχιστο πρωτεύον όρισμα. Η εφαπτό­μενη ΟΒ είναι της μορφής y = λχ . Από το σύστη-fy = λχ, λ :;t: Ο

μα 2 ( 1 )2 1 προκύπτει το τριώνυ-(χ - 1) + y+2 = 4

ΕΥΚΛΕΙΔΗΣ Β' λ.ε. τ.l/77

Page 80: Ευκλειδης Β 41

Μαθηματικά για την Γ Λυκείου

μο (ι + λ?' )χ2 + (λ- 2)χ + ι = ο . Για να εφάπτεται η ευθεία y = λχ στον κύκλο

πρέπει2 Δ = Ο <=:> (λ-2)2 - 4(1 + λ2 ) = 0 , οπότε 4 λ = -3 ( λ :;e Ο ).

Υ

χ

(C)

Σχ. 1 5 Σχ. 1 6 χ

λ 4 , 3 4 Ε Για = -- προ1αιπτει χ =- και y = -- . -3 5 5

' δ ' 3 4 . ' αλ ' πομενως ο μιγα ικος z = 5'- 5ι εχει το μεγ :υτε-ρο πρωτεύον όρισμα.

4) Θεωρούμε τους μιγαδικούς αριθμούς z = λ + (λ2 + t)i, λ ε IR. . Να βρείτε:

i) Τη γραμμή που διαγράφει η εικόνα Μ του z.

ii) Τους μιγαδικούς z που έχουν το μικρότερο και μεγαλύτερο πρωτεύον όρισμα.

Λύση (Σχ. 16) i) Είναι Μ {λ,λ?' + 1) . Επομένως χ = λ ,

y = λ2 + 1 . Με απαλοιφή το:υ λ παίρνουμε y = χ2 + 1 . Άρα το Μ κινείται στην παρεμβολή με εξίσωση y = x2 + 1 .

ii) Αρκεί να βρούμε τις εφαπτόμενες της C : y = χ2 + 1 πο:υ διέρχονται από την αρχή των α­ξόνων.

Θέτο:υμε3 f (χ) = χ2 + 1 , οπότε f'(x) = 2χ . Η εξίσωση της εφαπτόμενης ε της Cr σε ένα σημείο της M(x0 ,f (x0 )) είναι y - (χ� + 1) =

2χ0 (χ - χ0 ) • Η ευθεία διέρχεται από το Ο (Ο, Ο) , αν και μόνο αν Ο - (χ� + 1 ) = 2χ0 (Ο - χ0 ) <=> χ� = ι <=> Χο = ±1 . Άρα Υ ο = χ� + 1 = 2 . Επομένως Α ( 1, 2) και ο μιγαδικός z1 = 1 + 2i έχει το μικρό­τερο πρωτεύον όρισμα. Όμοια Β(-1,2) και ο z2 = -1 + 2i έχει το μεγαλύτερο πρωτεύον όρισμα.

2 Στην ουσία θέλουμε η ευθεία να έχει ένα μόνο κοινό σημείο με τον κύκλο 3 Η διαδικασία είναι μέρος της Ανάλυσης, που την "(\{Ο­ρίζουμε χονδρικά στα μαθήματα Γενικής Παιδείας, ενώ αργότερα στην κατεύθυνση θα μελετηθεί σε βάθος.

1. Αν Arg(z + 1) = π , να βρείτε τη μικρότε-4 ρη τιμή το:υ lzl .

J2 (Απαν.: - ) 2

2. Aν. lz - il = l να βρείτε τη μικρότερη και τη μεγαλύτερη τιμή το:υ lz + ιι .

(Απαν.: J2 - 1, J2 + 1 )

3. Να βρεθεί ο μιγαδικός z με το μέγιστο δ:υ­νατο μέτρο αν είναι γνωστό ότι: 2 lzl2 = (1 - i)z + (l + i)z .

(Απαν.: z = l + i )

4. Αν lz-'-"3 - il = 1 να βρείτε: i) Τη μέγιστη και ελάχιστη τιμή το:υ Arg(z) .

ii) Τους μιγαδικούς που έχουν το μικρότερο και μεγαλύτερο πρωτεύον όρισμα.

(Απαν.: z = '-"3 + �i , z = '-"3 + Oi ) 2 2

5. Αν z1 = -λ + (λ- Ι)ί και z2 = (λ- 3)�λi , λ ε IR. να βρείτε τη μικρότερη τιμή του lz1 - z2 1 .

(Απαν.: J2 ) 6. i) Αν lz + ll ::::: 2lz - 21 να βρείτε τη

γραμμή που διαγράφει η εικόνα Μ του z.

ii)A ν I Ζι + · ι = I Zz + · ι = 2 να βρείτε τη μεγα-Ζι - 2 z2 - 2 ·

λύτερη τιμή του lz1 - z2 1 . (Απαν. : 4)

7. Αν z -λ = eλi , λ ε IR. να βρείτε: i) Τη γραμμή που διαγράφει η εικόνα Μ του z. ii) Τον μιγαδικό z που έχει το μικρότερο

πρωτεύον όρισμα. (Απαν.: y = ex , z = l + ei )

ΕΥΚΛΕΙΔΗΣ Β' λ.ε. τ.l/78

Page 81: Ευκλειδης Β 41

Η Homo Mathematicus είναι μια στήλη στο περιοδικό μας, με σκοπό την ανταλλαγή απόψεων και την ανάπτυξη προβληματισμού πάνω στα εξής θέματα: Ι ) Τι είναι τα Μαθηματικά, 2) Πρέπει ή όχι να διδάσκονται, 3) Ποιοι είναι οι κλάδοι των Μαθηματικών και ποιο το αντικείμενο του καθενός, 4) Ποιες είναι οι εφαρμογές τους, 5) Ποιες επιστήμες ή κλάδοι επιστημών απαιτούν καλή γνώση των Μαθηματι­κών για να μπορέσει κάποιος να τους σπουδάσει. Κανόνας: Η στήλη Homo Mathematicus απευθύνεται μόνο σε μαθητές.

Σ το προηγούμενο τεύχος είδαμε τη σχέση

. ανάμεσα στα Μαθηματucά και στις εύστο­χες "καλαθιές" στο μπάσκετ. Λίγες μέρες

αργότερα ( αφότου κυκλοφόρησε ο 'ΈΥΚΛΕΙΔΗΣ Β'"), δύο αγαπητοί μου μαθητές, με περίμεναν στην αυλή του Σχολείου καταϊδρωμένοι και α­σθμαίνοντες. Και οι δύο είχαν επιχειρήσει να ε­φαρμόσουν στην πράξη αυτά που, διάβασαν, στο σχετικό άρθρο. Τα αποτελέσματα του εγχειρήμα­τός τους ήσαν .. . προδιαγεyραμμένα. Τους εξήγησα πως α) δεν έλαβαν υπ' όψη τους την αντίσταση του αέρα (αλλάζει τη συμπεριφορά της μπάλας) και β) ότι έπρεπε να είναι εφοδιασμένοι με . . . γωνιόμετρο ... Τους πρότεινα να εξασκήσουν τις μαθηματικές τους γνώσεις στο μπιλιάρδο.

Ασχολούνται οι Μαθηματικοί με το μπιλιάρδο; Όσο παράξενο κι αν σας φανεί, Μαθηματικοί

ερευνητές, μελετούν την τροχιά της μπάλας του μπιλιάρδου σε σενάρια τραπεζιών που έχουν τα πιο "τρελλά" καμπυλόγραμμα σχήματα. Να τι έ­γραφε ο Anatoly Saνin: «Λύσεις σε προβλήματα του συγκεκριμένου τύπου μας βοηθούν να κατανοή­σουμε τους νόμους της κίνησης των μορίων των αερίων ή δεσμών σωματιδίων σε κλειστούς όγκους, και τούτοι οι νόμοι είναι χρήσιμοι σε πολλές περιο­χές της φυσικής, ιδιαίτερα στην κβαντική ηλεκτρο­νική. Τα μόρια ανακλώνται στα τοιχώματα ακριβώς όπως μια μπάλα του μπιλιάρδου στο τοίχωμα του τραπεζιού»

Δεν πρέπει να ξεχνάμε ότι στα 1 835 ο περίφη­μος Γάλλος Φυσικός - Μηχανικός - Μαθηματικός Gaspard Coήolis έγραψε ολόκληρο βιβλίο με θέμα «Η μαθηματική θεωρία των φαινομένων του μπι­λιάρδου)), στο οποίο μελετούσε τα μη κεντρικά χτυπήματα της μπάλας. Ένα μη κεντρικό χτύπημα της μπάλας την εξαναγκάζει, μεταξύ των άλλων, και σε περιστροφική κίνηση που, εξ αιτίας της τρι­βής της με την τσόχα του τραπεζιού, καμπυλώνει την τροχιά της. Απαραίτητες γεωμετρικές γνώσεις για να γίνε-

Του Γιάννη Κερασαρίδη

τε .•• η πιο "δυνατή στέκα" Γωνία πρόσπτωσης και γωνία ανάκλασης: Στο (σχήμα 1 ) η ευθεία (ε) είναι εφαπτόμενη της κα­μπύλης C στο σημείο Α. Η ημιευθεία ΑΖ είναι κά­θετη στην ευθεία (ε) και η γωνία ΧΑ Ψ έχει διχο­τόμο την ημιευθεία ΑΖ. Αν μια μπάλα μπιλιάρδου διαγράφει την τροχιά Χ-Α-Ψ , τότε η γωνία ΧΑΖ λέγεται "γωνία πρόσπτωσης" ενώ η γωνία ΖΑ Ψ λέγεται "γωνία ανάκλασης". Από τη Φυσική (Οπτική) γνωρίζουμε ότι η γωνία πρόσπτωσης εί­ναι ίση με τη γωνία ανάκλασης.

� ε Α \

C σχ. 1

ψ

Συμμετρία ως προς άξονα: Θεωρούμε την ευθεία (ε) και ένα σημείο Α έξω απ' αυτήν (σχήμα 2). Α­πό το σημείο Α φέρνουμε μια ευθεία (ε') κάθετη στην (ε), που τέμνει την (ε) στο σημείο Ο. Πάνω στην (ε') παίρνουμε ένα τμήμα ΟΑ'=ΟΑ. Τα ση­μεία Α και Α' λέμε ότι είναι συμμετρικά μεταξύ τους με άξονα συμμετρίας την ευθεία (ε). Το ση­μείο Ο λέγεται προβολή του σημείου Α πάνω στην ευθεία (ε)

Α*�*

Α' σχ. 2

ε

Και τώρα, η ώρα του μπιλιάρδου. Α. Μπιλιάρδο σε τραπέζι σχήματος ορθογωνίου Πρόβλημα α' Στο τραπέζι (του σχήματος 3), μια μπάλα βρίσκεται στο σημείο Μ. Αν η κορυφή Δ του τραπεζιού είναι "τσέπη", ζητείται να βρεθεί η πορεία της μπάλας, για να μπει στην "τσέπη", α-

ΕΥΚΛΕΙΔΗΣ Β' λ.ε. τ.l/79

Page 82: Ευκλειδης Β 41

ΗΟΜΟ MATHEMAτiCUS

φού αυτή ανακλαστεί, υποχρεωτικά, στο τοίχωμα ΑΒ. (Σημ. "τσέπη" λέγεται η οπή που υπάρχει σε μια από τις κορυφές του τραπεζιού του μmλιάρ­δου, στην οποία, ο παίκτης, πρέπει να κατευθύνει τις μπάλες)

Απάντηση Έστω Μ-κ-Δ είναι η πορεία της μπάλας. Έ­

στω ακόμη ότι το Δ' είναι το συμμετρικό του Δ με άξονα συμμετρίας το τοίχωμα ΑΒ του τραπεζιού. Σ' αυτή την περίπτωση τα σημεία Μ, Κ, Δ' βρί­σκονται στην ίδια ευθεία. Αρκεί λοιπόν ο παίκτης να βάλει την στέκα στην προέκταση του ευθύ­γραμμου τμήματος Δ'Μ (προς το μέρος του Μ) και να χτυπήσει κεντρικά τη μπάλα. Βέβαια εδώ μπαίνει ένα ερώτημα· πως θα εντοπίζουμε κάθε φορά τη θέση του Α' ; Υπάρχουν δύο απαντήσεις:

Η πρώτη είναι να εξασκηθεί το μάτι ως προς τη

Δ'

Α

i'",x,"-, σχ. 3

Β

θέση του σημείου Δ' σε σχέση με αντικείμενα του περιβάλλοντος χώρου (εδώ πρέπει να σημειώσου­με ότι για οποιαδήποτε θέση της μπάλας Μ το ση­μείο Δ' είναι πάντα στην ίδια θέση).

Η δεύτερη απάντηση είναι να τοποθετηθεί ένα μικρό καθρεφτάκι ( σχ. 4) με την ανακλώσα επιφά­νεια προς το μέρος του παίκτη, κάθετα προς την επιφάνεια του τραπεζιού και κατά μήκος του τοι­χώματος ΑΒ. Αρκεί να χτυπήσουμε κεντρικά τη μπάλα,

καθρέφτης

Α

ι···--·---···-----Β

Γ

σχ. 4

τοποθετώντας τη στέκα στην προέκταση της ευ­θείας που συνδέει τη μπάλα Μ με το είδωλο Δ', της κορυφής Δ, ως προς τον καθρέφτη.

Πρόβλημα β' Στην τσόχα του μπιλιάρδου ΑΒΓ Δ ( σχ. 5) υπάρχουν δύο μπάλες, μια κόκκινη Κ και μια μαύρη Μ. Ζητείται, η μαύρη μπάλα να χτυπή­σει την κόκκινη, αφού προηγούμενα ανακλαστεί πρώτα στο τοίχωμα ΓΒ και μετά στο τοίχωμα ΒΑ.

Απάντηση Θεωρούμε το παραλ/μο Α'ΒΓΔ' που είναι συμ­

μετρικό του ΑΒΓ Δ (με άξονα το ΒΓ). Θεωρούμε, ακόμη, το παραλ/μο Α'ΒΓ Δ" που είναι συμμε­τρικό του Α'ΒΓΔ' (με άξονα το Α'Β). Έστω Κ' το συμμετρικό της Κ ( με άξονα το ΒΓ) και Κ" το συμμετρικό του Κ' (με άξονα το Α'Β)

Δ�.: .................................................. Γ'

! Κ" 1',_ ! ...... Α' r �:��+"�Β::..._

______ , Κ

-ιΑ

σχ. 5

Δ' !···········································τ···'---------4 Δ

Η ευθεία που συνδέει τα σημεία Μ, Κ" τέμνει το τοίχωμα ΒΓ στο σημείο Π και το "τοίχωμα" Α'Β στο σημείο Λ'. Αν Λ είναι το συμμετρικό του Λ' (με άξονα συμμετρίας το τοίχωμα ΒΓ), τότε η σωστή πορεία της μπάλας είναι η Μ - Π - Λ­Κ. Άρα ο παίκτης, αρκεί να χτυπήσει κεντρικά την μπάλα Μ, τοποθετώντας την στέκα στην προέκταη του ευθύγραμμου τμήματος Κ' 'Μ. Οι τρόποι με τους οποίους πετυχαίνουμε κάτι τέτοιο είναι οι ί­διοι, (όπως στην περίπτωση του προβλήματος α' ), μόνο που τώρα χρησιμοποωύμε δύο καθρεφτάκια · ένα κατά μήκος του τοιχώματος ΒΓ και ένα κατά μήκος του τοιχώματος ΒΑ.

Το μπιλιάρδο και το πρόβλημα του S. Pankov

Ένας γυάλινος αγωγός φωτός έχει τη μορφή κόλουρου κώνου και είναι καλυμμένος στο εσωτε­ρικό του από άργυρο (καθρέφτης). Τα επiπεδα των βάσεων του κώνου είναι κάθετα στον άξονά του, οι διάμετροί τους είναι d, d' και το ύψος h (h>d>d'). Μια δέσμη φωτός, παράλληλη με τον άξονα του κώνου, προσπίπτει στη μεγάλη του βά­ση (η φωτεινή δέσμη δεν είναι απαραίτητο να πε­ριέχει τον άξονα του κώνου). Ερώτημα: Όλες οι ακτίνες της φωτεινής δέσμης θα βγουν από τη μι­κρή βάση αφού υποστούν πολλαπλές ανακλάσεις στα εσωτερικά τοιχώματα; [Υπόδειξη: το ότι η ε­πιφάνεια είναι κωνική δεν έχει καμιά σημασία. Αρκεί να εξετάσετε τις "ακραίες" φωτεινές ακτίνες που προσπίπτουν στο σύνορο της μεγάλης βάσης]

Στο επόμενο Στο επόμενο τεύχος θα δούμε τη γεωμετρία α)

του κυ-κλικού μπιλιάρδου, β) του ελλειπτικού μπι­λιάρδου γ) άλλων μπιλιάρδων, δ) ίσως παίξουμε base-ball.

ΕΥΚΛΕΙΔΗΣ Β' λ.ε. τ.l/80

Page 83: Ευκλειδης Β 41

Εχδόσει, τη, ΕΛΛΗΝΙΚΗΣ ΜΑθΗΜΑΠΚΙΙΣ ΕΤΑΙΡΕΙΑΣ

HELJ..L'Ioκ Rεsuικ ,

.\ft..,.,;:J(H7!'S I "roιι.:;ncs "'

_ .......... _,_ ·---

... -

Δ 8 - -

δ;�·�τοσn ι ..... ... """" ........... ·-·

--- ·� .. -· .. ·� � "" � ................ ,_<81;

f \ \IJ''>;I IOI \f\ΗΗ\)\JιΙι.ΙΙ 8ΙM\If1f'I'\ΦI \

Ι niΙ'\I�fl� \IUtιt\1\Hkliι.) HII'HU nn_it:ΎΙ"<

Σrorxι.:ιαs.•tt rαι,ιετr 'λ ,.πο ANnrεNr Σικοnι.ι.. "όΙΙ<Ολ..ι'Ι!:I: ,l'l't.\11,1..\ffi; ·-

ΜΑΘΗΜΑτΙΚΗ ι:ί:J ΕΠ ΙΘΕQΡΗΣΗ �

LOUIS βRAND

ΜΑθΗΜΑΥΙΚΗ ΑΝΑΛΥΣΗ

-.........

�·�·( ,.,. .,.,..,.,_ ,... �.�..,.. "ιι-JΙΙ!οιιι ... """'"Jιιι. QΟ'

ΕυκλείδηςΑ': Τεύχος 600δρχ. θέματα εξετάσεων στα Α.Ε.Ι 2.500 δρχ (Ονόματα Γεωμετρικών όρων Συνδρομή 3.000 δρχ. 1976 · 1989: 1 .500 δρχ. 10ου Πανελληνίου Συνεδρίου ΓΕDΜΕΥΡΙΚΑ) 6.000 δρχ. (4 ΊΕύχη + 600 δρχ. ΠΙJ(\Jδρομι<ά) Πραιmκά: 5.000 δρχ (Μετρικά • Διόπτρα) 6.000 δρχ. Σχολεία: 2.400 δρχ. (4 τιύχη) 1ου Πανελληνίου Συνεδρίου 1 1ου Πανελληνίου Συνεδρίου Διαλέξεις: ο τόμος 1 .300 δρχ. ΕυκλείδηςΒ': Τεύχος 700δρχ. 2.000 δρχ 5.000 δρχ Μαθηματική Ανάλυση Συνδρομή 3.400 δρχ. 2ου Πανελληνίου Συvεδρίου 14ου Πανελληνίου Συνεδρίου Loυis Brand 7.000 δρχ. (4 ΊΕύχιι + 600 δρχ. 'RJ)(Uδρομικά) 2.000 δρχ 5·000 δρχ Διαφορικές Εξισώσεις Σχολεία: 2.800 δρχ. (4 τιύχη) 3ου Πανελληνίου Συνεδρίου 15ου Πανελληνίου Συνεδρίου

Stephenson 2.500 δρχ. Ευκλει'δηςΓ': Τεύχος 1 .500δρχ. 2.000 δρχ 5.000 δρχ 3.000 δρχ. (2 τιύχη) 4ου-5ου Πανελληνίου Συνεδρίου 16ου Πανελληνίου Συνεδρίου Ιστορία Μαθημαnκών Loήa

Μαθημ. Επιθεώρηση: 3.000 δρχ 5.000 δρχ (4 τόμοι) Α, Β, Γ Α• Γ Β ο τόμος

Τεύχος 1 .500 δρχ. βου Πανελληνίου Συνεδρίου Συνέδριο Hermis "92 (Αγγλικά) 2.000 δρχ.

Συνδρομή 3.000 δρχ. (2 τιύχη) 2.000 δρχ 7.000 δρχ. 70 Χρόνια Ε.Μ.Ε. 1 .000 δρχ.

Αστρολάβος: Τεύχος 1 .500 δρχ. 7ου Πανελληνίου Συνεδρίου Συνέδριο Hermis "94 (Αyyλικά) Ελληνική Μαθημαnκή Συνδρομή 3.000 δρχ. (2 τιύχη) 2.500 δρχ 2 τόμοι ο τόμος 4.500 δρχ. Βιβλιογραφία 1 .000 δρχ. Διλ'ήο (Bύletίrι): Τεύχος2.000 δρχ. θου Πανελληνίου Συνεδρίου Γραμμική Άλγεβρα Στοιχειώδης Γεωμετρία Διεθνής Μαθημαnκές Ολυμτπάδες 2.000 δρχ Gr. Mυncres 1 .500 δρχ. από Ανώτφη Σκοπιά 2.500 δρχ. 1959 - 1999: 6.000 δρχ. 9ου Πανελληνίου Συνεδρίου ΗΡDΝΟΣ ΑΛΕΞΑΝΔΡΕDΣ θεωρία Αριθμών 5.000 δρχ.

Το naλaι6τιρο τεύχη 6λων των tκδ6οtων nωλούντaι με τις τρέχοuσιι; nμέι; του 2001

Page 84: Ευκλειδης Β 41

Ράφια γεμάτα βιβλία, μυρωδιά ζεστού καφέ, μελωδικές

νότες, χώρος φιλόξενο.ς, ειδικά διαμορφωμένος για ευχά­

ριστη και ήρεμη ανάγνωση . Στα βιβλιοπωλεία " Β ΙΒΛΙΟρυθμός" θα

βρείτε όλες τις παλιές και νέες εκδόσεις όλων των εκδοτικών οίκων για όλα

τα θέματα: παιδικά, λογοτεχνία, ιστορία, ψυχολογία, κοινωνιολογία, εκπαίδευση, μεταφυσική,

φιλοσοφία, διδακτική , δοκίμια, οικολογία, ποίηση , μουσική , περιοδικά, ξενόγλωσσους τίτλους

καθώς και πλήθος επιλεγμένων cd . . . και κάθε μέρα ανάγνωση με τις μελωδίες του πιάνου.

ΘΕΣΣΑΛΟ Ν Ι ΚΗ : Β α σ . Η ρα κλείου 4 7 (απέναντι από το εμπορ ικό κέντρο) Τnλ. 031ο.ηο.226, Fax: 0310 .250. 972

ΑΘΗ ΝΑ: Ζωοδόχου Π nyriς 18 & Σόλωνος Τnλ. Ο Ι0 .33 . 0 1 .25 1 , Fax: 010.33 . 0 6 . 9 1 8